endocrine NCLEX, NCLEX Endocrine, NCLEX - ENDOCRINE, Lewis: Chapter 50: Endocrine Problems, Lewis: Chapter 50: Endocrine Problems, Lewis, Ch 50 - Endocrine, Lewis Ch. 48 Endocrine, Lewis Ch. 48 Endocrine System, Lewis Chapter 48, Endocrine, Lewis - E...

Ace your homework & exams now with Quizwiz!

The nursing instructor asks a student to describe the pathophysiology that occurs in Cushing's disease. Which statement by the student indicates an accurate understanding of this disorder?

"Cushing's disease is characterized by an oversecretion of glucocorticoid hormones."

The nurse is reinforcing discharge teaching with a client who has Cushing's syndrome. Which statement by the client indicates that the instructions related to dietary management were understood?

"I can eat foods that contain potassium."

The nurse has reinforced instructions to the client with hyperparathyroidism regarding home care measures related to exercise. Which statement by the client indicates a need for further teaching? Select all that apply.

"I need to limit playing football to only the weekends." "I should exercise in the evening to encourage a good sleep pattern."

The nurse is reinforcing home care instructions to a client with a diagnosis of Cushing's syndrome. Which client statement reflects a need for further teaching?

"I need to read the labels on any over-the-counter medications I purchase."

What disorder are you able to palpate the thyroid?

hyperthyroidism

graves disease is also known as

hyperthyroidism and causes insomnia and anxiety

chvostek's sign & trousseau sign are both expected findings of

hypocalcemia in which there is evidence of latent tetany

What electrolyte are you monitoring with a thyroidectomy and why?

hypocalcemia, b.c. it is possible that the parathyroid was also removed

What electrolyte should be monitored post opp parathyroidectomy?

hypocalcemia- rigid muscle tone, spasms, seizures

With SIADH, hypersecretion of ADH causes

hyponatremia

Central diabetes insipidus may be caused by damage to the

hypothalamus (or pituitary gland).

How do you determine insulin dose?

increase dose until blood sugar is normal & there is no glucose or ketones in the urine

expected finding for patient with diabetes insipidus

increased Hct level

what possible complications affect baby born from a mom with gestational diabetes is _____?

increased birth wt & hypoglycemia

PTU treatment goal

increased weight

HHNK has increased risk for ____?

infection

Diabetic Ketoacidosis can be caused by ___

infection, illness, or skipping insulin

Metabolic syndrome includes

insulin resistance, obesity, increased triglycerides, decreased HDL, increased B/P, CAD

With type 2 DM, the pancreas produces

insulin, but it's not enough and/or the body cannot use it properly (insulin resistance). Type 1 DM is when beta cells stop producing insulin.

You need ___ to carry glucose out of the vascular space and into the cell, without it, the glucose builds up in the vascular space, this causes the blood to become ___

insulin, hypertonic

What do you need to make hormones?

iodine (dietary, table salt)

what is HHNK

just like DKA with *no ketones* no acidosis & "hugely *huge*" numbers for *blood glucose*

Management of mild hyperparathyroidism includes increasing oral fluid intake to prevent

kidney stones from developing

for patient using a thermometer to test bathwater due to diabetic neuropathy, the temp should be

less than 43.3 C or 110 F

Lantus is considered ___ acting?

long, should not be mixed

In cushings pt, their serum K+ will be ___

low

A client is admitted to the emergency department and they diagnose of myxedema coma is made. What nursing action with the nurse prepared to carry out initially

maintain a patent airway

purpose of 24 hour urine collection in patient with pheochromocytoma

measure levels of catecholamines and metanephrines

myxedema coma causes

medical emergency -infection -drugs (opioids, tranquilizers, barbiturates) -exposure to cold -trauma

DKA = what kind of acidosis

metabolic

Type 2 diabetes should be evaluated for ____ syndrome

metabolic

Although a number of oral medications are available to treat type 2 DM,

metformin is usually one of the first to be used. It lowers the amount of glucose absorbed from food, lowers the amount of glucose produced by the liver, and increases the body's response to insulin.

with gestational diabetes, a mother will need ___ insulin than normal

more (2-3x more)

s/s of hyperkalemia

muscle twitching, weakness then flaccid paralysis

What are s/s of graves Disease?

nervous, wt loss, sweaty/hot, exophthalmos (bludgin eyes), decrease attention span, increase appetite, irritable, increase BP, thyroid larger

Pts on synthroid should promptly report:

- chest pain - weight loss - nervousness - tremors - insomnia

What dx indicate thyroid storm?

- decreased TSH and ^ free T4 - RAIU (radioactive uptake test)

Treatment of thyroid storm is aimed at:

- decreasing circulating thyroid hormone levels - correcting sx

Most people are prescribed metformin (Glucophage) when

newly diagnosed with type 2 diabetes.

patient teaching to prepare for vanillylmandellic acid test

avoid coffee, tea, bananas, chocolate for 2 to 3 days prior to the test

What medications is a patient given before a thyroidectomy? Why? Pre-op teaching?

- drugs to put into euthyroid state. (PTU) - iodine to restrict vascularization - support head

What is dx of hypothyroidism?

- elevated or normal TSH - Low T4

Nonsurgical tx includes:

- exercise - high fluid intake - mod. calcium intake, phosphate supplement

What are the 3 classess of adrenal cortex steroid hormones? Give an example of each:

- glucocortocoids - cortisol - mineralocorticoids - aldosterone - androgens - epi/norepi

What are potential complications of the removal of an adenoma on the parathyroid?

- hemorrhage - F/E imbalance - tetany (sudden dec. of Ca++)

What nutritional tx is appropriate for a patient w/ hyperthyroidism? (3)

- high calorie (4,000-5,000/d) - 6 full meals/day w/ snacks - increase protein, carbs, minerals, vitamins

Hypothyroidism is an insidious, nonspecific slowing of body processes w/ a mirage of sx, including:

- hoarseness - fatiuge - cold, dry skin - hair loss - constipation - cold intolerance - personality and mental changes

DI is characterized by:

- hypernatremia (fluid loss) - polydipsia - polyuria (5-20 >/day); s.g. <1.005 - elevated serum osm. >295 mOsm/kg

SIADH is associated w/ what electrolyte imbalance? S/S of SIADH?

- hyponatremia (muscle cramping, weakness, pain, twitching - thirst - fatigue - dyspnea on exertion - decreased output - increased body weight w/out edema

Graves disease can be triggered by:

- low iodine - infection - stress 75% of hyperthyroid cases

Drug tx for hyperparathyroidism includes drugs that:

- lower calcium (eg. biophosphonates) - estrogens - oral phosphates - diuretics - calcimimetics

What ax should be done q2h?

- monitor s/s of hemorrhage - tracheal compression (edema) - tetany - pain - vitals

What are s/s of hyperparathyroidism?

- muscle weakness - dec. appetite - fatigue - osteoporosis - fractures - kidney stones - constipation

What are sx of hypercalcemia?

- muscle weakness - lack of appetite - fatigue - kidney stones

What ax findings are consistent w/ hyperthyroid?

- palpation of goiter (maybe) - bruits at thyroid (^ blood supply) - ophthalmopathy

List sx of DI: (3)

- polydipsia - fatigue/insomnia r/t nocturia - weakness

What's the difference b/n primary and secondary hypothyroidism?

- primary - r/t thyroid or hormone sythesis - secondary - r/t pituitary or hypothalamic dysfunction

What are s/s of Addison's?

- progressive weakness - fatigue - wt. loss - hyperpigmintation

What physiologic effect does Graves have?

- pt develops antibodies to TSH receptor on thyroid - stimulates thyroid to release T3, T4, or both

List clinical manifestations of pheochromocytoma?

- severe, pounding headache - tacycardia w/ palpitations - profuse sweating - unexplained abd. or chest pain

How is pheochromocytoma treated? Potential complications?

- surgical removal of tumor - PC: hypertensive encephalopathy, DM, cardiomyopathy, death

What are s/s of hypoparathyroidism? Tx:

- tetany - laryngospasm - calcium supplements

Why is RAI used? What is a complication?

- to depress hyperthyroid - 80% have post-tx hypothyroidism

List s/s of Cushings:

- truncal obesity, moon face, buffalo hump - hirsutism in women - easy bruising, prolonged wound healing - insomnia, amenorrhea, impotence - ^ BP, unexplained hypokalemia

Tx for Addison's focuses on:

- tx of underlying cause - hormone tx w/ ^ doses during stressful situations - increase dietary salt

What will you see in an ax of a pt w/ DI? (7)

-hypernatremia - weight loss - constipation - poor skin turgor - low BP, tachycardia - shock - irritability, mental dullness, coma

for a patient taking desmopressin in the treatment of diabetes insipidus, adjustment is needed in the medication dosage for

nocturia

Addison's disease patients have____?

not enough glucocorticoids, mineralcorticoids or sex hormones

Patho of DKA

not enough insulin --> high blood sugar --> polyuria, polydipsia, polyphagia -> fat breakdown (acidosis) --> Kussmaul Resp (blow off CO2 to compensate for metabolic acidosis --> client becomes more acidotic as the LOC goes down

adverse effects of long term corticosteroid therapy include

osteroperosis, moon face, increased risk of infection, weigt gain, blurred vision

Graves' disease - an autoimmune disorder that leads to

overactivity of the thyroid gland.

in patient with pheochromocytoma avoid

palpating the abdomen and provide darkened room to promote rest

hyperthyroid expected findings

palpitations, heat intolerance, tremor, diaphoresis, insomnia

when should you eat related to diabetes?

peak of insulin

s/s of diabetes type 1

polyuria: excessive voiding polydipsia: excessive thirst polyphagia: excessive hunger

After thyroidectomy teach the patient to report and____?

preasure

What happens in nephropathy?

problems with impotence & decreased sensation, nerve damage, foot & leg problems, pain, paresthesia, numbness, burn themselves a lot, bladder incontinence (may have retention), gastroperesis ( delayed stomach emptying so increased risk for aspiration)

What medications stops the thyroid from making thyroid hormone?

propylthiouracil & methimazole monitor CBC with differential, TSH levels, PTT - report bruising or bleeding, avoid shellfish/iodine products

Why wouldn't you find protein in the cushing's pt's urine?

protein is a big molecule & cant go through glomeruli unless glomeruli is damaged.

why would you limit protein?

pt has renal disease

what does PTH do

pulls calcium from bone

hyperosmolar hyperglycemic state (HHS)

rare but deadly metabolic state is more common in the elderly with type 2 DM. HHS is characterized by hyperglycemia and severe dehydration without ketoacidosis.

A nurse is collecting data regarding a client after a thyroidectomy and notes that the client has developed hoarseness and a week voice. What nursing action is appropriate

reassure the client that is usually a temporary condition

During tx for hyperthyroidism or thyroid storm, the nurse should monitor for:

s/s of hypothyroidism: - bruising - cold intolerance

How is Iodine compounds given and WHY?

should be given with milk or juice and use a straw. WHY? they can stain the teeth

GI speed for a pt with myxedema

slow

Treatment for type 2 diabetes?

start with diet & exercise, than add oral agents, insulin as needed

what do oral hypolycemic meds do?

stimulate pancreas

What should a patient stop taking prior to thyroid scan?

stop taking all iodine medication (amiodarone) 1 week before

Adrenal glands handle ____?

stress

Why do diabetics have trouble with feet?

sugar damages vascular system resulting in poor circulation

how does steroids affect immune system

suppresses it

thyroid storm s/s

tachycardia, hypertension, fever, sweating, restlessness, tremors, abdominal pain

The water deprivation test DETERMINES

the cause of polydipsia and polyuria - central diabetes insipidus (DI), nephrogenic DI, or psychogenic polydipsia.

Myxedema coma

the extreme manifestation of hypothyroidism; uncommon but potentially lethal.

a somogyi effect iscaused by

the release of stress hormones to counter low blood sugar during the night

thyroid storm usually follows

thyroid removal or acute stress, trauma, infection or hyperthyroidism

an indication of hypocalcemia following a thyroidectomy

tingling in hands & feet (paresthesia); cool, clammy, nervous, tachycardia, anxiety, confusion

what must you keep at bedside with thyroid surgery

trach set

A nurses caring for a client after a thyroidectomy and notes that calcium gluconate is prescribed for the client. The nurse determines that this medicine has been prescribed to

treat hypoglycemic tetany

pheochromocytoma is a

tumor of the adrenal glands

Extra PTH results in hypercalcemia. There is also increased calcium in the

urine, which may cause kidney stones. If the client's serum calcium levels are only slightly elevated, s/he should drink plenty of fluids to minimize the risk of kidney stones.

If you enter a diabetic patient's room and they are unconscious , how should you treat them? then what and what

use D50W w/ large bore needle (hard to push), if no IV access use injectable glucagon (GlucaGen) given IM

sildenafil (Viagra) is prescribed for a client to treat erectile dysfunction. A nurse reviews the clients medical record and would question the prescription if which of the following is noted in the clients history

use of nitroglycerin

test used to screen for pheochromocytoma

vanillylmandellic acid test

Why would you hold fludrocortisone dose?

wt gain or loss of more than 3 lbs, medications needs to be adjusted, wt loss indicates FVD because pt is not retaining Na+ and H2O risk for shock

s/s of addison's disease include

•Muscle weakness, fatigue, •Weight loss, decreased appetite, hyperpigmentation (dark), Low BP, fainting, Salt craving, hypoglycemia, NDV, muscle or joint pains, Irritability, depression, Body hair loss or sexual dysfunction in women

myxedema coma symptoms

-mental sluggishness -drowsiness -lethargy -low temperature -hyponatremia -hypotension, bradycardia -hypoventilation -cardiovascular collapse -lactic acidosis -generalized non-pitting edema

Myxedema

-severe, long-standing hypothyroidism -alters the physical appearance of the skin and subcutaneous tissues with puffiness, facial and periorbital edema, and a mask like affect -accumulation of hydrophilic mucopolysaccharides in the dermis and other tissues

Myxedema coma treatment

-thyroid hormone -glucocorticoids -IV fluid -blankets -vasopressor

An indication of Chvostek' sign is: Answers: A. Twitching of the lips after tapping the face B. Elevated blood sugar after glucose infusion C. Inability to hold one's arms straight D. Spasms of the hand after blood circulation is cut off

. A Twitching of the lips after tapping the face in the right place is an indication of Chvostek's sign and a sign of hypocalcaemia. Spasms of the hand are associated with Trousseau's sign.

What is a hormone secreted from the posterior lobe of the pituitary gland? Answers: A. LH B. MSH C. ADH D. GnRH

. C ADH is secreted from the posterior pituitary. LH comes from the anterior pituitary, MSH from the intermediate. GnRH is released from the hypothalamus.

A client with Graves' disease experiences a thyroid storm and has tachycardia and hypertension. What medication is most likely to be used? Answers: A. Levofloxcin B. Chlorothiazide C. Percocet D. Propylthiouracil

. D Propylthiouracil is a commonly used medication for treating hyperthyroidism. Levofloxacin is an antibiotic, chlorothiazide is a diuretic, and Percocet a painkiller.

. The nurse assessing a female client with Cushing's syndrome would expect to note which of the following? a) hirsutism b) hypotension c) hypoglycemia d) pallor

1) A - An increased production of androgens that accompanies a rise in cortisol levels with Cushing's syndrome produces hirsutism and acne in women. Other clinical findings of Cushing's syndrome include hypertension caused by sodium retention, impaired glucose tolerance or diabetes mellitus caused by cortisol's anti-insulin effect and ability to enhance gluconeogenesis, and skin changes including bruising and purplish red striae caused by protein catabolism.

Identify five hormonees that are replaced when panhypopituitarism from radiation therapy or total hypophysectomy as treatment for pituitary tumors

1) cortisol 2) thyroid 3) vasopressin/ ADH analog 4) Growth hormone 5) sex hormones: testosterone, follicle stimulating hormone (FSH), and leutinizing hormone (LH) if fertility is desired; estrogen/progesterone if fertility is not an issue

A client with diabetes mellitus who takes insulin is seen in the health care clinic. The client tells the nurse that after giving the injection, the insulin seems to leak through the skin. The nurse can appropriately determine the problem by asking the client which of the following?

1. "Are you placing an air bubble in the syringe before injection?" 2. "Are you using a 1-inch needle to give the injection?" 3. "Are you aspirating before you inject the insulin?" *4. "Are you rotating the injection site?"* *rationale* The client should be instructed that insulin injection sites should be rotated within one anatomical area before moving to another. This rotation process promotes uniform absorption of insulin and reduces the chances of irritation. Options 1, 2, and 3 are not associated with the condition (skin leakage of insulin) presented in the question.

A nursing instructor asks a student to describe the pathophysiology that occurs in Cushing's disease. Which statement by the student indicates an accurate understanding of this disorder?

1. "Cushing's disease is characterized by an oversecretion of insulin." *2. "Cushing's disease is characterized by an oversecretion of glucocorticoid hormones."* 3. "Cushing's disease is characterized by an undersecretion of corticotropic hormones." 4. "Cushing's disease is characterized by an undersecretion of glucocorticoid hormones." *rationale* Cushing's syndrome is characterized by an oversecretion of glucocorticoid hormones. Addison's disease is characterized by the failure of the adrenal cortex to produce and secrete adrenocortical hormones. Options 1 and 4 are inaccurate regarding Cushing's syndrome.

A nurse is collecting data on a client with hyperparathyroidism. Which of the following questions would elicit the accurate information about this condition from the client?

1. "Do you have tremors in your hands?" *2. "Are you experiencing pain in your joints?"* 3. "Have you had problems with diarrhea lately?" 4. "Do you notice swelling in your legs at night?" *rationale* Hyperparathyroidism causes an oversecretion of parathyroid hormone (PTH), which causes excessive osteoblast growth and activity within the bones. When bone reabsorption is increased, calcium is released from the bones into the blood, causing hypercalcemia. The bones suffer demineralization as a result of calcium loss, leading to bone and joint pain, and pathological fractures.

Which of the following symptoms is not typical of Cushing's syndrome? Answers: A. Osteoporosis B. Weight loss C. Diabetes D. Mood instability

B Cushing's syndrome tends to produce rapid weight gain, not weight loss.

What are some functions of cortisol?

Cortisol is necessary to maintain life. Its functions include regulation of blood glucose concentration, inhibition of inflammatory action, and support in response to stress.

The nurse is instructing a client with Addison's disease about a newly prescribed medication, fludrocortisone acetate (Florinef). Which statement by the client indicates a need for further teaching?

"I will be glad to gain weight."

A client with pheochromocytoma is scheduled for surgery and says to the nurse, "I'm not sure that surgery is the best thing to do." Which response by the nurse is appropriate?

"You have concerns about the surgical treatment for your condition?"

A nurse is reviewing discharge teaching with a client who has Cushing's syndrome. Which statement by the client indicates that the instructions related to dietary management were understood? 1. "I can eat foods that contain potassium." 2. "I will need to limit the amount of protein in my diet." 3. "I am fortunate that I can eat all the salty foods I enjoy." 4. "I am fortunate that I do not need to follow any special diet."

*1. "I can eat foods that contain potassium."* *rationale* A diet that is low in calories, carbohydrates, and sodium but ample in protein and potassium content is encouraged for a client with Cushing's syndrome. Such a diet promotes weight loss, the reduction of edema and hypertension, the control of hypokalemia, and the rebuilding of wasted tissue.

A nurse is reviewing discharge teaching with a client who has Cushing's syndrome. Which statement by the client indicates that the instructions related to dietary management were understood?

*1. "I can eat foods that contain potassium."* 2. "I will need to limit the amount of protein in my diet." 3. "I am fortunate that I can eat all the salty foods I enjoy." 4. "I am fortunate that I do not need to follow any special diet." *rationale* A diet that is low in calories, carbohydrates, and sodium but ample in protein and potassium content is encouraged for a client with Cushing's syndrome. Such a diet promotes weight loss, the reduction of edema and hypertension, the control of hypokalemia, and the rebuilding of wasted tissue.

Which statement by the client would cause the nurse to suspect that the thyroid test results drawn on the client this morning may be inaccurate? 1. "I had a radionuclide test done 3 days ago." 2. "When I exercise I sweat more than normal." 3. "I drank some water before the blood was drawn." 4. "That hamburger I ate before the test sure tasted good."

*1. "I had a radionuclide test done 3 days ago."* *rationale* Option 1 indicates that a recent radionuclide scan had been performed. Recent radionuclide scans performed before the test can affect thyroid laboratory results. No food, fluid, or activity restrictions are required for this test, so options 2, 3, and 4 are incorrect.

Which statement by the client would cause the nurse to suspect that the thyroid test results drawn on the client this morning may be inaccurate?

*1. "I had a radionuclide test done 3 days ago."* 2. "When I exercise I sweat more than normal." 3. "I drank some water before the blood was drawn." 4. "That hamburger I ate before the test sure tasted good." *rationale* Option 1 indicates that a recent radionuclide scan had been performed. Recent radionuclide scans performed before the test can affect thyroid laboratory results. No food, fluid, or activity restrictions are required for this test, so options 2, 3, and 4 are incorrect.

A nurse is assigned to care for a client at home who has a diagnosis of type 1 diabetes mellitus. When the nurse arrives to care for the client, the client tells the nurse that she has been vomiting and has diarrhea. Which additional statement by the client indicates a need for further teaching?

*1. "I need to stop my insulin."* 2. "I need to increase my fluid intake." 3. "I need to call my health care provider." 4. "I need to monitor my blood glucose every 4 to 6 hours." *rationale* When a client with diabetes is unable to eat normally because of illness, the client should still take the prescribed insulin or oral medication. Additional fluids should be consumed and a call placed to the health care provider. The client should monitor the blood glucose levels every 4 to 6 hours.

A nurse has reinforced home care measures to a client diagnosed with diabetes mellitus regarding exercise and insulin administration. Which statement by the client indicates a need for further instruction?

*1. "I should perform my exercise at peak insulin time."* 2. "I should always carry a quick-acting carbohydrate when I exercise." 3. "I should always wear a Medic-Alert bracelet especially when I exercise." 4. "I should avoid exercising at times when a hypoglycemic reaction is likely to occur." *rationale* The client should be instructed to avoid exercise at peak insulin time because this is when a hypoglycemic reaction is likely to occur. If exercise is performed at this time, the client should be instructed to eat an hour before the exercise and drink a carbohydrate liquid. Options 2, 3, and 4 are correct statements regarding exercise, insulin, and diabetic control.

A nurse is assigned to assist in caring for a client admitted to the emergency department with diabetic ketoacidosis (DKA). In the acute phase, the priority nursing action is to plan to prepare to:

*1. Administer intravenous (IV) regular insulin.* 2. Administer IV 5% dextrose. 3. Correct the acidosis. 4. Apply an electrocardiogram (ECG) monitor. *rationale* Lack (absolute or relative) of insulin is the primary cause leading to DKA. Treatment consists of IV fluids (normal saline initially), regular insulin administration, and potassium replacement, followed by correcting the acidosis. An ECG monitor may be applied but is not the priority in this situation.

A client has just been admitted with a diagnosis of myxedema coma. If all of the following interventions were prescribed, the nurse would place highest priority on completing which of the following first? 1. Administering oxygen 2. Administering thyroid hormone 3. Warming the client 4. Giving fluid replacement

*1. Administering oxygen* *rationale* As part of maintaining a patent airway, oxygen would be administered first. This would be quickly followed by fluid replacement, keeping the client warm, monitoring vital signs, and administering thyroid hormones.

A client has just been admitted with a diagnosis of myxedema coma. If all of the following interventions were prescribed, the nurse would place highest priority on completing which of the following first?

*1. Administering oxygen* 2. Administering thyroid hormone 3. Warming the client 4. Giving fluid replacement *rationale* As part of maintaining a patent airway, oxygen would be administered first. This would be quickly followed by fluid replacement, keeping the client warm, monitoring vital signs, and administering thyroid hormones.

A nurse is assisting in preparing a care plan for a client with diabetes mellitus who has hyperglycemia. The nurse focuses on which potential problem for this client?

*1. Dehydration* 2. The need for knowledge about the causes of hyperglycemia 3. Lack of knowledge about nutrition 4. Inability of family to cope with the client's diagnosis *rationale* Increased blood glucose will cause the kidneys to excrete the glucose in the urine. This glucose is accompanied by fluids and electrolytes, causing an osmotic diuresis that leads to dehydration. This fluid loss must be replaced when it becomes severe. Options 2, 3, and 4 may be concerns at some point but are not the priority with hyperglycemia.

A nurse is collecting data on a client with a diagnosis of hypothyroidism. Which of these behaviors, if present in the client's history, would the nurse determine as being likely related to the manifestations of this disorder? 1. Depression 2. Nervousness 3. Irritability 4. Anxiety

*1. Depression* *rationale* Hypothyroid clients experience a slow metabolic rate, and its manifestation includes apathy, fatigue, sleepiness, and depression. Options 2, 3, and 4 identify the clinical manifestations of hyperthyroidism.

A nurse is collecting data on a client with a diagnosis of hypothyroidism. Which of these behaviors, if present in the client's history, would the nurse determine as being likely related to the manifestations of this disorder?

*1. Depression* 2. Nervousness 3. Irritability 4. Anxiety *rationale* Hypothyroid clients experience a slow metabolic rate, and its manifestation includes apathy, fatigue, sleepiness, and depression. Options 2, 3, and 4 identify the clinical manifestations of hyperthyroidism.

A nurse is reinforcing dietary instructions to a client newly diagnosed with diabetes mellitus. The nurse instructs the client that it is best to:

*1. Eat meals at approximately the same time each day.* 2. Adjust mealtimes depending on blood glucose levels. 3. Vary mealtimes if insulin is not administered at the same time every day. 4. Avoid being concerned about the time of meals as long as snacks are taken on time. *rationale* Mealtimes must be approximately the same time each day to maintain a stable blood glucose level. The client should not be instructed that mealtimes are varied, depending on blood glucose levels or insulin administration. Mealtimes should not be adjusted based on blood glucose levels or snacks.

A client with a diagnosis of diabetic ketoacidosis (DKA) is being treated in the emergency department. Which of the following findings would the nurse expect to note as confirming this diagnosis?

*1. Elevated blood glucose and low plasma bicarbonate* 2. Decreased urine output 3. Increased respirations and an increase in pH 4. Coma *rationale* In DKA, the arterial pH is less than 7.35, plasma bicarbonate is less than 15 mEq/L, the blood glucose level is higher than 250 mg/dL, and ketones are present in the blood and urine. The client would be experiencing polyuria and Kussmaul's respirations. Coma may occur if DKA is not treated, but coma would not confirm the diagnosis.

A nurse is reviewing a plan of care for a client with Addison's disease. The nurse notes that the client is at risk for dehydration and suggests nursing interventions that will prevent this occurrence. Which nursing intervention is an appropriate component of the plan of care? *Select all that apply.* 1. Encouraging fluid intake of at least 3000 mL/day 2. Encouraging an intake of low-protein foods 3. Monitoring for changes in mental status 4. Monitoring intake and output 5. Maintaining a low-sodium diet

*1. Encouraging fluid intake of at least 3000 mL/day* *3. Monitoring for changes in mental status* *4. Monitoring intake and output* *rationale* The client at risk for deficient fluid volume should be encouraged to eat regular meals and snacks and to increase the intake of sodium, protein, and complex carbohydrates. Oral replacement of sodium losses is necessary, and maintenance of adequate blood glucose levels is required.

A nurse is reviewing a plan of care for a client with Addison's disease. The nurse notes that the client is at risk for dehydration and suggests nursing interventions that will prevent this occurrence. Which nursing intervention is an appropriate component of the plan of care? *Select all that apply.*

*1. Encouraging fluid intake of at least 3000 mL/day* 2. Encouraging an intake of low-protein foods *3. Monitoring for changes in mental status* *4. Monitoring intake and output* 5. Maintaining a low-sodium diet *rationale* The client at risk for deficient fluid volume should be encouraged to eat regular meals and snacks and to increase the intake of sodium, protein, and complex carbohydrates. Oral replacement of sodium losses is necessary, and maintenance of adequate blood glucose levels is required.

A nurse is monitoring a client who has been newly diagnosed with diabetes mellitus for signs of complications. Which of the following, if exhibited by the client, would indicate hyperglycemia and thus warrant health care provider notification?

*1. Polyuria* 2. Bradycardia 3. Diaphoresis 4. Hypertension *rationale* The classic symptoms of hyperglycemia include polydipsia, polyuria, and polyphagia. Options 2, 3, and 4 are not signs of hyperglycemia.

A client with Addison's disease asks the nurse how a newly prescribed medication, fludrocortisone acetate (Florinef), will improve the condition. When formulating a response, the nurse should incorporate that a key action of this medication is to: 1. Help restore electrolyte balance. 2. Make the body produce more cortisol. 3. Replace insufficient circulating estrogens. 4. Alter the body's immune system functioning.

*1. Help restore electrolyte balance.* *rationale* Fludrocortisone acetate is a long-acting oral medication with mineralocorticoid and moderate glucocorticoid activity. It is prescribed for the long-term management of Addison's disease. Mineralocorticoids cause renal reabsorption of sodium and chloride ions and the excretion of potassium and hydrogen ions. These actions help restore electrolyte balance in the body. The other options are incorrect.

A client with Addison's disease asks the nurse how a newly prescribed medication, fludrocortisone acetate (Florinef), will improve the condition. When formulating a response, the nurse should incorporate that a key action of this medication is to:

*1. Help restore electrolyte balance.* 2. Make the body produce more cortisol. 3. Replace insufficient circulating estrogens. 4. Alter the body's immune system functioning. *rationale* Fludrocortisone acetate is a long-acting oral medication with mineralocorticoid and moderate glucocorticoid activity. It is prescribed for the long-term management of Addison's disease. Mineralocorticoids cause renal reabsorption of sodium and chloride ions and the excretion of potassium and hydrogen ions. These actions help restore electrolyte balance in the body. The other options are incorrect.

In planning nutrition for the client with hypoparathyroidism, which diet would be appropriate? 1. High in calcium and low phosphorous 2. Low in vitamins A, D, E, and K 3. High in sodium with no fluid restriction 4. Low in water and insoluble fiber

*1. High in calcium and low phosphorous* *rationale* Hypocalcemia is the end result of hypoparathyroidism resulting from either a lack of parathyroid hormone (PTH) secretion or ineffective PTH influence on tissue. Calcium is the major controlling factor of PTH secretion. Because of this, the diet needs to be high in calcium but low in phosphorus because these two electrolytes must exist in inverse proportions in the body. The other options are not dietary interventions with hypoparathyroidism.

In planning nutrition for the client with hypoparathyroidism, which diet would be appropriate?

*1. High in calcium and low phosphorous* 2. Low in vitamins A, D, E, and K 3. High in sodium with no fluid restriction 4. Low in water and insoluble fiber *rationale* Hypocalcemia is the end result of hypoparathyroidism resulting from either a lack of parathyroid hormone (PTH) secretion or ineffective PTH influence on tissue. Calcium is the major controlling factor of PTH secretion. Because of this, the diet needs to be high in calcium but low in phosphorus because these two electrolytes must exist in inverse proportions in the body. The other options are not dietary interventions with hypoparathyroidism.

A client is admitted to the hospital with a diagnosis of diabetic ketoacidosis (DKA). The initial serum glucose level was 950 mg/dL. Intravenous (IV) insulin was started along with rehydration with IV normal saline. The serum glucose level is now 240 mg/dL. The nurse who is assisting in caring for the client obtains which of the following items, anticipating a health care provider's prescription?

*1. IV infusion containing 5% dextrose* 2. NPH insulin and a syringe for subcutaneous injection 3. An ampule of 50% dextrose 4. Phenytoin (Dilantin) for prevention of seizures *rationale* During management of DKA, when the blood glucose level falls to 300 mg/dL, the infusion rate is reduced and 5% dextrose is added to maintain a blood glucose level of about 250 mg/dL, or until the client recovers from ketosis. NPH insulin is not used to treat DKA; 50% dextrose is used to treat hypoglycemia. Phenytoin is not a normal treatment measure in DKA.

A client with hypoparathyroidism has hypocalcemia. The nurse avoids giving the client the prescribed vitamin and calcium supplement with which of the following liquids? 1. Milk 2. Water 3. Iced tea 4. Fruit juice

*1. Milk* *rationale* Milk products are high in phosphates, which should be avoided by a client with hypoparathyroidism. Otherwise, calcium products are best absorbed with milk because the vitamin D in the milk promotes calcium absorption.

A client with hypoparathyroidism has hypocalcemia. The nurse avoids giving the client the prescribed vitamin and calcium supplement with which of the following liquids?

*1. Milk* 2. Water 3. Iced tea 4. Fruit juice *rationale* Milk products are high in phosphates, which should be avoided by a client with hypoparathyroidism. Otherwise, calcium products are best absorbed with milk because the vitamin D in the milk promotes calcium absorption.

A nurse notes in the medical record that a client with Cushing's syndrome is experiencing fluid overload. Which interventions should be included in the plan of care? *Select all that apply.* 1. Monitoring daily weight 2. Monitoring intake and output 3. Maintaining a low-potassium diet 4. Monitoring extremities for edema 5. Maintaining a low-sodium diet

*1. Monitoring daily weight* *2. Monitoring intake and output* *4. Monitoring extremities for edema* *5. Maintaining a low-sodium diet* *rationale* The client with Cushing's syndrome experiencing fluid overload should be maintained on a high-potassium and low-sodium diet. Decreased sodium intake decreases renal retention of sodium and water. Monitoring weight, intake, output, and extremities for edema are all appropriate interventions for such a nursing diagnosis.

A nurse notes in the medical record that a client with Cushing's syndrome is experiencing fluid overload. Which interventions should be included in the plan of care? *Select all that apply.*

*1. Monitoring daily weight* *2. Monitoring intake and output* 3. Maintaining a low-potassium diet *4. Monitoring extremities for edema* *5. Maintaining a low-sodium diet* *rationale* The client with Cushing's syndrome experiencing fluid overload should be maintained on a high-potassium and low-sodium diet. Decreased sodium intake decreases renal retention of sodium and water. Monitoring weight, intake, output, and extremities for edema are all appropriate interventions for such a nursing diagnosis.

When a nurse notes that a client with type 1 diabetes mellitus has lipodystrophy on both upper thighs, what information should the nurse obtain from the client?

*1. Plan of injection rotation* 2. Consistency of aspiration 3. Preparation of the injection site 4. Angle at which the medication is administered *rationale* Lipodystrophy (i.e., the hypertrophy of subcutaneous tissue at the injection site) occurs in some diabetic clients when the same injection sites are used for prolonged periods of time. Thus, clients are instructed to adhere to a rotating injection site plan to avoid tissue changes. Preparation of the site, aspiration, and the angle of insulin administration do not produce tissue damage.

A nurse is caring for a client with a diagnosis of hypoparathyroidism. The nurse reviews the laboratory results drawn on the client and notes that the calcium level is extremely low. The nurse would expect to note which of the following on data collection of the client? 1. Positive Trousseau's sign 2. Negative Chvostek's sign 3. Unresponsive pupils 4. Hyperactive bowel sounds

*1. Positive Trousseau's sign* *rationale* Hypoparathyroidism is related to a lack of parathyroid hormone secretion or to a decreased effectiveness of parathyroid hormone on target tissues. The end result of this disorder is hypocalcemia. When serum calcium levels are critically low, the client may exhibit positive Chvostek's and Trousseau's signs, which indicate potential tetany. Options 2, 3, and 4 are not related to the presence of hypocalcemia.

A nurse is caring for a client with a diagnosis of hypoparathyroidism. The nurse reviews the laboratory results drawn on the client and notes that the calcium level is extremely low. The nurse would expect to note which of the following on data collection of the client?

*1. Positive Trousseau's sign* 2. Negative Chvostek's sign 3. Unresponsive pupils 4. Hyperactive bowel sounds *rationale* Hypoparathyroidism is related to a lack of parathyroid hormone secretion or to a decreased effectiveness of parathyroid hormone on target tissues. The end result of this disorder is hypocalcemia. When serum calcium levels are critically low, the client may exhibit positive Chvostek's and Trousseau's signs, which indicate potential tetany. Options 2, 3, and 4 are not related to the presence of hypocalcemia.

A nurse is monitoring the results of periodic serum laboratory studies drawn on a client with diabetic ketoacidosis (DKA) receiving an insulin infusion. The nurse determines that which of the following values needs to be reported?

*1. Potassium 3.1 mEq/L* 2. Calcium 9.2 mg/dL 3. Sodium 137 mEq/L 4. Serum osmolality 288 mOsm/kg H2O *rationale* The client with DKA initially becomes hyperkalemic as potassium leaves the cells in response to lowered pH. Once fluid replacement and insulin therapy are started, the potassium level drops quickly. This occurs because potassium is carried into the cells along with glucose and insulin and because potassium is excreted in the urine once rehydration has occurred. Thus the nurse carefully monitors the results of serum potassium levels and reports hypokalemia (option 1) promptly. The other laboratory values are within the normal ranges.

A client with diabetes mellitus visits the health care clinic. The client previously had been well controlled with glyburide (Diabeta), but recently, the fasting blood glucose has been running 180 to 200 mg/dL. Which of the following medications, if added to the client's regimen, may be contributing to the hyperglycemia?

*1. Prednisone* 2. Atenolol (Tenormin) 3. Phenelzine (Nardil) 4. Allopurinol (Zyloprim) *rationale* Prednisone may decrease the effect of oral hypoglycemics, insulin, diuretics, and potassium supplements. Options 2, a β-blocker, and 3, a monoamine oxidase inhibitor, have their own intrinsic hypoglycemic activity. Option 4 decreases urinary excretion of sulfonylurea agents, causing increased levels of the oral medications, which can lead to hypoglycemia.

A health care provider has prescribed propylthiouracil (PTU) for a client with hyperthyroidism, and the nurse assists in developing a plan of care for the client. A priority nursing measure to be included in the plan regarding this medication is to monitor the client for: 1. Signs and symptoms of hypothyroidism 2. Signs and symptoms of hyperglycemia 3. Relief of pain 4. Signs of renal toxicity

*1. Signs and symptoms of hypothyroidism* *rationale* Excessive dosing with propylthiouracil may convert the client from a hyperthyroid state to a hypothyroid state. If this occurs, the dosage should be reduced. Temporary administration of thyroid hormone may be required. Propylthiouracil is not used for pain and does not cause hyperglycemia or renal toxicity.

A health care provider has prescribed propylthiouracil (PTU) for a client with hyperthyroidism, and the nurse assists in developing a plan of care for the client. A priority nursing measure to be included in the plan regarding this medication is to monitor the client for:

*1. Signs and symptoms of hypothyroidism* 2. Signs and symptoms of hyperglycemia 3. Relief of pain 4. Signs of renal toxicity *rationale* Excessive dosing with propylthiouracil may convert the client from a hyperthyroid state to a hypothyroid state. If this occurs, the dosage should be reduced. Temporary administration of thyroid hormone may be required. Propylthiouracil is not used for pain and does not cause hyperglycemia or renal toxicity.

A nurse is preparing to discharge a client who has had a parathyroidectomy. When teaching the client about the prescribed oral calcium supplement, what information should the nurse include? 1. Take the calcium 30 to 60 minutes following a meal. 2. Avoid sunlight because it can cause skin color change. 3. Store the calcium in the refrigerator to maintain potency. 4. Check the pulse daily and hold the dosage if it is below 60 beats per minute.

*1. Take the calcium 30 to 60 minutes following a meal.* *rationale* Oral calcium supplements can be taken 30 to 60 minutes after meals to enhance their absorption and decrease gastrointestinal irritation. All the other options are unrelated to oral calcium therapy.

A nurse is preparing to discharge a client who has had a parathyroidectomy. When teaching the client about the prescribed oral calcium supplement, what information should the nurse include?

*1. Take the calcium 30 to 60 minutes following a meal.* 2. Avoid sunlight because it can cause skin color change. 3. Store the calcium in the refrigerator to maintain potency. 4. Check the pulse daily and hold the dosage if it is below 60 beats per minute. *rationale* Oral calcium supplements can be taken 30 to 60 minutes after meals to enhance their absorption and decrease gastrointestinal irritation. All the other options are unrelated to oral calcium therapy.

A nurse is collecting data from a client newly diagnosed with diabetes mellitus regarding the client's learning readiness. Which client behavior indicates to the nurse that the client is not ready to learn?

*1. The client complains of fatigue whenever the nurse plans a teaching session.* 2. The client asks if the spouse can attend the classes also. 3. The client asks for written materials about diabetes before class. 4. The client asks appropriate questions about what will be taught. *rationale* Physical symptoms can interfere with an individual's ability to learn and can indicate to the teacher that the learner lacks motivation to learn if the symptoms repeatedly recur when teaching is initiated. Options 2, 3, and 4 identify the client as actively seeking information.

A 26 year old female client presents with the symptom of unwanted facial hair. What of the following conditions is most likely? Answers: A. Graves' disease B. PCOS C. Hyperthyroidism D. Addison's disease

B PCOS is well known to cause hormonal irregularities in women which can result in hair growth.

A client with diabetes mellitus is being discharged following treatment for hyperglycemic hyperosmolar nonketotic syndrome (HHNS) precipitated by acute illness. The client states to the nurse, "I will call the doctor next time I can't eat for more than a day or so." The nurse plans care, understanding that which of the following accurately reflects this client's level of knowledge?

*1. The client needs immediate education before discharge.* 2. The client's statement is accurate, but knowledge should be evaluated further. 3. The client's statement is inaccurate, and the client should be scheduled for outpatient diabetic counseling. 4. The client requires follow-up teaching regarding the administration of insulin. *rationale* If the client becomes ill and cannot retain fluids or food for a period of 4 hours, the health care provider should be notified. The client's statement in this question indicates a need for immediate education to prevent HHNS, a life-threatening emergency situation.

An older client with a history of hyperparathyroidism and severe osteoporosis is hospitalized. The nurse caring for the client plans first to address which problem? 1. The possibility of injury 2. Constipation 3. Urinary retention 4. Need for teaching about the disorder

*1. The possibility of injury* *rationale* The client with severe osteoporosis as a result of hyperparathyroidism is at risk for injury as a result of pathological fractures that can occur from bone demineralization. The client may also have a risk for constipation from the disease process but this is a lesser priority than client safety. The client may or may not have urinary elimination problems, depending on other factors in the client's history. There is no information in the question to support whether the client needs teaching.

An older client with a history of hyperparathyroidism and severe osteoporosis is hospitalized. The nurse caring for the client plans first to address which problem?

*1. The possibility of injury* 2. Constipation 3. Urinary retention 4. Need for teaching about the disorder *rationale* The client with severe osteoporosis as a result of hyperparathyroidism is at risk for injury as a result of pathological fractures that can occur from bone demineralization. The client may also have a risk for constipation from the disease process but this is a lesser priority than client safety. The client may or may not have urinary elimination problems, depending on other factors in the client's history. There is no information in the question to support whether the client needs teaching.

A nurse is monitoring a client following a thyroidectomy for signs of hypocalcemia. Which of the following signs, if noted in the client, likely indicates the presence of hypocalcemia? 1. Tingling around the mouth 2. Negative Chvostek's sign 3. Flaccid paralysis 4. Bradycardia

*1. Tingling around the mouth* *rationale* Following a thyroidectomy, the nurse assesses the client for signs of hypocalcemia and tetany. Early signs include tingling around the mouth and fingertips, muscle twitching or spasms, palpitations or dysrhythmias, and positive Chvostek's and Trousseau's signs. Options 2, 3, and 4 are not signs of hypocalcemia.

A nurse is monitoring a client following a thyroidectomy for signs of hypocalcemia. Which of the following signs, if noted in the client, likely indicates the presence of hypocalcemia?

*1. Tingling around the mouth* 2. Negative Chvostek's sign 3. Flaccid paralysis 4. Bradycardia *rationale* Following a thyroidectomy, the nurse assesses the client for signs of hypocalcemia and tetany. Early signs include tingling around the mouth and fingertips, muscle twitching or spasms, palpitations or dysrhythmias, and positive Chvostek's and Trousseau's signs. Options 2, 3, and 4 are not signs of hypocalcemia.

Which client complaint would alert the nurse to a possible hypoglycemic reaction?

*1. Tremors* 2. Anorexia 3. Hot, dry skin 4. Muscle cramps *rationale* Decreased blood glucose levels produce automatic nervous system symptoms, which are classically manifested as nervousness, irritability, and tremors. Option 3 is more likely to occur with hyperglycemia. Options 2 and 4 are unrelated to the signs of hypoglycemia.

During routine postoperative assessment of a client who has undergone hypophysectomy, the client complains of thirst and frequent urination. Knowing the expected complications of this surgery, the nurse would next check the: 1. Urine specific gravity 2. Serum glucose 3. Respiratory rate 4. Blood pressure

*1. Urine specific gravity* *rationale* Following hypophysectomy, diabetes insipidus can occur temporarily because of antidiuretic hormone (ADH) deficiency. This deficiency is related to surgical manipulation. The nurse should assess specific gravity and notify the registered nurse if the results are less than 1.005. Although options 2, 3, and 4 may be components of the assessment, the nurse would next check urine specific gravity.

During routine postoperative assessment of a client who has undergone hypophysectomy, the client complains of thirst and frequent urination. Knowing the expected complications of this surgery, the nurse would next check the:

*1. Urine specific gravity* 2. Serum glucose 3. Respiratory rate 4. Blood pressure *rationale* Following hypophysectomy, diabetes insipidus can occur temporarily because of antidiuretic hormone (ADH) deficiency. This deficiency is related to surgical manipulation. The nurse should assess specific gravity and notify the registered nurse if the results are less than 1.005. Although options 2, 3, and 4 may be components of the assessment, the nurse would next check urine specific gravity.

A nurse has reinforced dietary instructions to a client with a diagnosis of hypoparathyroidism. The nurse instructs the client to include which of the following items in the diet? 1. Vegetables 2. Meat 3. Fish 4. Cereals

*1. Vegetables* *rationale* The client with hypoparathyroidism is instructed to follow a calcium-rich diet and to restrict the amount of phosphorus in the diet. The client should limit meat, poultry, fish, eggs, cheese, and cereals. Vegetables are allowed in the diet.

A nurse has reinforced dietary instructions to a client with a diagnosis of hypoparathyroidism. The nurse instructs the client to include which of the following items in the diet?

*1. Vegetables* 2. Meat 3. Fish 4. Cereals *rationale* The client with hypoparathyroidism is instructed to follow a calcium-rich diet and to restrict the amount of phosphorus in the diet. The client should limit meat, poultry, fish, eggs, cheese, and cereals. Vegetables are allowed in the diet.

A nurse is caring for a client with pheochromocytoma. The client is scheduled for an adrenalectomy. During the preoperative period, the priority nursing action would be to monitor the: 1. Vital signs 2. Intake and output 3. Blood urea nitrogen (BUN) level 4. Urine for glucose and acetone

*1. Vital signs* *rationale* Hypertension is the hallmark of pheochromocytoma. Severe hypertension can precipitate a brain attack (stroke) or sudden blindness. Although all of the options are accurate nursing interventions for the client with pheochromocytoma, the priority nursing action is to monitor the vital signs, particularly the blood pressure.

A nurse is caring for a client with pheochromocytoma. The client is scheduled for an adrenalectomy. During the preoperative period, the priority nursing action would be to monitor the:

*1. Vital signs* 2. Intake and output 3. Blood urea nitrogen (BUN) level 4. Urine for glucose and acetone *rationale* Hypertension is the hallmark of pheochromocytoma. Severe hypertension can precipitate a brain attack (stroke) or sudden blindness. Although all of the options are accurate nursing interventions for the client with pheochromocytoma, the priority nursing action is to monitor the vital signs, particularly the blood pressure.

A client with diabetes mellitus is scheduled to have a fasting blood glucose level determined in the morning. The nurse tells the client not to eat or drink after midnight. When the client asks for further information, the nurse clarifies by stating that which of the following would be acceptable to take before the test?

*1. Water* 2. Coffee without any milk 3. Tea without any sugar 4. Clear liquids such as apple juice *rationale* When a client is scheduled for a fasting blood glucose level, the client should not eat or drink anything except water after midnight. This is needed to ensure accurate test results, which form the basis for adjustments or continuance of treatment. Options 2, 3, and 4 are inaccurate, and the client should not consume these items before the test.

A nurse is collecting data on a client with hyperparathyroidism. Which of the following questions would elicit the accurate information about this condition from the client? 1. "Do you have tremors in your hands?" 2. "Are you experiencing pain in your joints?" 3. "Have you had problems with diarrhea lately?" 4. "Do you notice swelling in your legs at night?"

*2. "Are you experiencing pain in your joints?"* *rationale* Hyperparathyroidism causes an oversecretion of parathyroid hormone (PTH), which causes excessive osteoblast growth and activity within the bones. When bone reabsorption is increased, calcium is released from the bones into the blood, causing hypercalcemia. The bones suffer demineralization as a result of calcium loss, leading to bone and joint pain, and pathological fractures.

A nursing instructor asks a student to describe the pathophysiology that occurs in Cushing's disease. Which statement by the student indicates an accurate understanding of this disorder? 1. "Cushing's disease is characterized by an oversecretion of insulin." 2. "Cushing's disease is characterized by an oversecretion of glucocorticoid hormones." 3. "Cushing's disease is characterized by an undersecretion of corticotropic hormones." 4. "Cushing's disease is characterized by an undersecretion of glucocorticoid hormones."

*2. "Cushing's disease is characterized by an oversecretion of glucocorticoid hormones."* *rationale* Cushing's syndrome is characterized by an oversecretion of glucocorticoid hormones. Addison's disease is characterized by the failure of the adrenal cortex to produce and secrete adrenocortical hormones. Options 1 and 4 are inaccurate regarding Cushing's syndrome.

A nurse is reinforcing home care instructions to a client with a diagnosis of Cushing's syndrome. Which statement reflects a need for further client education? 1. "Taking my medications exactly as prescribed is essential." 2. "I need to read the labels on any over-the-counter medications I purchase." 3. "My family needs to be familiar with the signs and symptoms of hypoadrenalism." 4. "I could experience the signs and symptoms of hyperadrenalism because of Cushing's."

*2. "I need to read the labels on any over-the-counter medications I purchase."* *rationale* The client with Cushing's syndrome should be instructed to take the medications exactly as prescribed. The nurse should emphasize the importance of continuing medications, consulting with the health care provider before purchasing any over-the-counter medications, and maintaining regular follow-up care. The nurse should also instruct the client in the signs and symptoms of both hypoadrenalism and hyperadrenalism.

A nurse has just supervised a newly diagnosed diabetes mellitus client self-inject NPH insulin at 7:30 ᴀᴍ. The nurse reviews the time frames for peak insulin action with the client, telling the client to be especially watchful for a hypoglycemic reaction between: 1. 7:30 ᴀᴍ and 9:30 ᴀᴍ 2. 1:30 ᴘᴍ and 7:30 ᴘᴍ 3. 8:30 ᴘᴍ and 12:00 ᴀᴍ 4. 2:30 ᴀᴍ and 4:30 ᴀᴍ

*2. 1:30 ᴘᴍ and 7:30 ᴘᴍ* *rationale* NPH is an intermediate-acting insulin. It begins to work in 1 to 2 hours (onset), peaks in 6 to 12 hours, and lasts for 18 to 24 hours (duration). Hypoglycemic reactions most likely occur during peak time, which in this case is option 2.

Which of the following clients is at risk for developing thyrotoxicosis? 1. A client with hypothyroidism 2. A client with Graves' disease who is having surgery 3. A client with diabetes mellitus scheduled for debridement of a foot ulcer 4. A client with diabetes insipidus scheduled for an invasive diagnostic test

*2. A client with Graves' disease who is having surgery* *rationale* Thyrotoxicosis is usually seen in clients with Graves' disease with the symptoms precipitated by a major stressor. This complication typically occurs during periods of severe physiological or psychological stress such as trauma, sepsis, the birth process, or major surgery. It also must be recognized as a potential complication following a thyroidectomy.

A nurse is caring for a client with Addison's disease. The nurse checks the vital signs and determines that the client has orthostatic hypotension. The nurse determines that this finding relates to which of the following? 1. A decrease in cortisol release 2. A decreased secretion of aldosterone 3. An increase in epinephrine secretion 4. Increased levels of androgens

*2. A decreased secretion of aldosterone* *rationale* A decreased secretion of aldosterone results in a limited reabsorption of sodium and water; therefore the client experiences fluid volume deficit. A decrease in cortisol, an increase in epinephrine, and an increase in androgen secretion do not result in orthostatic hypotension.

While collecting data on a client being prepared for an adrenalectomy, the nurse obtains a temperature reading of 100.8° F. The nurse analyzes this temperature reading as: 1. Within normal limits 2. A finding that needs to be reported immediately 3. An expected finding caused by the operative stress response 4. Slightly abnormal but an insignificant finding

*2. A finding that needs to be reported immediately* *rationale* An adrenalectomy is performed because of excess adrenal gland function. Excess cortisol production impairs the immune response, which puts the client at risk for infection. Because of this, the client needs to be protected from infection, and minor variations in normal vital sign values must be reported so that infections are detected early, before they become overwhelming. In addition, the surgeon may elect to postpone surgery in the event of a fever because it can be indicative of infection. Options 1, 3, and 4 are not correct interpretations.

A nurse is reviewing the postoperative prescriptions for a client who had a transsphenoidal hypophysectomy. Which health care provider's prescription, if noted on the record, indicates the need for clarification? 1. Instruct the client about the need for a Medic-Alert bracelet. 2. Apply a loose dressing if any clear drainage is noted. 3. Monitor vital signs and neurological status. 4. Instruct the client to avoid blowing the nose.

*2. Apply a loose dressing if any clear drainage is noted.* *rationale* The nurse should observe for clear nasal drainage, constant swallowing, and a severe, persistent, generalized, or frontal headache. These signs and symptoms indicate cerebrospinal fluid leak into the sinuses. If clear drainage is noted following this procedure, the health care provider needs to be notified immediately. Options 1, 3, and 4 indicate appropriate postoperative interventions.

A client is admitted with a diagnosis of pheochromocytoma. The nurse would monitor which of the following to detect the most common sign of pheochromocytoma? 1. Skin temperature 2. Blood pressure 3. Urine ketones 4. Weight

*2. Blood pressure* *rationale* Hypertension is the major symptom associated with pheochromocytoma and is monitored by taking the client's blood pressure. Glycosuria, weight loss, and diaphoresis are other clinical manifestations of pheochromocytoma; however, hypertension is the most common sign.

A client with a pituitary tumor will undergo transsphenoidal hypophysectomy. The nurse includes which priority item in the preoperative teaching plan for the client? 1. Brushing the teeth vigorously and frequently is important to minimize bacteria in the mouth. 2. Blowing the nose following surgery is prohibited. 3. A small area will be shaved at the base of the neck. 4. It will be necessary to cough and deep breathe following the surgery.

*2. Blowing the nose following surgery is prohibited.* *rationale* The approach used for this surgery is the oronasal route, specifically where the upper lip meets the gum. The surgeon then uses a route through the sphenoid sinus to get to the pituitary gland. The client is not allowed to blow the nose, sneeze, or cough vigorously because these activities could raise intracranial pressure. The client also is not allowed to brush the teeth, to avoid disrupting the surgical site. Alternate methods for performing mouth care are used.

A nurse is caring for a client diagnosed with hyperparathyroidism who is prescribed furosemide (Lasix). The nurse reinforces dietary instructions to the client. Which of the following is an appropriate instruction? 1. Increase dietary intake of calcium. 2. Drink at least 2 to 3 L of fluid daily. 3. Eat sparely when experiencing nausea. 4. Decrease dietary intake of potassium.

*2. Drink at least 2 to 3 L of fluid daily.* *rationale* The aim of treatment in the client with hyperparathyroidism is to increase the renal excretion of calcium and decrease gastrointestinal absorption and bone resorption. This is aided by the sufficient intake of fluids. Dietary restriction of calcium may be used as a component of therapy. The parathyroid is responsible for calcium production, and the term, "hyperparathyroidism" can be indicative of an increase in calcium. The client should eat foods high in potassium, especially if the client is taking furosemide. Limiting nutrients is not advisable.

When caring for a client diagnosed with pheochromocytoma, what information should the nurse know when assisting with planning care? 1. Profound hypotension may occur. 2. Excessive catecholamines are released. 3. The condition is not curable and is treated symptomatically. 4. Hypoglycemia is the primary presenting symptom.

*2. Excessive catecholamines are released.* *rationale* Pheochromocytoma is a catecholamine-producing tumor of the adrenal gland and causes secretion of excessive amounts of epinephrine and norepinephrine. Hypertension is the principal manifestation, and the client has episodes of a high blood pressure accompanied by pounding headaches. The excessive release of catecholamine also results in excessive conversion of glycogen into glucose in the liver. Consequently, hyperglycemia and glucosuria occur during attacks. Pheochromocytoma is curable. The primary treatment is surgical removal of one or both of the adrenal glands, depending on whether the tumor is unilateral or bilateral.

A client scheduled for a thyroidectomy says to the nurse, "I am so scared to get cut in my neck." Based on the client's statement, the nurse determines that the client is experiencing which problem? 1. Inadequate knowledge about the surgical procedure 2. Fear about impending surgery 3. Embarrassment about the changes in personal appearance 4. Lack of support related to the surgical procedure

*2. Fear about impending surgery* *rationale* The client is having a difficult time coping with the scheduled surgery. The client is able to express fears but is scared. No data in the question support options 1, 3, and 4.

A nurse is monitoring a client with Graves' disease for signs of thyrotoxic crisis (thyroid storm). Which of the following signs and symptoms, if noted in the client, will alert the nurse to the presence of this crisis? *Select all that apply.* 1. Bradycardia 2. Fever 3. Sweating 4. Agitation 5. Pallor

*2. Fever* *3. Sweating* *4. Agitation* *rationale* Thyrotoxic crisis (thyroid storm) is an acute, potentially life-threatening state of extreme thyroid activity that represents a breakdown in the body's tolerance to a chronic excess of thyroid hormones. The clinical manifestations include fever greater than 100° F, severe tachycardia, flushing and sweating, and marked agitation and restlessness. Delirium and coma can occur.

A nurse is caring for a postoperative adrenalectomy client. Which of the following does the nurse specifically monitor for in this client? 1. Peripheral edema 2. Bilateral exophthalmos 3. Signs and symptoms of hypocalcemia 4. Signs and symptoms of hypovolemia

*4. Signs and symptoms of hypovolemia* *rationale* Following adrenalectomy, the client is at risk for hypovolemia. Aldosterone, secreted by the adrenal cortex, plays a major role in fluid volume balance by retaining sodium and water. A deficiency of adrenocortical hormones does not cause the clinical manifestations noted in options 1, 2, and 3.

A nurse is caring for a client with pheochromocytoma. The client asks for a snack and something warm to drink. The appropriate choice for this client to meet nutritional needs would be which of the following? 1. Crackers with cheese and tea 2. Graham crackers and warm milk 3. Toast with peanut butter and cocoa 4. Vanilla wafers and coffee with cream and sugar

*2. Graham crackers and warm milk* *rationale* The client with pheochromocytoma needs to be provided with a diet that is high in vitamins, minerals, and calories. Of particular importance is that food or beverages that contain caffeine (e.g., chocolate, coffee, tea, and cola) are prohibited.

A nurse is caring for a client after thyroidectomy and monitoring for signs of thyroid storm. The nurse understands that which of the following is a manifestation associated with this disorder? 1. Bradycardia 2. Hypotension 3. Constipation 4. Hypothermia

*2. Hypotension* *rationale* Clinical manifestations associated with thyroid storm include a fever as high as 106° F (41.1° C), severe tachycardia, profuse diarrhea, extreme vasodilation, hypotension, atrial fibrillation, hyperreflexia, abdominal pain, diarrhea, and dehydration. With this disorder, the client's condition can rapidly progress to coma and cardiovascular collapse.

A nurse is caring for a client following an adrenalectomy and is monitoring for signs of adrenal insufficiency. Which of the following, if noted in the client, indicates signs and symptoms related to adrenal insufficiency? *Select all that apply.* 1. Double vision 2. Hypotension 3. Mental status changes 4. Weakness 5. Fever

*2. Hypotension* *3. Mental status changes* *4. Weakness* *5. Fever* *rationale* The nurse should be alert to signs and symptoms of adrenal insufficiency in a client following adrenalectomy. These signs and symptoms include weakness, hypotension, fever, and mental status changes. Double vision is generally not associated with this condition.

A client with Cushing's disease is being admitted to the hospital after a stab wound to the abdomen. The nurse plans care and places highest priority on which potential problem? 1. Nervousness 2. Infection 3. Concern about appearance 4. Inability to care for self

*2. Infection* *rationale* The client with a stab wound has a break in the body's first line of defense against infection. The client with Cushing's disease is at great risk for infection because of excess cortisol secretion and subsequent impaired antibody function and decreased proliferation of lymphocytes. The client may also have a potential for the problems listed in the other options but these are not the highest priority at this time.

A nurse is collecting data on a client admitted to the hospital with a diagnosis of myxedema. Which data collection technique will provide data necessary to support the admitting diagnosis? 1. Auscultation of lung sounds 2. Inspection of facial features 3. Percussion of the thyroid gland 4. Palpation of the adrenal glands

*2. Inspection of facial features* *rationale* Inspection of facial features will reveal the characteristic coarse features, presence of edema around the eyes and face, and a blank expression that are characteristic of myxedema. The techniques in the remaining options will not reveal any data that would support the diagnosis of myxedema.

A nurse is providing discharge instructions to a client who had a unilateral adrenalectomy. Which of the following will be a component of the instructions? 1. The reason for maintaining a diabetic diet 2. Instructions about early signs of a wound infection 3. Teaching regarding proper application of an ostomy pouch 4. The need for lifelong replacement of all adrenal hormones

*2. Instructions about early signs of a wound infection* *rationale* A client who is undergoing a unilateral adrenalectomy will be placed on corticosteroids temporarily to avoid a cortisol deficiency. These medications will be gradually weaned in the postoperative period until they are discontinued. Because of the anti-inflammatory properties of corticosteroids, clients who undergo an adrenalectomy are at increased risk for developing wound infections. Because of this increased risk for infection, it is important for the client to know measures to prevent infection, early signs of infection, and what to do if an infection is present. Options 1, 3, and 4 are incorrect instructions.

A nurse is caring for a postoperative parathyroidectomy client. Which of the following would require the nurse's immediate attention? 1. Incisional pain 2. Laryngeal stridor 3. Difficulty voiding 4. Abdominal cramps

*2. Laryngeal stridor* *rationale* During the postoperative period, the nurse carefully observes the client for signs of hemorrhage, which cause swelling and the compression of adjacent tissue. Laryngeal stridor is a harsh, high-pitched sound heard on inspiration and expiration that is caused by the compression of the trachea and that leads to respiratory distress. It is an acute emergency situation that requires immediate attention to avoid the complete obstruction of the airway.

After several diagnostic tests, a client is diagnosed with diabetes insipidus. The nurse understands that which symptom is indicative of this disorder? 1. Diarrhea 2. Polydipsia 3. Weight gain 4. Blurred vision

*2. Polydipsia* *rationale* Polydipsia and polyuria are classic symptoms of diabetes insipidus. The urine is pale in color, and its specific gravity is low. Anorexia and weight loss occur. Diarrhea, weight loss, and blurred vision are not manifestations of the disorder.

A nurse is collecting data from a client who is being admitted to the hospital for a diagnostic workup for primary hyperparathyroidism. The nurse understands that which client complaint would be characteristic of this disorder? 1. Diarrhea 2. Polyuria 3. Polyphagia 4. Weight gain

*2. Polyuria* *rationale* Hypercalcemia is the hallmark of hyperparathyroidism. Elevated serum calcium levels produce osmotic diuresis (polyuria). This diuresis leads to dehydration and the client would lose weight. Options 1, 3, and 4 are gastrointestinal (GI) symptoms but are not associated with the common GI symptoms typical of hyperparathyroidism (nausea, vomiting, anorexia, constipation).

The anticipated intended effect of fludrocortisone acetate (Florinef) for the treatment of Addison's disease is to: 1. Stimulate the immune response. 2. Promote electrolyte balance. 3. Stimulate thyroid production. 4. Stimulate thyrotropin production.

*2. Promote electrolyte balance.* *rationale* Florinef is a long-acting oral medication with mineralocorticoid and moderate glucocorticoid activity used for long-term management of Addison's disease. Mineralocorticoids act on the renal distal tubules to enhance the reabsorption of sodium and chloride ions and the excretion of potassium and hydrogen ions. In small doses, fludrocortisone acetate causes sodium retention and increased urinary potassium excretion. The client rapidly can develop hypotension and fluid and electrolyte imbalance if the medication is discontinued abruptly. Options 1, 3, and 4 are not associated with the effects of this medication.

What would the nurse anticipate being included in the plan of care for a client who has been diagnosed with Graves' disease? 1. Provide a high-fiber diet. 2. Provide a restful environment. 3. Provide three small meals per day. 4. Provide the client with extra blankets.

*2. Provide a restful environment.* *rationale* Because of the hypermetabolic state, the client with Graves' disease needs to be provided with an environment that is restful both physically and mentally. Six full meals a day that are well balanced and high in calories are required, because of the accelerated metabolic rate. Foods that increase peristalsis (e.g., high-fiber foods) need to be avoided. These clients suffer from heat intolerance and require a cool environment.

A nurse is caring for a client with hypothyroidism who is overweight. Which food items would the nurse suggest to include in the plan? 1. Peanut butter, avocado, and red meat 2. Skim milk, apples, whole-grain bread, and cereal 3. Organ meat, carrots, and skim milk 4. Seafood, spinach, and cream cheese

*2. Skim milk, apples, whole-grain bread, and cereal* *rationale* Clients with hypothyroidism may have a problem with being over-weight because of their decreased metabolic need. They should consume foods from all food groups, which will provide them with the necessary nutrients; however, the foods should be low in calories. Option 2 is the only option that identifies food items that are low in calories.

When caring for a client who is having clear drainage from his nares after transsphenoidal hypophysectomy, which action by the nurse is appropriate? 1. Lower the head of the bed. 2. Test the drainage for glucose. 3. Obtain a culture of the drainage. 4. Continue to observe the drainage.

*2. Test the drainage for glucose.* *rationale* After hypophysectomy, the client should be monitored for rhinorrhea, which could indicate a cerebrospinal fluid (CSF) leak. If this occurs, the drainage should be collected and tested for glucose, indicating the presence of CSF. The head of the bed should not be lowered to prevent increased intracranial pressure. Clear nasal drainage would not indicate the need for a culture. Continuing to observe the drainage without taking action could result in a serious complication.

A client who returned to the nursing unit 8 hours ago after hypophysectomy has clear drainage saturating the nasal dressing. The nurse should take which action first? 1. Continue to observe for further drainage. 2. Test the drainage for glucose.* 3. Put the head of the bed flat. 4. Test the drainage for occult blood.

*2. Test the drainage for glucose.* *rationale* Following hypophysectomy the client should be monitored for rhinorrhea (clear nasal drainage), which could indicate a cerebrospinal fluid (CSF) leak. If this occurs, the drainage should be collected and tested for the presence of CSF by testing it for glucose. CSF tests positive for glucose, whereas true nasal secretions would not. It is not necessary to test drainage that is clear for occult blood. The head of the bed should not be lowered, to prevent a rise in intracranial pressure. Continuing to observe the drainage without taking action could put the client at risk for developing a serious complication.

A nurse has reinforced instructions to the client with hyperparathyroidism regarding home care measures related to exercise. Which statement by the client indicates a need for further instruction? *Select all that apply.* 1. "I enjoy exercising but I need to be careful." 2. "I need to pace my activities throughout the day." 3. "I need to limit playing football to only the weekends." 4. "I should gauge my activity level by my energy level." 5. "I should exercise in the evening to encourage a good sleep pattern."

*3. "I need to limit playing football to only the weekends."* *5. "I should exercise in the evening to encourage a good sleep pattern."* *rationale* The client should be instructed to avoid high-impact activity or contact sports such as football. Exercising late in the evening may interfere with restful sleep. The client with hyperparathyroidism should pace activities throughout the day and plan for periods of uninterrupted rest. The client should plan for at least 30 minutes of walking each day to support calcium movement into the bones. The client should be instructed to use energy level as a guide to activity.

A client with Cushing's syndrome verbalizes concern to the nurse regarding the appearance of the buffalo hump that has developed. Which response by the nurse is appropriate? 1. "Don't be concerned, this problem can be covered with clothing." 2. "This is permanent, but looks are deceiving and not that important." 3. "Usually, these physical changes slowly improve following treatment." 4. "Try not to worry about it. There are other things to be concerned about."

*3. "Usually, these physical changes slowly improve following treatment."* *rationale* The client with Cushing's syndrome should be reassured that most physical changes resolve with treatment. Options 1, 2, and 4 are not therapeutic responses.

Following hypophysectomy, a client complains of being very thirsty and having to urinate frequently. The initial nursing action is to: 1. Document the complaints. 2. Increase fluid intake. 3. Check the urine specific gravity. 4. Check for urinary glucose.

*3. Check the urine specific gravity.* *rationale* Following hypophysectomy, diabetes insipidus can occur temporarily because of antidiuretic hormone deficiency. This deficiency is related to surgical manipulation. The nurse should check the urine for specific gravity and report the results if they are less than 1.005. Urinary glucose and diabetes mellitus is not a concern here. In this situation, increasing fluid intake would require a health care provider's prescription. The client's complaint would be documented but not as an initial action.

A nurse is caring for a client with pheochromocytoma. Which data would indicate a potential complication associated with this disorder? 1. A urinary output of 50 mL/hr 2. A coagulation time of 5 minutes 3. Congestion heard on auscultation of the lungs 4. A blood urea nitrogen (BUN) level of 20 mg/dL

*3. Congestion heard on auscultation of the lungs* *rationale* The complications associated with pheochromocytoma include hypertensive retinopathy and nephropathy, myocarditis, congestive heart failure (CHF), increased platelet aggregation, and stroke. Death can occur from shock, stroke, renal failure, dysrhythmias, or dissecting aortic aneurysm. Congestion heard on auscultation of the lungs is indicative of CHF. A urinary output of 50 mL/hr is an appropriate output; the nurse would become concerned if the output were less than 30 mL/hr. A coagulation time of 5 minutes is normal. A BUN level of 20 mg/dL is a normal finding.

What are s/s of thyroid storm? (6)

*Life threatening* - ^ HR, dec. BP, ^ Respirations - HF - Shock - ^ Temp >= 105.3 - seizures, n/v, coma

Post- op priority for a pt after a thyroid cancer removal is:

*airway* - ax for bleeding, edema - hypocalcemia (if para. was accidentally nicked)

A nurse is preparing to provide instructions to a client with Addison's disease regarding diet therapy. The nurse understands that which of the following diets would likely be prescribed for this client? 1. Low-protein diet 2. Low-sodium diet 3. High-sodium diet 4. Low-carbohydrate diet

*3. High-sodium diet* *rationale* A high-sodium, high-complex carbohydrate, and high-protein diet will be prescribed for the client with Addison's disease. To prevent excess fluid and sodium loss, the client is instructed to maintain an adequate salt intake of up to 8 g of sodium daily and to increase salt intake during hot weather, before strenuous exercise, and in response to fever, vomiting, or diarrhea.

A client is diagnosed with hyperparathyroidism. The nurse teaching the client about dietary alterations to manage the disorder tells the client to limit which of the following foods in the diet? 1. Bananas 2. Oatmeal 3. Ice cream 4. Chicken breast

*3. Ice cream* *rationale* The client with hyperparathyroidism is likely to have elevated calcium levels. This client should reduce intake of dairy products such as milk, cheese, ice cream, or yogurt. Apples, bananas, chicken, oatmeal, and pasta are low-calcium foods.

A nurse would expect to note which interventions in the plan of care for a client with hypothyroidism? *Select all that apply.* 1. Provide a cool environment for the client. 2. Instruct the client to consume a high-fat diet. 3. Instruct the client about thyroid replacement therapy. 4. Encourage the client to consume fluids and high-fiber foods in the diet. 5. Instruct the client to contact the health care provider if episodes of chest pain occur. 6. Inform the client that iodine preparations will be prescribed to treat the disorder.

*3. Instruct the client about thyroid replacement therapy.* *4. Encourage the client to consume fluids and high-fiber foods in the diet.* *5. Instruct the client to contact the health care provider if episodes of chest pain occur.* *rationale* The clinical manifestations of hypothyroidism are the result of decreased metabolism from low levels of thyroid hormone. Interventions are aimed at replacement of the hormones and providing measures to support the signs and symptoms related to a decreased metabolism. The nurse encourages the client to consume a well-balanced diet that is low in fat for weight reduction and high in fluids and high-fiber foods to prevent constipation. The client often has cold intolerance and requires a warm environment. The client would notify the health care provider if chest pain occurs since it could be an indication of overreplacement of thyroid hormone. Iodine preparations are used to treat hyperthyroidism. These medications decrease blood flow through the thyroid gland and reduce the production and release of thyroid hormone.

A client with newly diagnosed Cushing's syndrome expresses concern about personal appearance, specifically about the "buffalo hump" that has developed at the base of the neck. When counseling the client about this manifestation, the nurse should incorporate the knowledge that: 1. This is a permanent feature. 2. It can be minimized by wearing tight clothing. 3. It may slowly improve with treatment of the disorder. 4. It will quickly disappear once medication therapy is started.

*3. It may slowly improve with treatment of the disorder.* *rationale* The client with Cushing's syndrome should be reassured that most physical changes resolve over time with treatment. The other options are incorrect.

A client has been diagnosed with hypoparathyroidism. The nurse teaches the client to include foods in the diet that are: 1. High in phosphorus and low in calcium 2. Low in phosphorus and low in calcium 3. Low in phosphorus and high in calcium 4. High in phosphorus and high in calcium

*3. Low in phosphorus and high in calcium* *rationale* Hypoparathyroidism results in hypocalcemia. A therapeutic diet for this disorder is one that is high in calcium but low in phosphorus because these two electrolytes have inverse proportions in the body. All of the other options are unrelated to this disorder and are incorrect.

A nurse reviews a plan of care for a postoperative client following a thyroidectomy and notes that the client is at risk for breathing difficulty. Which of the following nursing interventions will the nurse suggest to include in the plan of care? 1. Maintain a supine position. 2. Encourage coughing and deep breathing exercises. 3. Monitor neck circumference frequently. 4. Maintain a pressure dressing on the operative site.

*3. Monitor neck circumference frequently.* *rationale* Following a thyroidectomy, the client should be placed in an upright position to facilitate air exchange. The nurse should assist the client with deep breathing exercises, but coughing is minimized to prevent tissue damage and stress to the incision. A pressure dressing is not placed on the operative site because it could affect breathing. The nurse should monitor the dressing closely and should loosen the dressing if necessary. Neck circumference is monitored at least every 4 hours to assess for postoperative edema.

Which nursing action would be appropriate to implement when a client has a diagnosis of pheochromocytoma? 1. Weigh the client. 2. Test the client's urine for glucose. 3. Monitor the client's blood pressure. 4. Palpate the client's skin to determine warmth.

*3. Monitor the client's blood pressure.* *rationale* Hypertension is the major symptom that is associated with pheochromocytoma. The blood pressure status is monitored by taking the client's blood pressure. Glycosuria, weight loss, and diaphoresis are also clinical manifestations of pheochromocytoma, but hypertension is the major symptom.

A nurse assists in developing a plan of care for a client with hyperparathyroidism receiving calcitonin-human (Cibacalcin). Which outcome has the highest priority regarding this medication? 1. Relief of pain 2. Absence of side effects 3. Reaching normal serum calcium levels 4. Verbalization of appropriate medication knowledge

*3. Reaching normal serum calcium levels* *rationale* Hypercalcemia can occur in clients with hyperparathyroidism, and calcitonin is used to lower plasma calcium level. The highest priority outcome in this client situation would be a reduction in serum calcium level. Option 1 is unrelated to this medication. Although options 2 and 4 are expected outcomes, they are not the highest priority for administering this medication.

What dx confirm Cushings?

- 24-hour urine for free cortisol - plasma cortisol levels for day/night variation

How is Acromegaly diagnosed? (3)

- CT, MRI - OGTT - ?

What treatment is used to correct HHNK?

- IV isotonic fluids - Insulin therapy - repletion of electrolytes (especially potassium) - Antipyretics, antiemetics, & antibiotics when appropriate to control fever, vomiting, & treat underlying infection if suspected.

What drugs are used to tx thyroid storm? (3)

- antithyroid drugs (eg. Tapazole) - iodine (SSKI) - Beta blocker (Inderal)

A nurse caring for a client scheduled for a transsphenoidal hypophysectomy to remove a tumor in the pituitary gland assists to develop a plan of care for the client. The nurse suggests including which specific information in the preoperative teaching plan? 1. Hair will need to be shaved. 2. Deep breathing and coughing will be needed after surgery. 3. Toothbrushing will not be permitted for at least 2 weeks following surgery. 4. Spinal anesthesia is used.

*3. Toothbrushing will not be permitted for at least 2 weeks following surgery.* *rationale* Based on the location of the surgical procedure, spinal anesthesia would not be used. In addition, the hair would not be shaved. Although coughing and deep breathing are important, specific to this procedure is avoiding toothbrushing to prevent disruption of the surgical site. Also, coughing may disrupt the surgical site.

A preoperative client is scheduled for adrenalectomy to remove a pheochromocytoma. The nurse would most closely monitor which of the following items in the preoperative period? 1. Intake and output 2. Blood urea nitrogen (BUN) 3. Vital signs 4. Urine glucose and ketones

*3. Vital signs* *rationale* Hypertension is the hallmark of pheochromocytoma. Severe hypertension can precipitate a stroke or sudden blindness. Although all the items are appropriate nursing assessments for the client with pheochromocytoma, the priority is to monitor the vital signs, especially the blood pressure.

A health care provider prescribes a 24-hour urine collection for vanillylmandelic acid (VMA). The nurse instructs the client in the procedure for the collection of the urine. Which statement by the client would indicate a need for further instruction? 1. "I will start the collection in 2 days. I cannot eat or drink any tea, chocolate, vanilla, or fruit until the test is completed." 2. "When I start the collection, I will urinate and discard that specimen." 3. "I will pour the urine into the collection bottle each time I urinate and refrigerate the urine." 4. "I can take any medications if I need to before the collection."

*4. "I can take any medications if I need to before the collection."* *rationale* Because a 24-hour urine collection is a timed quantitative determination, it is essential that the client start the test with an empty bladder. Therefore the client is instructed to void and discard the first urine and note the time and start the test. The 24-hour urine specimen collection bottle must be kept on ice or refrigerated. In a VMA collection, the client is instructed to avoid tea, chocolate, vanilla, and all fruits for 2 days before urine collection begins. Also clients are reminded not to take certain medications for 2 to 3 days before the test.

A client with myxedema has changes in intellectual function such as impaired memory, decreased attention span, and lethargy. The client's husband is upset and shares his concerns with the nurse. Which statement by the nurse is helpful to the client's husband? 1. "Would you like me to ask the health care provider for a prescription for a stimulant?" 2. "Give it time. I've seen dozens of clients with this problem that fully recover." 3. "I don't blame you for being frustrated, because the symptoms will only get worse." 4. "It's obvious that you are concerned about your wife's condition, but the symptoms may improve with continued therapy."

*4. "It's obvious that you are concerned about your wife's condition, but the symptoms may improve with continued therapy."* *rationale* Using therapeutic communication techniques, the nurse acknowledges the husband's concerns and conveys that the client's symptoms are common with myxedema. With thyroid hormone therapy, these symptoms should decrease, and cognitive function often returns to normal. Option 1 is not helpful, and it blocks further communication. Option 3 is pessimistic and untrue. Option 2 is not appropriate and offers false reassurance.

A client with pheochromocytoma is scheduled for surgery and says to the nurse, "I'm not sure that surgery is the best thing to do." What response by the nurse is appropriate? 1. "I think you are making the right decision to have the surgery." 2. "You are very ill. Your health care provider has made the correct decision." 3. "There is no reason to worry. Your health care provider is a wonderful surgeon." 4. "You have concerns about the surgical treatment for your condition."

*4. "You have concerns about the surgical treatment for your condition."* *rationale* Paraphrasing is restating the client's message in the nurse's own words. Option 4 addresses the therapeutic communication technique of paraphrasing. The client is reaching out for understanding. In option 3, the nurse is offering a false reassurance, and this type of response will block communication. Option 2 also represents a communication block because it reflects a lack of the client's right to an opinion. In option 1, the nurse is expressing approval, which can be harmful to a nurse-client relationship.

The nurse caring for a client who has had a subtotal thyroidectomy reviews the plan of care and determines which problem is the priority for this client in the immediate postoperative period? 1. Dehydration 2. Infection 3. Urinary retention 4. Bleeding

*4. Bleeding* *rationale* Hemorrhage is one of the most severe complications that can occur following thyroidectomy. The nurse must frequently check the neck dressing for bleeding and monitor vital signs to detect early signs of hemorrhage, which could lead to shock. T3 and T4 do not regulate fluid volumes in the body. Infection is a concern for any postoperative client but is not the priority in the immediate postoperative period. Urinary retention can occur in postoperative clients as a result of medication and anesthesia but is not the priority from the options provided.

A nurse working on an endocrine nursing unit understands that which correct concept is used in planning care? 1. Clients with Cushing's syndrome are likely to experience episodic hypotension. 2. Clients with hyperthyroidism must be monitored for weight gain. 3. Clients who have diabetes insipidus should be assessed for fluid excess. 4. Clients who have hyperparathyroidism should be protected against falls.

*4. Clients who have hyperparathyroidism should be protected against falls.* *rationale* Hyperparathyroidism is a disease that involves excess secretion of parathyroid hormone (PTH). Elevation of PTH causes excess calcium to be removed from the bones. There is a decline in bone mass, which may cause a fracture if a fall occurs. Cushing's syndrome is likely to cause hypertension. Clients with hypothyroidism must be monitored for weight gain and clients with hyperthyroidism must be monitored for weight loss. Clients who have diabetes insipidus should be assessed for fluid deficit.

Nsg interventions for DI includes:

- ax for polyuria - polydispia - I&O - titrated fluid replacement - daily weights

A nursing student notes in the medical record that a client with Cushing's syndrome is experiencing body image disturbances. The need for additional education regarding this problem is identified when the nursing student suggests which nursing intervention? 1. Encouraging the client's expression of feelings 2. Evaluating the client's understanding of the disease process 3. Encouraging family members to share their feelings about the disease process 4. Evaluating the client's understanding that the body changes need to be dealt with

*4. Evaluating the client's understanding that the body changes need to be dealt with* *rationale* Evaluating the client's understanding that the body changes that occur in this disorder need to be dealt with is an inappropriate nursing intervention. This option does not address the client's feelings. Options 1, 2, and 3 are appropriate because they address the client and family feelings regarding the disorder.

Which clinical manifestation should the nurse expect to note when assessing a client with Addison's disease? 1. Edema 2. Obesity 3. Hirsutism 4. Hypotension

*4. Hypotension* *rationale* Common manifestations of Addison's disease include postural hypotension from fluid loss, syncope, muscle weakness, anorexia, nausea, vomiting, abdominal cramps, weight loss, depression, and irritability. The manifestations in options 1, 2, and 3 are not associated with Addison's disease.

A client is brought to the emergency department in an unresponsive state, and a diagnosis of hyperglycemic hyperosmolar nonketotic syndrome (HHNS) is made. The nurse who is assisting to care for the client obtains which of the following immediately in preparation for the treatment of this syndrome? 1. NPH insulin 2. A nasal cannula 3. Intravenous (IV) infusion of sodium bicarbonate 4. IV infusion of normal saline

*4. IV infusion of normal saline* *rationale* The primary goal of treatment is to rehydrate the client to restore fluid volume and to correct electrolyte deficiency. IV fluid replacement is similar to that administered in diabetic ketoacidosis (DKA) and begins with IV infusion of normal saline. Regular insulin, not NPH insulin would be administered. The use of sodium bicarbonate to correct acidosis is avoided because it can precipitate a further drop in serum potassium levels. A nasal cannula for oxygen administration is not necessarily required to treat HHNS.

A client has an endocrine system dysfunction of the pancreas. The nurse anticipates that the client will exhibit impaired secretion of which of the following substances? 1. Amylase 2. Lipase 3. Trypsin 4. Insulin

*4. Insulin* *rationale* The pancreas produces both endocrine and exocrine secretions as part of its normal function. The organ secretes insulin as a key endocrine hormone to regulate the blood glucose level. Other pancreatic endocrine hormones are glucagon and somatostatin. The exocrine pancreas produces digestive enzymes such as amylase, lipase, and trypsin.

A nurse is caring for a client following a thyroidectomy. The client tells the nurse that she is concerned because of voice hoarseness. The client asks the nurse whether the hoarseness will subside. The nurse appropriately tells the client that the hoarseness: 1. Indicates nerve damage 2. Is harmless but permanent 3. Will worsen before it subsides 4. Is normal and will gradually subside

*4. Is normal and will gradually subside* *rationale* Hoarseness that develops in the postoperative period is usually the result of laryngeal pressure or edema and will resolve within a few days. The client should be reassured that the effects are transitory. Options 1, 2, and 3 are incorrect.

Which nursing measure would be effective in preventing complications in a client with Addison's disease? 1. Restricting fluid intake 2. Offering foods high in potassium 3. Checking family support systems 4. Monitoring the blood glucose

*4. Monitoring the blood glucose* *rationale* The decrease in cortisol secretion that characterizes Addison's disease can result in hypoglycemia. Therefore monitoring the blood glucose would detect the presence of hypoglycemia so that it can be treated early to prevent complications. Fluid intake should be encouraged to compensate for dehydration. Potassium intake should be restricted because of hyperkalemia. Option 3 is not a priority for this client.

A client with Graves' disease has exophthalmos and is experiencing photophobia. Which intervention would best assist the client with this problem? 1. Administering methimazole (Tapazole) every 8 hours 2. Lubricating the eyes with tap water every 2 to 4 hours 3. Instructing the client to avoid straining or heavy lifting 4. Obtaining dark glasses for the client

*4. Obtaining dark glasses for the client* *rationale* Because photophobia (light intolerance) accompanies this disorder, dark glasses are helpful in alleviating the symptom. Medical therapy for Graves' disease does not help alleviate the clinical manifestation of exophthalmos. Other interventions may be used to relieve the drying that occurs from not being able to completely close the eyes; however, the question is asking what the nurse can do for photophobia. Tap water, which is hypotonic, could actually cause more swelling to the eye because it could pull fluid into the interstitial space. In addition, the client is at risk for developing an eye infection because the solution is not sterile. There is no need to prevent straining with exophthalmos.

A nurse is caring for a client experiencing thyroid storm. Which of the following would be a priority concern for this client? 1. Inability to cope with the treatment plan 2. Lack of sexual drive 3. Self-consciousness about body appearance 4. Potential for cardiac disturbances

*4. Potential for cardiac disturbances* *rationale* Clients in thyroid storm are experiencing a life-threatening event, which is associated with uncontrolled hyperthyroidism. It is characterized by high fever, severe tachycardia, delirium, dehydration, and extreme irritability. The signs and symptoms of the disorder develop quickly, and therefore emergency measures must be taken to prevent death. These measures include maintaining hemodynamic status and patency of airway as well as providing adequate ventilation. Options 1, 2, and 3 are not a priority in the care of the client in thyroid storm.

A nurse is collecting data regarding a client after a thyroidectomy and notes that the client has developed hoarseness and a weak voice. Which nursing action is appropriate? 1. Check for signs of bleeding. 2. Administer calcium gluconate. 3. Notify the registered nurse immediately. 4. Reassure the client that this is usually a temporary condition.

*4. Reassure the client that this is usually a temporary condition.* *rationale* Weakness and hoarseness of the voice can occur as a result of trauma of the laryngeal nerve. If this develops, the client should be reassured that the problem will subside in a few days. Unnecessary talking should be discouraged. It is not necessary to notify the registered nurse immediately. These signs do not indicate bleeding or the need to administer calcium gluconate.

What dx confirm hyperparathyroidism?

- blood tests (^ PTH, ^ Ca, dec. P) - MRI, CT

What are key diagnostics w/ SIADH? How do you know it's SIADH?

- Na < 134 - serum osmolarity < 280 - urine specific gravity > 1.005

Post thyroidectomy, what interventions should the nurse be prepared for?

- O2 - suction - tracheostomy tray at bedside

List two conditions of the posterior pituitary:

- SIADH - DI

SIADH: serum osmolality, sodium, and urine specific gravity will be at what levels?

- Serum osmolality: <280 - Na+ < 134 - urine s.g. >1.005

What s/s are characteristic of Graves? (4)

- Tremor, nervousness, insomnia - Exophthalmos (change in pressure, not reversible)

What are s/s of Addisonian crisis?

- ^HR, dec. BP - confusion - dehydration (hyponatremia, seiaures) - shock (cyanosis, pallor ^HR, RR, dec. BP_

What nursing interventions can be taken to correct sx of thyroid storm?

- address resp. distress - fever - fluid replacement - decrease stressors

A client with Cushing's syndrome verbalizes concern to the nurse regarding the appearance of the buffalo hump that has developed. Which response by the nurse is appropriate?

1. "Don't be concerned, this problem can be covered with clothing." 2. "This is permanent, but looks are deceiving and not that important." *3. "Usually, these physical changes slowly improve following treatment."* 4. "Try not to worry about it. There are other things to be concerned about." *rationale* The client with Cushing's syndrome should be reassured that most physical changes resolve with treatment. Options 1, 2, and 4 are not therapeutic responses.

A nurse has reinforced instructions to the client with hyperparathyroidism regarding home care measures related to exercise. Which statement by the client indicates a need for further instruction? *Select all that apply.*

1. "I enjoy exercising but I need to be careful." 2. "I need to pace my activities throughout the day." *3. "I need to limit playing football to only the weekends."* 4. "I should gauge my activity level by my energy level." *5. "I should exercise in the evening to encourage a good sleep pattern."* *rationale* The client should be instructed to avoid high-impact activity or contact sports such as football. Exercising late in the evening may interfere with restful sleep. The client with hyperparathyroidism should pace activities throughout the day and plan for periods of uninterrupted rest. The client should plan for at least 30 minutes of walking each day to support calcium movement into the bones. The client should be instructed to use energy level as a guide to activity.

A nurse has reinforced instructions about measuring blood glucose levels to a client newly diagnosed with diabetes mellitus. The nurse determines that the client understands the procedure when making which most accurate statement?

1. "I should check my blood glucose level before eating a big meal." *2. "I should check my blood glucose level before eating each meal, regardless of how much I eat."* 3. "I should check my blood glucose level 2 hours after each meal." 4. "I should check my blood glucose level once a day." *rationale* The most effective and accurate measure for testing blood glucose is to test the level before each meal regardless of the amount of food to be eaten. The client should also check the blood glucose level at bedtime. Checking the level after the meal will provide an inaccurate assessment of diabetic control. Checking the level once daily will not provide enough data related to controlling the diabetes mellitus.

A client with type 1 diabetes mellitus calls the nurse to report recurrent episodes of hypoglycemia. Which statement by the client indicates a correct understanding of NPH insulin and exercise?

1. "I should not exercise after lunch." 2. "I should not exercise after breakfast." 3. "I should not exercise in the late evening." *4. "I should not exercise in the late afternoon."* *rationale* A hypoglycemic reaction may occur in response to increased exercise. Clients should avoid exercise during the peak time of insulin. NPH insulin peaks at 12 to 14 hours; therefore, late afternoon exercise would occur during the peak of the medication.

A nurse is collecting data from a client with type 2 diabetes mellitus. Which statement by the client indicates an understanding of the medication regimen?

1. "I should take my metformin (Glucophage) only if my blood glucose is elevated." 2. "By taking these medications, I am able to eat more." 3. "When I become ill, I need to increase the number of pills I take." *4. "The medication that I am taking helps release the insulin I already make."* *rationale* Clients with type 2 diabetes mellitus have decreased or impaired insulin secretion. Oral hypoglycemic agents are given to these clients to facilitate glucose use and need to be taken on a regular schedule as prescribed. To maintain normal blood glucose levels throughout the day, oral hypoglycemic agents such as metformin are not taken on an as-needed basis depending on the blood glucose levels. Insulin injections may be given during times of stress-induced hyperglycemia. Oral insulin is not available or effective because of the breakdown of the insulin by digestion.

A client with pheochromocytoma is scheduled for surgery and says to the nurse, "I'm not sure that surgery is the best thing to do." What response by the nurse is appropriate?

1. "I think you are making the right decision to have the surgery." 2. "You are very ill. Your health care provider has made the correct decision." 3. "There is no reason to worry. Your health care provider is a wonderful surgeon." *4. "You have concerns about the surgical treatment for your condition."* *rationale* Paraphrasing is restating the client's message in the nurse's own words. Option 4 addresses the therapeutic communication technique of paraphrasing. The client is reaching out for understanding. In option 3, the nurse is offering a false reassurance, and this type of response will block communication. Option 2 also represents a communication block because it reflects a lack of the client's right to an opinion. In option 1, the nurse is expressing approval, which can be harmful to a nurse-client relationship.

A health care provider prescribes a 24-hour urine collection for vanillylmandelic acid (VMA). The nurse instructs the client in the procedure for the collection of the urine. Which statement by the client would indicate a need for further instruction?

1. "I will start the collection in 2 days. I cannot eat or drink any tea, chocolate, vanilla, or fruit until the test is completed." 2. "When I start the collection, I will urinate and discard that specimen." 3. "I will pour the urine into the collection bottle each time I urinate and refrigerate the urine." *4. "I can take any medications if I need to before the collection."* *rationale* Because a 24-hour urine collection is a timed quantitative determination, it is essential that the client start the test with an empty bladder. Therefore the client is instructed to void and discard the first urine and note the time and start the test. The 24-hour urine specimen collection bottle must be kept on ice or refrigerated. In a VMA collection, the client is instructed to avoid tea, chocolate, vanilla, and all fruits for 2 days before urine collection begins. Also clients are reminded not to take certain medications for 2 to 3 days before the test.

When the nurse is teaching a client who has been newly diagnosed with type 1 diabetes mellitus, which statement by the client would indicate that teaching has been effective?

1. "I will stop taking my insulin if I'm too sick to eat." 2. "I will decrease my insulin dose during times of illness." 3. "I will adjust my insulin dose according to the level of glucose in my urine." *4. "I will notify my health care provider if my blood glucose level is greater than 250 mg/dL."* *rationale* During illness, the client should monitor the blood glucose level, and he or she should notify the health care provider (HCP) if the level is greater than 250 mg/dL. Insulin should never be stopped. In fact, insulin may need to be increased during times of illness. Doses should not be adjusted without the HCP's advice.

A nurse provides dietary instructions to a client with diabetes mellitus regarding the prescribed diabetic diet. Which statement, if made by the client, indicates the need for further teaching?

1. "I'll eat a balanced meal plan." 2. "I need to drink diet soft drinks." *3. "I need to buy special dietetic foods."* 4. "I will snack on fruit instead of cake." *rationale* It is important to emphasize to the client and family that they are not eating a diabetic diet but rather following a balanced meal plan. Adherence to nutrition principles is an important component of diabetic management, and an individualized meal plan should be developed for the client. It is not necessary for the client to purchase special dietetic foods.

A nurse in an outpatient diabetes clinic is assisting in caring for a client on insulin pump therapy. Which statement by the client indicates that a knowledge deficit exists regarding insulin pump therapy?

1. "If my blood sugars are elevated, I can bolus myself with additional insulin as prescribed." 2. "I'll need to check my blood sugars before meals in case I need a pre-meal insulin bolus." *3. "Now that I have this pump, I don't have to worry about insulin reactions or ketoacidosis occurring again."* 4. "I still need to follow an appropriate diet and exercise plan even though I don't have to inject myself daily anymore." *rationale* All of the statements are correct in regard to insulin pump therapy, except the one that mentions insulin reactions and ketoacidosis. Hypoglycemic reactions can occur if there is an error in calculating the insulin dose or if the pump malfunctions. Ketoacidosis can occur if too little insulin is used or if there is an increase in metabolic need. The pump does not have a built-in blood glucose monitoring feedback system, so the client is subject to the usual complications associated with insulin administration without the use of a pump.

A client with type 1 diabetes mellitus takes NPH insulin every morning and checks the blood glucose level four times per day. The client tells the nurse that yesterday the late afternoon blood glucose was 60 mg/dL and that she "felt funny." Which statement by the client would indicate an understanding of this occurrence?

1. "My blood glucose levels are running low because I'm tired." *2. "I forgot to take my usual afternoon snack yesterday."* 3. "I took less insulin this morning so I won't feel funny today." 4. "I don't know why I have to check my blood glucose four times a day. That seems too much." *rationale* Hypoglycemia is a blood glucose level of 60 mg/dL or less. The causes are multiple, but, in this case, omitting the afternoon snack is the cause. Fatigue and self-adjustment of dose are incorrect options. Recommended blood glucose testing for the client with type 1 diabetes mellitus is at least four times a day.

Which of the following statements made by the nursing student demonstrates an understanding of the hormone oxytocin?

1. "Production of oxytocin occurs in the ovaries." 2. "It is produced by the anterior pituitary gland." *3. "It causes contractions of the uterus during birth."* 4. "Release of oxytocin stimulates the pancreas to produce insulin." *rationale* Oxytocin is produced by the posterior pituitary, not the anterior pituitary gland, and stimulates the uterus to produce contractions during birth. The ovaries are the endocrine glands that produce estrogen and progesterone. The pancreas produces insulin and other enzymes that aid digestion. Oxytocin does not stimulate the pancreas to produce insulin.

A nurse is reinforcing home care instructions to a client with a diagnosis of Cushing's syndrome. Which statement reflects a need for further client education?

1. "Taking my medications exactly as prescribed is essential." *2. "I need to read the labels on any over-the-counter medications I purchase."* 3. "My family needs to be familiar with the signs and symptoms of hypoadrenalism." 4. "I could experience the signs and symptoms of hyperadrenalism because of Cushing's." *rationale* The client with Cushing's syndrome should be instructed to take the medications exactly as prescribed. The nurse should emphasize the importance of continuing medications, consulting with the health care provider before purchasing any over-the-counter medications, and maintaining regular follow-up care. The nurse should also instruct the client in the signs and symptoms of both hypoadrenalism and hyperadrenalism.

A client with myxedema has changes in intellectual function such as impaired memory, decreased attention span, and lethargy. The client's husband is upset and shares his concerns with the nurse. Which statement by the nurse is helpful to the client's husband?

1. "Would you like me to ask the health care provider for a prescription for a stimulant?" 2. "Give it time. I've seen dozens of clients with this problem that fully recover." 3. "I don't blame you for being frustrated, because the symptoms will only get worse." *4. "It's obvious that you are concerned about your wife's condition, but the symptoms may improve with continued therapy."* *rationale* Using therapeutic communication techniques, the nurse acknowledges the husband's concerns and conveys that the client's symptoms are common with myxedema. With thyroid hormone therapy, these symptoms should decrease, and cognitive function often returns to normal. Option 1 is not helpful, and it blocks further communication. Option 3 is pessimistic and untrue. Option 2 is not appropriate and offers false reassurance.

A nurse is caring for a client newly diagnosed with diabetes mellitus. The client asks the nurse whether eating at a restaurant will affect the diabetic control and whether this is allowed. Which nursing response is appropriate?

1. "You really should not eat in restaurants." 2. "If you plan to eat in a restaurant, you need to avoid carbohydrates." *3. "You should order a half-portion meal and have fresh fruit for dessert."* 4. "You should increase your daily dose of insulin by half on the day you plan to eat out." *rationale* Clients with diabetes mellitus are instructed to make adjustments in their total daily intake to plan for meals at restaurants or parties. Some useful strategies include ordering half portions, salads with dressing on the side, fresh fruit for dessert, and baked or steamed entrées. Clients are not instructed to avoid any food group or to increase their prescribed insulin dosage.

What are the effects of glucocorticoids?

1. *Mood changes* - insomnia, depressed, psychotic 2. *Immunosuppression* 3. *Breakdown fat & proteins* 4. *Inhibit insulin* - hyperglycemic

A licensed practical nurse (LPN) is assisting a high school nurse in conducting a session with female adolescents regarding the menstrual cycle. The LPN tells the adolescents that the normal duration of the menstrual cycle is about:

1. 14 days *2. 28 days* 3. 30 days 4. 45 days *rationale* The normal duration of the menstrual cycle is about 28 days, although it may range from 20 to 45 days. The first day of the menstrual period is counted as day 1 of the woman's cycle. Options 1, 3, and 4 are incorrect.

A nurse has just supervised a newly diagnosed diabetes mellitus client self-inject NPH insulin at 7:30 ᴀᴍ. The nurse reviews the time frames for peak insulin action with the client, telling the client to be especially watchful for a hypoglycemic reaction between:

1. 7:30 ᴀᴍ and 9:30 ᴀᴍ *2. 1:30 ᴘᴍ and 7:30 ᴘᴍ* 3. 8:30 ᴘᴍ and 12:00 ᴀᴍ 4. 2:30 ᴀᴍ and 4:30 ᴀᴍ *rationale* NPH is an intermediate-acting insulin. It begins to work in 1 to 2 hours (onset), peaks in 6 to 12 hours, and lasts for 18 to 24 hours (duration). Hypoglycemic reactions most likely occur during peak time, which in this case is option 2.

Which of the following clients is at risk for developing thyrotoxicosis?

1. A client with hypothyroidism *2. A client with Graves' disease who is having surgery* 3. A client with diabetes mellitus scheduled for debridement of a foot ulcer 4. A client with diabetes insipidus scheduled for an invasive diagnostic test *rationale* Thyrotoxicosis is usually seen in clients with Graves' disease with the symptoms precipitated by a major stressor. This complication typically occurs during periods of severe physiological or psychological stress such as trauma, sepsis, the birth process, or major surgery. It also must be recognized as a potential complication following a thyroidectomy.

A nurse is caring for a client with Addison's disease. The nurse checks the vital signs and determines that the client has orthostatic hypotension. The nurse determines that this finding relates to which of the following?

1. A decrease in cortisol release *2. A decreased secretion of aldosterone* 3. An increase in epinephrine secretion 4. Increased levels of androgens *rationale* A decreased secretion of aldosterone results in a limited reabsorption of sodium and water; therefore the client experiences fluid volume deficit. A decrease in cortisol, an increase in epinephrine, and an increase in androgen secretion do not result in orthostatic hypotension.

A nurse is monitoring a client receiving glipizide (Glucotrol). Which outcome indicates an ineffective response from the medication?

1. A decrease in polyuria 2. A decrease in polyphagia 3. A fasting plasma glucose of 100 mg/dL *4. A glycosylated hemoglobin level of 12%* *rationale* Glipizide (Glucotrol) is an oral hypoglycemic agent administered to decrease the serum glucose level and the signs and symptoms of hyperglycemia. Therefore, a decrease in both polyuria and polyphagia would indicate a therapeutic response. Laboratory values are also used to monitor a client's response to treatment. A fasting blood glucose level of 100 mg/dL is within normal limits. However, glycosylated hemoglobin of 12% indicates poor glycemic control.

A nurse is reviewing the prescriptions of a client diagnosed with diabetes mellitus who was admitted because of an infected foot ulcer. Which health care provider's prescription supports the treatment of this condition?

1. A decreased amount of NPH daily insulin *2. An increased amount of NPH daily insulin* 3. An increased-calorie diet 4. A decreased-calorie diet *rationale* Infection is a physiological stressor that can cause an increase in the level of epinephrine in the body. An increase in epinephrine causes an increase in blood glucose levels. When the client is under stress, such as when an infection exists, the client will require an increase in the dose of insulin to facilitate the transport of excess glucose into the cells. The client does not necessarily need an adjustment in the daily diet.

A nurse is caring for a client with pheochromocytoma. Which data would indicate a potential complication associated with this disorder?

1. A urinary output of 50 mL/hr 2. A coagulation time of 5 minutes *3. Congestion heard on auscultation of the lungs* 4. A blood urea nitrogen (BUN) level of 20 mg/dL *rationale* The complications associated with pheochromocytoma include hypertensive retinopathy and nephropathy, myocarditis, congestive heart failure (CHF), increased platelet aggregation, and stroke. Death can occur from shock, stroke, renal failure, dysrhythmias, or dissecting aortic aneurysm. Congestion heard on auscultation of the lungs is indicative of CHF. A urinary output of 50 mL/hr is an appropriate output; the nurse would become concerned if the output were less than 30 mL/hr. A coagulation time of 5 minutes is normal. A BUN level of 20 mg/dL is a normal finding.

Acromegaly is most frequently diagnosed in: a. Middle-aged adults b. Newborns c. Children ages 2 to 5 d. Adults age 65 and older

1. A: Acromegaly results from benign tumors on the pituitary gland that produce excessive amounts of growth hormone. Although symptoms may present at any age, the diagnosis generally occurs in middle-aged persons. Untreated, the consequences of acromegaly include type 2 diabetes, hypertension and increased risk of cardiovascular disease, arthritis and colon polyps.

After several diagnostic tests, a client is diagnosed with diabetes insipidus. The nurse understands that which symptom is indicative of this disorder?

1. Diarrhea *2. Polydipsia* 3. Weight gain 4. Blurred vision *rationale* Polydipsia and polyuria are classic symptoms of diabetes insipidus. The urine is pale in color, and its specific gravity is low. Anorexia and weight loss occur. Diarrhea, weight loss, and blurred vision are not manifestations of the disorder.

A client with diabetes mellitus demonstrates acute anxiety when admitted to the hospital for the treatment of hyperglycemia. The appropriate intervention to decrease the client's anxiety would be to:

1. Administer a sedative. *2. Convey empathy, trust, and respect toward the client.* 3. Ignore the signs and symptoms of anxiety so that they will soon disappear. 4. Make sure the client knows all the correct medical terms so that he or she can understand what is happening. *rationale* The appropriate intervention is to address the client's feelings related to the anxiety and to convey empathy, trust, and respect toward the client. Administering a sedative is not the most appropriate intervention. The nurse should not ignore the client's anxious feelings. A client will not relate to medical terms, particularly when anxiety exists.

A client with Graves' disease has exophthalmos and is experiencing photophobia. Which intervention would best assist the client with this problem?

1. Administering methimazole (Tapazole) every 8 hours 2. Lubricating the eyes with tap water every 2 to 4 hours 3. Instructing the client to avoid straining or heavy lifting *4. Obtaining dark glasses for the client* *rationale* Because photophobia (light intolerance) accompanies this disorder, dark glasses are helpful in alleviating the symptom. Medical therapy for Graves' disease does not help alleviate the clinical manifestation of exophthalmos. Other interventions may be used to relieve the drying that occurs from not being able to completely close the eyes; however, the question is asking what the nurse can do for photophobia. Tap water, which is hypotonic, could actually cause more swelling to the eye because it could pull fluid into the interstitial space. In addition, the client is at risk for developing an eye infection because the solution is not sterile. There is no need to prevent straining with exophthalmos.

A client has an endocrine system dysfunction of the pancreas. The nurse anticipates that the client will exhibit impaired secretion of which of the following substances?

1. Amylase 2. Lipase 3. Trypsin *4. Insulin* *rationale* The pancreas produces both endocrine and exocrine secretions as part of its normal function. The organ secretes insulin as a key endocrine hormone to regulate the blood glucose level. Other pancreatic endocrine hormones are glucagon and somatostatin. The exocrine pancreas produces digestive enzymes such as amylase, lipase, and trypsin.

A nurse is collecting data on a client admitted to the hospital with a diagnosis of myxedema. Which data collection technique will provide data necessary to support the admitting diagnosis?

1. Auscultation of lung sounds *2. Inspection of facial features* 3. Percussion of the thyroid gland 4. Palpation of the adrenal glands *rationale* Inspection of facial features will reveal the characteristic coarse features, presence of edema around the eyes and face, and a blank expression that are characteristic of myxedema. The techniques in the remaining options will not reveal any data that would support the diagnosis of myxedema.

A hospitalized client is newly diagnosed with diabetes mellitus. The client must take both NPH and Regular insulin for glucose control. The nurse develops a teaching plan to help the client meet which outcome as a first step in managing the disease?

1. Avoid all strenuous exercise. 2. Maintain health at an optimum level. 3. Lose 40 pounds to achieve ideal body weight. *4. Adjust insulin according to capillary blood glucose levels.* *rationale* There are many learning goals for the client who is newly diagnosed with diabetes mellitus. The client must learn dietary control, medication management, and proper exercise in order to control the disease. As a first step, the client learns to adjust medication (insulin) according to blood glucose results as prescribed by the health care provider. The client should then focus on long-term dietary control and weight loss, which will often lead to a decreased need for insulin. At the same time that diet is being controlled, the client should begin a regular exercise program to aid in weight loss.

A client is diagnosed with hyperparathyroidism. The nurse teaching the client about dietary alterations to manage the disorder tells the client to limit which of the following foods in the diet?

1. Bananas 2. Oatmeal *3. Ice cream* 4. Chicken breast *rationale* The client with hyperparathyroidism is likely to have elevated calcium levels. This client should reduce intake of dairy products such as milk, cheese, ice cream, or yogurt. Apples, bananas, chicken, oatmeal, and pasta are low-calcium foods.

A nurse is monitoring a client with Graves' disease for signs of thyrotoxic crisis (thyroid storm). Which of the following signs and symptoms, if noted in the client, will alert the nurse to the presence of this crisis? *Select all that apply.*

1. Bradycardia *2. Fever* *3. Sweating* *4. Agitation* 5. Pallor *rationale* Thyrotoxic crisis (thyroid storm) is an acute, potentially life-threatening state of extreme thyroid activity that represents a breakdown in the body's tolerance to a chronic excess of thyroid hormones. The clinical manifestations include fever greater than 100° F, severe tachycardia, flushing and sweating, and marked agitation and restlessness. Delirium and coma can occur.

A nurse is caring for a client after thyroidectomy and monitoring for signs of thyroid storm. The nurse understands that which of the following is a manifestation associated with this disorder?

1. Bradycardia *2. Hypotension* 3. Constipation 4. Hypothermia *rationale* Clinical manifestations associated with thyroid storm include a fever as high as 106° F (41.1° C), severe tachycardia, profuse diarrhea, extreme vasodilation, hypotension, atrial fibrillation, hyperreflexia, abdominal pain, diarrhea, and dehydration. With this disorder, the client's condition can rapidly progress to coma and cardiovascular collapse.

A client with a pituitary tumor will undergo transsphenoidal hypophysectomy. The nurse includes which priority item in the preoperative teaching plan for the client?

1. Brushing the teeth vigorously and frequently is important to minimize bacteria in the mouth. *2. Blowing the nose following surgery is prohibited.* 3. A small area will be shaved at the base of the neck. 4. It will be necessary to cough and deep breathe following the surgery. *rationale* The approach used for this surgery is the oronasal route, specifically where the upper lip meets the gum. The surgeon then uses a route through the sphenoid sinus to get to the pituitary gland. The client is not allowed to blow the nose, sneeze, or cough vigorously because these activities could raise intracranial pressure. The client also is not allowed to brush the teeth, to avoid disrupting the surgical site. Alternate methods for performing mouth care are used.

The wife of a client with diabetes mellitus who takes insulin calls the nurse in a health care provider's office about her husband. She states that her husband is sleepy and that his skin is warm and flushed. She adds that his breathing is faster than normal and his pulse rate seems fast. Which of the following should the nurse tell the woman to do first?

1. Call an ambulance. 2. Take his temperature. *3. Check his blood glucose level.* 4. Drive him to the health care provider's office. *rationale* The client's signs and symptoms are consistent with hyperglycemia. The wife should first obtain a blood glucose reading, which the nurse would then report to the health care provider. Option 1 or 4 may be done at a later time if required. Option 2 is unrelated to the client's immediate problem.

A nurse is caring for a client with a diagnosis of myasthenia gravis. The health care provider plans to perform an Enlon test on the client to determine the presence of cholinergic crisis. In addition to planning care for the client during this testing, which of the following will the nurse ensure is at the bedside?

1. Cardiac monitor *2. Oxygen equipment* 3. Vial of protamine sulfate and a syringe 4. Potassium injection and a liter of normal saline solution *rationale* An Enlon test is performed to distinguish between myasthenic and cholinergic crisis. Following administration of Enlon, if symptoms intensify, the crisis is cholinergic. Because the symptoms of cholinergic crisis will worsen with the administration of Enlon, atropine sulfate and oxygen should be immediately available whenever Enlon is used.

A nurse is collecting data regarding a client after a thyroidectomy and notes that the client has developed hoarseness and a weak voice. Which nursing action is appropriate?

1. Check for signs of bleeding. 2. Administer calcium gluconate. 3. Notify the registered nurse immediately. *4. Reassure the client that this is usually a temporary condition.* *rationale* Weakness and hoarseness of the voice can occur as a result of trauma of the laryngeal nerve. If this develops, the client should be reassured that the problem will subside in a few days. Unnecessary talking should be discouraged. It is not necessary to notify the registered nurse immediately. These signs do not indicate bleeding or the need to administer calcium gluconate.

A nurse working on an endocrine nursing unit understands that which correct concept is used in planning care?

1. Clients with Cushing's syndrome are likely to experience episodic hypotension. 2. Clients with hyperthyroidism must be monitored for weight gain. 3. Clients who have diabetes insipidus should be assessed for fluid excess. *4. Clients who have hyperparathyroidism should be protected against falls.* *rationale* Hyperparathyroidism is a disease that involves excess secretion of parathyroid hormone (PTH). Elevation of PTH causes excess calcium to be removed from the bones. There is a decline in bone mass, which may cause a fracture if a fall occurs. Cushing's syndrome is likely to cause hypertension. Clients with hypothyroidism must be monitored for weight gain and clients with hyperthyroidism must be monitored for weight loss. Clients who have diabetes insipidus should be assessed for fluid deficit.

A nurse is planning to instruct a client with diabetes mellitus who has hypertension about "sick day management." Which of the following does the nurse avoid putting on a list of easily consumed carbohydrate-containing beverages for use when the client cannot tolerate food orally?

1. Cola 2. Ginger ale 3. Apple juice *4. Mineral water* *rationale* Diabetic clients should take in approximately 15 g of carbohydrate every 1 to 2 hours when unable to tolerate food because of illness. Each of the beverages listed in options 1, 2, and 3 provides approximately 13 to 15 g of carbohydrate in a half-cup serving. Mineral water is incorrect for two reasons. First, it contains sodium and should not be used by the client with hypertension. Second, it is not a source of carbohydrates.

What is the treatment for Addison's disease?

1. Combat shock 2. Increase Na+ - juice, canned soup 3. daily wt's, maintain 2-3lbs -adjust meds 4. mineralocorticoid durgs

A client who returned to the nursing unit 8 hours ago after hypophysectomy has clear drainage saturating the nasal dressing. The nurse should take which action first?

1. Continue to observe for further drainage. *2. Test the drainage for glucose.* 3. Put the head of the bed flat. 4. Test the drainage for occult blood. *rationale* Following hypophysectomy the client should be monitored for rhinorrhea (clear nasal drainage), which could indicate a cerebrospinal fluid (CSF) leak. If this occurs, the drainage should be collected and tested for the presence of CSF by testing it for glucose. CSF tests positive for glucose, whereas true nasal secretions would not. It is not necessary to test drainage that is clear for occult blood. The head of the bed should not be lowered, to prevent a rise in intracranial pressure. Continuing to observe the drainage without taking action could put the client at risk for developing a serious complication.

A nurse is caring for a client with pheochromocytoma. The client asks for a snack and something warm to drink. The appropriate choice for this client to meet nutritional needs would be which of the following?

1. Crackers with cheese and tea *2. Graham crackers and warm milk* 3. Toast with peanut butter and cocoa 4. Vanilla wafers and coffee with cream and sugar *rationale* The client with pheochromocytoma needs to be provided with a diet that is high in vitamins, minerals, and calories. Of particular importance is that food or beverages that contain caffeine (e.g., chocolate, coffee, tea, and cola) are prohibited.

A nurse is providing instructions to a client newly diagnosed with diabetes mellitus. The nurse gives the client a list of the signs of hyperglycemia. Which of the following specific signs of this complication should be included on the list?

1. Decreased urine output 2. Profuse sweating *3. Increased thirst* 4. Shakiness *rationale* The classic signs of hyperglycemia include polydipsia, polyuria, and polyphagia. Profuse sweating and shakiness would be noted in a hypoglycemic condition.

The nurse caring for a client who has had a subtotal thyroidectomy reviews the plan of care and determines which problem is the priority for this client in the immediate postoperative period?

1. Dehydration 2. Infection 3. Urinary retention *4. Bleeding* *rationale* Hemorrhage is one of the most severe complications that can occur following thyroidectomy. The nurse must frequently check the neck dressing for bleeding and monitor vital signs to detect early signs of hemorrhage, which could lead to shock. T3 and T4 do not regulate fluid volumes in the body. Infection is a concern for any postoperative client but is not the priority in the immediate postoperative period. Urinary retention can occur in postoperative clients as a result of medication and anesthesia but is not the priority from the options provided.

A client with type 2 diabetes mellitus has a blood glucose of more than 600 mg/dL and is complaining of polydipsia, polyuria, weight loss, and weakness. The nurse reviews the health care provider's documentation and would expect to note which of the following diagnoses?

1. Diabetic ketoacidosis (DKA) 2. Hypoglycemia *3. Hyperglycemic hyperosmolar nonketotic syndrome (HHNS)* 4. Pheochromocytoma *rationale* HHNS is seen primarily in individuals with type 2 diabetes who experience a relative deficiency of insulin. The onset of symptoms may be gradual. The symptoms may include polyuria, polydipsia, dehydration, mental status alterations, weight loss, and weakness. DKA normally occurs in type 1 diabetes mellitus. The clinical manifestations noted in the question are not signs of hypoglycemia. Pheochromocytoma is not related to these clinical manifestations.

A nurse is collecting data from a client who is being admitted to the hospital for a diagnostic workup for primary hyperparathyroidism. The nurse understands that which client complaint would be characteristic of this disorder?

1. Diarrhea *2. Polyuria* 3. Polyphagia 4. Weight gain *rationale* Hypercalcemia is the hallmark of hyperparathyroidism. Elevated serum calcium levels produce osmotic diuresis (polyuria). This diuresis leads to dehydration and the client would lose weight. Options 1, 3, and 4 are gastrointestinal (GI) symptoms but are not associated with the common GI symptoms typical of hyperparathyroidism (nausea, vomiting, anorexia, constipation).

Following hypophysectomy, a client complains of being very thirsty and having to urinate frequently. The initial nursing action is to:

1. Document the complaints. 2. Increase fluid intake. *3. Check the urine specific gravity.* 4. Check for urinary glucose. *rationale* Following hypophysectomy, diabetes insipidus can occur temporarily because of antidiuretic hormone deficiency. This deficiency is related to surgical manipulation. The nurse should check the urine for specific gravity and report the results if they are less than 1.005. Urinary glucose and diabetes mellitus is not a concern here. In this situation, increasing fluid intake would require a health care provider's prescription. The client's complaint would be documented but not as an initial action.

A nurse is caring for a client following an adrenalectomy and is monitoring for signs of adrenal insufficiency. Which of the following, if noted in the client, indicates signs and symptoms related to adrenal insufficiency? *Select all that apply.*

1. Double vision *2. Hypotension* *3. Mental status changes* *4. Weakness* *5. Fever* *rationale* The nurse should be alert to signs and symptoms of adrenal insufficiency in a client following adrenalectomy. These signs and symptoms include weakness, hypotension, fever, and mental status changes. Double vision is generally not associated with this condition.

A nurse is reinforcing instructions with a client with diabetes mellitus who is recovering from diabetic ketoacidosis (DKA) regarding measures to prevent a recurrence. Which instruction is important for the nurse to emphasize?

1. Eat six small meals daily. 2. Test the urine ketone levels. *3. Monitor blood glucose levels frequently.* 4. Receive appropriate follow-up health care. *rationale* Client education after DKA should emphasize the need for home glucose monitoring four to five times per day. It is also important to instruct the client to notify the health care provider when illness occurs. The presence of urinary ketones indicates that DKA has already occurred. The client should eat well-balanced meals with snacks, as prescribed.

Which clinical manifestation should the nurse expect to note when assessing a client with Addison's disease?

1. Edema 2. Obesity 3. Hirsutism *4. Hypotension* *rationale* Common manifestations of Addison's disease include postural hypotension from fluid loss, syncope, muscle weakness, anorexia, nausea, vomiting, abdominal cramps, weight loss, depression, and irritability. The manifestations in options 1, 2, and 3 are not associated with Addison's disease.

A nursing student notes in the medical record that a client with Cushing's syndrome is experiencing body image disturbances. The need for additional education regarding this problem is identified when the nursing student suggests which nursing intervention?

1. Encouraging the client's expression of feelings 2. Evaluating the client's understanding of the disease process 3. Encouraging family members to share their feelings about the disease process *4. Evaluating the client's understanding that the body changes need to be dealt with* *rationale* Evaluating the client's understanding that the body changes that occur in this disorder need to be dealt with is an inappropriate nursing intervention. This option does not address the client's feelings. Options 1, 2, and 3 are appropriate because they address the client and family feelings regarding the disorder.

A nurse caring for a client scheduled for a transsphenoidal hypophysectomy to remove a tumor in the pituitary gland assists to develop a plan of care for the client. The nurse suggests including which specific information in the preoperative teaching plan?

1. Hair will need to be shaved. 2. Deep breathing and coughing will be needed after surgery. *3. Toothbrushing will not be permitted for at least 2 weeks following surgery.* 4. Spinal anesthesia is used. *rationale* Based on the location of the surgical procedure, spinal anesthesia would not be used. In addition, the hair would not be shaved. Although coughing and deep breathing are important, specific to this procedure is avoiding toothbrushing to prevent disruption of the surgical site. Also, coughing may disrupt the surgical site.

A client has been diagnosed with hypoparathyroidism. The nurse teaches the client to include foods in the diet that are:

1. High in phosphorus and low in calcium 2. Low in phosphorus and low in calcium *3. Low in phosphorus and high in calcium* 4. High in phosphorus and high in calcium *rationale* Hypoparathyroidism results in hypocalcemia. A therapeutic diet for this disorder is one that is high in calcium but low in phosphorus because these two electrolytes have inverse proportions in the body. All of the other options are unrelated to this disorder and are incorrect.

A client is brought to the emergency department with suspected diabetic ketoacidosis (DKA). Which of the following findings would the nurse note as being consistent with this diagnosis?

1. High serum glucose level and an increase in pH 2. Low serum potassium and high serum bicarbonate level *3. High serum glucose level and low serum bicarbonate level* 4. Decreased urine output and Kussmaul's respirations *rationale* In DKA the blood glucose level is higher than 250 mg/dL, and ketones are present in the blood and urine. The arterial pH is low, less than 7.35. The plasma bicarbonate is also low. The client would exhibit polyuria and Kussmaul's respirations. The potassium level usually is elevated as a result of dehydration.

A comatose client with an admitting diagnosis of diabetic ketoacidosis (DKA) has a blood glucose of 368 mg/dL, arterial pH of 7.2, arterial bicarbonate of 14 mEq/L, and a positive for serum ketones. The diagnosis is supported by which noted data?

1. Hypertension *2. Fruity breath odor* 3. Slow regular breathing 4. Moist mucous membranes *rationale* Diabetic ketoacidotic coma is usually identified with a fruity breath odor, dry cracked mucous membranes, hypotension, and rapid deep breathing.

A nurse is caring for a client experiencing thyroid storm. Which of the following would be a priority concern for this client?

1. Inability to cope with the treatment plan 2. Lack of sexual drive 3. Self-consciousness about body appearance *4. Potential for cardiac disturbances* *rationale* Clients in thyroid storm are experiencing a life-threatening event, which is associated with uncontrolled hyperthyroidism. It is characterized by high fever, severe tachycardia, delirium, dehydration, and extreme irritability. The signs and symptoms of the disorder develop quickly, and therefore emergency measures must be taken to prevent death. These measures include maintaining hemodynamic status and patency of airway as well as providing adequate ventilation. Options 1, 2, and 3 are not a priority in the care of the client in thyroid storm.

A client scheduled for a thyroidectomy says to the nurse, "I am so scared to get cut in my neck." Based on the client's statement, the nurse determines that the client is experiencing which problem?

1. Inadequate knowledge about the surgical procedure *2. Fear about impending surgery* 3. Embarrassment about the changes in personal appearance 4. Lack of support related to the surgical procedure *rationale* The client is having a difficult time coping with the scheduled surgery. The client is able to express fears but is scared. No data in the question support options 1, 3, and 4.

17. Untreated hyperthyroidism during pregnancy may result in all of the following except: a. Premature birth and miscarriage b. Low birthweight c. Autism d. Preeclampsia

17. C: In addition to the above-mentioned complications of uncontrolled hyperthyroidism in pregnancy, expectant mothers may suffer congestive heart failure and thyroid storm, which is life-threatening thyrotoxicosis with symptoms that include agitation, confusion, tachycardia, shaking, sweating, diarrhea, fever, and restlessness.

A nurse is caring for a postoperative parathyroidectomy client. Which of the following would require the nurse's immediate attention?

1. Incisional pain *2. Laryngeal stridor* 3. Difficulty voiding 4. Abdominal cramps *rationale* During the postoperative period, the nurse carefully observes the client for signs of hemorrhage, which cause swelling and the compression of adjacent tissue. Laryngeal stridor is a harsh, high-pitched sound heard on inspiration and expiration that is caused by the compression of the trachea and that leads to respiratory distress. It is an acute emergency situation that requires immediate attention to avoid the complete obstruction of the airway.

A nurse is caring for a client diagnosed with hyperparathyroidism who is prescribed furosemide (Lasix). The nurse reinforces dietary instructions to the client. Which of the following is an appropriate instruction?

1. Increase dietary intake of calcium. *2. Drink at least 2 to 3 L of fluid daily.* 3. Eat sparely when experiencing nausea. 4. Decrease dietary intake of potassium. *rationale* The aim of treatment in the client with hyperparathyroidism is to increase the renal excretion of calcium and decrease gastrointestinal absorption and bone resorption. This is aided by the sufficient intake of fluids. Dietary restriction of calcium may be used as a component of therapy. The parathyroid is responsible for calcium production, and the term, "hyperparathyroidism" can be indicative of an increase in calcium. The client should eat foods high in potassium, especially if the client is taking furosemide. Limiting nutrients is not advisable.

A nurse is caring for a client following a thyroidectomy. The client tells the nurse that she is concerned because of voice hoarseness. The client asks the nurse whether the hoarseness will subside. The nurse appropriately tells the client that the hoarseness:

1. Indicates nerve damage 2. Is harmless but permanent 3. Will worsen before it subsides *4. Is normal and will gradually subside* *rationale* Hoarseness that develops in the postoperative period is usually the result of laryngeal pressure or edema and will resolve within a few days. The client should be reassured that the effects are transitory. Options 1, 2, and 3 are incorrect.

A nurse is reinforcing instructions to a client newly diagnosed with diabetes mellitus regarding insulin administration. The health care provider has prescribed a mixture of NPH and regular insulin. The nurse should stress that the first step is to:

1. Inject air equal to the amount of regular insulin prescribed into the vial of regular insulin. *2. Inject air equal to the amount of NPH insulin prescribed into the vial of NPH insulin.* 3. Draw up the correct dosage of regular insulin into the syringe. 4. Draw up the correct dosage of NPH insulin into the syringe. *rationale* The initial step in preparing an injection of insulin that is a mixture of NPH and regular is to inject air into the NPH bottle equal to the amount of insulin prescribed. The client is instructed to next inject an amount of air equal to the amount of prescribed insulin into the regular insulin bottle. The regular insulin should then be withdrawn followed by the NPH insulin. Contamination of regular insulin with NPH insulin will convert part of the regular insulin into a longer-acting form.

A nurse is reviewing the postoperative prescriptions for a client who had a transsphenoidal hypophysectomy. Which health care provider's prescription, if noted on the record, indicates the need for clarification?

1. Instruct the client about the need for a Medic-Alert bracelet. *2. Apply a loose dressing if any clear drainage is noted.* 3. Monitor vital signs and neurological status. 4. Instruct the client to avoid blowing the nose. *rationale* The nurse should observe for clear nasal drainage, constant swallowing, and a severe, persistent, generalized, or frontal headache. These signs and symptoms indicate cerebrospinal fluid leak into the sinuses. If clear drainage is noted following this procedure, the health care provider needs to be notified immediately. Options 1, 3, and 4 indicate appropriate postoperative interventions.

A preoperative client is scheduled for adrenalectomy to remove a pheochromocytoma. The nurse would most closely monitor which of the following items in the preoperative period?

1. Intake and output 2. Blood urea nitrogen (BUN) *3. Vital signs* 4. Urine glucose and ketones *rationale* Hypertension is the hallmark of pheochromocytoma. Severe hypertension can precipitate a stroke or sudden blindness. Although all the items are appropriate nursing assessments for the client with pheochromocytoma, the priority is to monitor the vital signs, especially the blood pressure.

A client who is managing diabetes mellitus with insulin injections asks the nurse for information about any necessary changes in her diet to avoid hyperinsulinism. Which of the following diets would be appropriate for the client?

1. Low-fiber, high-fat diet 2. Limit carbohydrate intake to three meals per day 3. Large amounts of carbohydrates between low protein meals *4. Small frequent meals with protein, fat, and carbohydrates at each meal* *rationale* The definition of hyperinsulinism is an excessive insulin secretion in response to carbohydrate-rich foods leading to hypoglycemia. It is often treated with a diet that provides for limited stimulation of the pancreas. Carbohydrates can produce a rapid rise in blood glucose levels. However, carbohydrates are necessary in the diet. Proteins do not stimulate insulin secretion. Fats are needed in the diet to provide calories. The best diet for hyperinsulinism will contain proteins and fats whenever carbohydrates are consumed and delivered in frequent but portion-controlled meals. Diets high in soluble fiber may be beneficial.

A nurse is preparing to provide instructions to a client with Addison's disease regarding diet therapy. The nurse understands that which of the following diets would likely be prescribed for this client?

1. Low-protein diet 2. Low-sodium diet *3. High-sodium diet* 4. Low-carbohydrate diet *rationale* A high-sodium, high-complex carbohydrate, and high-protein diet will be prescribed for the client with Addison's disease. To prevent excess fluid and sodium loss, the client is instructed to maintain an adequate salt intake of up to 8 g of sodium daily and to increase salt intake during hot weather, before strenuous exercise, and in response to fever, vomiting, or diarrhea.

When caring for a client who is having clear drainage from his nares after transsphenoidal hypophysectomy, which action by the nurse is appropriate?

1. Lower the head of the bed. *2. Test the drainage for glucose.* 3. Obtain a culture of the drainage. 4. Continue to observe the drainage. *rationale* After hypophysectomy, the client should be monitored for rhinorrhea, which could indicate a cerebrospinal fluid (CSF) leak. If this occurs, the drainage should be collected and tested for glucose, indicating the presence of CSF. The head of the bed should not be lowered to prevent increased intracranial pressure. Clear nasal drainage would not indicate the need for a culture. Continuing to observe the drainage without taking action could result in a serious complication.

A nurse reviews a plan of care for a postoperative client following a thyroidectomy and notes that the client is at risk for breathing difficulty. Which of the following nursing interventions will the nurse suggest to include in the plan of care?

1. Maintain a supine position. 2. Encourage coughing and deep breathing exercises. *3. Monitor neck circumference frequently.* 4. Maintain a pressure dressing on the operative site. *rationale* Following a thyroidectomy, the client should be placed in an upright position to facilitate air exchange. The nurse should assist the client with deep breathing exercises, but coughing is minimized to prevent tissue damage and stress to the incision. A pressure dressing is not placed on the operative site because it could affect breathing. The nurse should monitor the dressing closely and should loosen the dressing if necessary. Neck circumference is monitored at least every 4 hours to assess for postoperative edema.

A client is brought to the emergency department in an unresponsive state, and a diagnosis of hyperglycemic hyperosmolar nonketotic syndrome (HHNS) is made. The nurse who is assisting to care for the client obtains which of the following immediately in preparation for the treatment of this syndrome?

1. NPH insulin 2. A nasal cannula 3. Intravenous (IV) infusion of sodium bicarbonate *4. IV infusion of normal saline* *rationale* The primary goal of treatment is to rehydrate the client to restore fluid volume and to correct electrolyte deficiency. IV fluid replacement is similar to that administered in diabetic ketoacidosis (DKA) and begins with IV infusion of normal saline. Regular insulin, not NPH insulin would be administered. The use of sodium bicarbonate to correct acidosis is avoided because it can precipitate a further drop in serum potassium levels. A nasal cannula for oxygen administration is not necessarily required to treat HHNS.

An adult client just admitted to the hospital with heart failure also has a history of diabetes mellitus. The nurse calls the health care provider to verify a prescription for which medication that the client was taking before admission?

1. NPH insulin 2. Regular insulin 3. Acarbose (Precose) *4. Chlorpropamide* *rationale* Chlorpropamide is an oral hypoglycemic agent that exerts an antidiuretic effect and should be administered cautiously or avoided in the client with cardiac impairment or fluid retention. It is a first-generation sulfonylurea. Insulin does not cause or aggravate fluid retention. Acarbose is a miscellaneous oral hypoglycemic agent.

A client newly diagnosed with diabetes mellitus takes NPH insulin every day at 7:00 ᴀᴍ. The nurse has taught the client how to recognize the signs of hypoglycemia. The nurse determines that the client understands the information presented if the client watches for which of the following signs in the late afternoon?

1. Nausea and vomiting, and abdominal pain *2. Hunger; shakiness; and cool, clammy skin* 3. Drowsiness; red, dry skin; and fruity breath odor 4. Increased urination; thirst; and rapid, deep breathing *rationale* The client taking NPH insulin obtains peak medication effects approximately 6 to 12 hours after administration. At the time that the medication peaks, the client is at risk of hypoglycemia if food intake is insufficient. The nurse should teach the client to watch for signs and symptoms of hypoglycemia including anxiety, confusion, difficulty concentrating, blurred vision, cold sweating, headache, increased pulse, shakiness, and hunger. The other options list various signs and symptoms of hyperglycemia.

A client with Cushing's disease is being admitted to the hospital after a stab wound to the abdomen. The nurse plans care and places highest priority on which potential problem?

1. Nervousness *2. Infection* 3. Concern about appearance 4. Inability to care for self *rationale* The client with a stab wound has a break in the body's first line of defense against infection. The client with Cushing's disease is at great risk for infection because of excess cortisol secretion and subsequent impaired antibody function and decreased proliferation of lymphocytes. The client may also have a potential for the problems listed in the other options but these are not the highest priority at this time.

A client has a blood glucose level drawn for suspected hyperglycemia. After interviewing the client, the nurse determines that the client ate lunch approximately 2 hours before the blood specimen was drawn. The laboratory reports that the blood glucose to be 180 mg/dL, and the nurse analyzes this result to be:

1. Normal 2. Lower than the normal value *3. Elevated from the normal value* 4. A dangerously high value requiring immediate health care provider notification *rationale* Normal fasting blood glucose values range from 70 to 120 mg/dL. A 2-hour postprandial blood glucose level should be less than 140 mg/dL. In this situation, the blood glucose value was 180 mg/dL 2 hours after the client ate, which is an elevated value as compared to normal. Although the result may be reported to the health care provider, it is not a dangerously high one.

A client with diabetes mellitus calls the clinic nurse to report that the blood glucose level is 150 mg/dL. After obtaining further data from the client, the nurse determines that the client ate lunch approximately 2 hours ago. The nurse would interpret these results to be:

1. Normal 2. Lower than the normal value *3. Slightly higher than the normal value* 4. A value that indicates immediate health care provider notification *rationale* Normal fasting blood glucose values range from 70 to 120 mg/dL. The 2-hour postprandial blood glucose level should be less than 140 mg/dL. In this situation, the blood glucose value was 150 mg/dL 2 hours after the client ate, which is slightly elevated above normal. This value does not require health care provider notification.

A male client recently diagnosed with diabetes mellitus requiring insulin tells the clinic nurse that he is traveling by air throughout the next week. The client asks the nurse for any suggestions about managing the disorder while traveling. The nurse tells the client to:

1. Obtain referrals to health care providers in the destination cities. 2. Check the blood glucose every 2 hours during the flight. *3. Keep snacks in carry-on luggage to prevent hypoglycemia during the flight.* 4. Pad the insulin and syringes against breakage and place in a suitcase to be stowed. *rationale* A frequent concern of diabetics during air travel is the availability of food at times that correspond with the timing and peak action of the client's insulin. For this reason, the nurse may suggest that the client have carbohydrate snacks on hand. Insulin equipment and supplies should always be placed in carry-on luggage (not stowed). This provides ready access to treat hyperglycemia, if needed, and prevents loss of equipment if luggage is lost. Options 1 and 2 are unnecessary.

A maternity nursing instructor asks a nursing student to identify the hormones that are produced by the ovaries. Which of the following, if identified by the student, indicates an understanding of the hormones produced by this endocrine gland?

1. Oxytocin 2. Luteinizing hormone (LH) *3. Estrogen and progesterone* 4. Follicle-stimulating hormone (FSH) *rationale* The ovaries are the endocrine glands that produce estrogen and progesterone. Oxytocin is produced by the posterior pituitary gland and stimulates the uterus to produce contractions. LH and FSH are produced by the anterior pituitary gland.

A nurse reviews the nursing care plan of an older client with diabetic neuropathy of the lower extremities as a result of type 2 diabetes mellitus. The nurse plans care, knowing that which problem has the highest priority for this client?

1. Pain as a result of intermittent claudication 2. Lack of self-confidence as a result of impaired ability to walk 3. Lack of self-esteem as a result of perceived loss of abilities *4. The possibility of injury as a result of decreased sensation in the legs and feet* *rationale* The client with diabetic neuropathy of the lower extremities has diminished ability to feel sensations in the legs and feet. This client is at risk for tissue injury and for falls as a result of this nervous system impairment. Thus the highest priority problem is option 4, which can be determined using Maslow's Hierarchy of Needs theory. Options 2 and 3 represent problems that are more psychosocial in nature, and as such are secondary needs using Maslow's theory. Option 1 is incorrect because intermittent claudication is not directly associated with diabetic neuropathy.

A nurse is caring for a client with hypothyroidism who is overweight. Which food items would the nurse suggest to include in the plan?

1. Peanut butter, avocado, and red meat *2. Skim milk, apples, whole-grain bread, and cereal* 3. Organ meat, carrots, and skim milk 4. Seafood, spinach, and cream cheese *rationale* Clients with hypothyroidism may have a problem with being over-weight because of their decreased metabolic need. They should consume foods from all food groups, which will provide them with the necessary nutrients; however, the foods should be low in calories. Option 2 is the only option that identifies food items that are low in calories.

A nurse is caring for a postoperative adrenalectomy client. Which of the following does the nurse specifically monitor for in this client?

1. Peripheral edema 2. Bilateral exophthalmos 3. Signs and symptoms of hypocalcemia *4. Signs and symptoms of hypovolemia* *rationale* Following adrenalectomy, the client is at risk for hypovolemia. Aldosterone, secreted by the adrenal cortex, plays a major role in fluid volume balance by retaining sodium and water. A deficiency of adrenocortical hormones does not cause the clinical manifestations noted in options 1, 2, and 3.

A nurse reinforces teaching with a client with diabetes mellitus regarding differentiating between hypoglycemia and ketoacidosis. The client demonstrates an understanding of the teaching by stating that glucose will be taken if which symptom develops?

1. Polyuria *2. Shakiness* 3. Blurred vision 4. Fruity breath odor *rationale* Shakiness is a sign of hypoglycemia, and it would indicate the need for food or glucose. Fruity breath odor, blurred vision, and polyuria are signs of hyperglycemia.

A client is in metabolic acidosis caused by diabetic ketoacidosis (DKA). The nurse prepares for the administration of which of the following medications as a primary treatment for this problem?

1. Potassium *2. Regular insulin* 3. Sodium bicarbonate 4. Calcium gluconate *rationale* The primary treatment for any acid-base imbalance is treatment of the underlying disorder that caused the problem. In this case, the underlying cause of the metabolic acidosis is anaerobic metabolism as a result of the lack of ability to use circulating glucose. Administration of regular insulin corrects this problem.

A nurse enters the room of a client with type 1 diabetes mellitus and finds the client difficult to arouse. The client's skin is warm and flushed, and the pulse and respiratory rate are elevated from the client's baseline. The nurse would immediately:

1. Prepare for the administration of an insulin drip. 2. Give the client a glass of orange juice. 3. Prepare for the administration of a bolus dose of 50% dextrose. *4. Check the client's capillary blood glucose.* *rationale* The nurse must first obtain a blood glucose reading to determine the client's problem. Options 2 and 3 would be implemented as needed in the treatment of hypoglycemia. Insulin therapy is guided by blood glucose measurement.

When caring for a client diagnosed with pheochromocytoma, what information should the nurse know when assisting with planning care?

1. Profound hypotension may occur. *2. Excessive catecholamines are released.* 3. The condition is not curable and is treated symptomatically. 4. Hypoglycemia is the primary presenting symptom. *rationale* Pheochromocytoma is a catecholamine-producing tumor of the adrenal gland and causes secretion of excessive amounts of epinephrine and norepinephrine. Hypertension is the principal manifestation, and the client has episodes of a high blood pressure accompanied by pounding headaches. The excessive release of catecholamine also results in excessive conversion of glycogen into glucose in the liver. Consequently, hyperglycemia and glucosuria occur during attacks. Pheochromocytoma is curable. The primary treatment is surgical removal of one or both of the adrenal glands, depending on whether the tumor is unilateral or bilateral.

A nurse would expect to note which interventions in the plan of care for a client with hypothyroidism? *Select all that apply.*

1. Provide a cool environment for the client. 2. Instruct the client to consume a high-fat diet. *3. Instruct the client about thyroid replacement therapy.* *4. Encourage the client to consume fluids and high-fiber foods in the diet.* *5. Instruct the client to contact the health care provider if episodes of chest pain occur.* 6. Inform the client that iodine preparations will be prescribed to treat the disorder. *rationale* The clinical manifestations of hypothyroidism are the result of decreased metabolism from low levels of thyroid hormone. Interventions are aimed at replacement of the hormones and providing measures to support the signs and symptoms related to a decreased metabolism. The nurse encourages the client to consume a well-balanced diet that is low in fat for weight reduction and high in fluids and high-fiber foods to prevent constipation. The client often has cold intolerance and requires a warm environment. The client would notify the health care provider if chest pain occurs since it could be an indication of overreplacement of thyroid hormone. Iodine preparations are used to treat hyperthyroidism. These medications decrease blood flow through the thyroid gland and reduce the production and release of thyroid hormone.

What would the nurse anticipate being included in the plan of care for a client who has been diagnosed with Graves' disease?

1. Provide a high-fiber diet. *2. Provide a restful environment.* 3. Provide three small meals per day. 4. Provide the client with extra blankets. *rationale* Because of the hypermetabolic state, the client with Graves' disease needs to be provided with an environment that is restful both physically and mentally. Six full meals a day that are well balanced and high in calories are required, because of the accelerated metabolic rate. Foods that increase peristalsis (e.g., high-fiber foods) need to be avoided. These clients suffer from heat intolerance and require a cool environment.

A nurse has collected data on a client with diabetes mellitus. Findings include a fasting blood glucose of 130 mg/dL, temperature 101° F, pulse of 88 beats per minute, respirations of 22 breaths per minute, and a blood pressure of 118/78 mm Hg. Which finding would be of concern to the nurse?

1. Pulse and respirations 2. Blood pressure 3. Blood glucose *4. Temperature* *rationale* Elevated temperature may be indicative of infection, which is a leading cause of hyperglycemic hyperosmolar nonketotic syndrome (HHNS) or diabetic ketoacidosis (DKA). Options 1, 2, and 3 are findings that are within a normal range.

A nurse assists in developing a plan of care for a client with hyperparathyroidism receiving calcitonin-human (Cibacalcin). Which outcome has the highest priority regarding this medication?

1. Relief of pain 2. Absence of side effects *3. Reaching normal serum calcium levels* 4. Verbalization of appropriate medication knowledge *rationale* Hypercalcemia can occur in clients with hyperparathyroidism, and calcitonin is used to lower plasma calcium level. The highest priority outcome in this client situation would be a reduction in serum calcium level. Option 1 is unrelated to this medication. Although options 2 and 4 are expected outcomes, they are not the highest priority for administering this medication.

Which nursing measure would be effective in preventing complications in a client with Addison's disease?

1. Restricting fluid intake 2. Offering foods high in potassium 3. Checking family support systems *4. Monitoring the blood glucose* *rationale* The decrease in cortisol secretion that characterizes Addison's disease can result in hypoglycemia. Therefore monitoring the blood glucose would detect the presence of hypoglycemia so that it can be treated early to prevent complications. Fluid intake should be encouraged to compensate for dehydration. Potassium intake should be restricted because of hyperkalemia. Option 3 is not a priority for this client.

A nurse participating in a free health screening at the local mall obtains a random blood glucose level of 200 mg/dL on an otherwise healthy client. The nurse tells the client to do which of the following as a next step?

1. Seek treatment for diabetes mellitus. 2. Ask the pharmacist about starting insulin therapy. 3. Begin blood glucose monitoring three times a day. *4. Call the health care provider to have the value rechecked as soon as possible.* *rationale* Adult diabetes mellitus can be diagnosed either by symptoms (polydipsia, polyuria, polyphagia) or by laboratory values. Diabetes is also diagnosed by an abnormal glucose tolerance test, when random plasma glucose levels are greater than 200 mg/dL, or fasting plasma glucose levels are greater than 140 mg/dL on two separate occasions. Further confirmation of this result is needed to ensure appropriate diagnosis and therapy.

A client is admitted with a diagnosis of pheochromocytoma. The nurse would monitor which of the following to detect the most common sign of pheochromocytoma?

1. Skin temperature *2. Blood pressure* 3. Urine ketones 4. Weight *rationale* Hypertension is the major symptom associated with pheochromocytoma and is monitored by taking the client's blood pressure. Glycosuria, weight loss, and diaphoresis are other clinical manifestations of pheochromocytoma; however, hypertension is the most common sign.

A nurse is assisting in preparing a plan of care for the client with diabetes mellitus and plans to reinforce the client's understanding regarding the symptoms of hypoglycemia. Which symptoms will the nurse review?

1. Slow pulse; lethargy; and warm, dry skin 2. Elevated pulse; lethargy; and warm, dry skin *3. Elevated pulse; shakiness; and cool, clammy skin* 4. Slow pulse, confusion, and increased urine output *rationale* Symptoms of mild hypoglycemia include tachycardia; shakiness; and cool, clammy skin. Options 1, 2, and 4 are not symptoms of hypoglycemia.

A nurse is assisting with preparing a teaching plan for the client with diabetes mellitus regarding proper foot care. Which instruction should be included in the plan of care?

1. Soak the feet in hot water. 2. Avoid using soap to wash the feet. *3. Apply a moisturizing lotion to dry feet, but not between the toes.* 4. Always have a podiatrist cut your toenails; never cut them yourself. *rationale* The client should use a moisturizing lotion on his or her feet, but should avoid applying the lotion between the toes. The client should also be instructed to not soak the feet and to avoid hot water to prevent burns. The client may cut the toenails straight across and even with the toe itself, but he or she should consult a podiatrist if the toenails are thick or hard to cut or if his or her vision is poor. The client should be instructed to wash the feet daily with a mild soap.

The anticipated intended effect of fludrocortisone acetate (Florinef) for the treatment of Addison's disease is to:

1. Stimulate the immune response. *2. Promote electrolyte balance.* 3. Stimulate thyroid production. 4. Stimulate thyrotropin production. *rationale* Florinef is a long-acting oral medication with mineralocorticoid and moderate glucocorticoid activity used for long-term management of Addison's disease. Mineralocorticoids act on the renal distal tubules to enhance the reabsorption of sodium and chloride ions and the excretion of potassium and hydrogen ions. In small doses, fludrocortisone acetate causes sodium retention and increased urinary potassium excretion. The client rapidly can develop hypotension and fluid and electrolyte imbalance if the medication is discontinued abruptly. Options 1, 3, and 4 are not associated with the effects of this medication.

A nurse is caring for a client with type 1 diabetes mellitus who is hyperglycemic. Which problem would the nurse consider first, when planning care for this client?

1. The need for knowledge about the diagnosis 2. Insomnia 3. Lack of appetite *4. Signs of dehydration* *rationale* Hyperglycemia can develop into ketoacidosis in the client with type 1 diabetes mellitus. Polyuria develops as the body attempts to get rid of the excess glucose, and the client will lose large amounts of fluid. Because glucose is hyperosmotic, fluid is pulled from the tissue. Nausea and vomiting can occur as a result of hyperglycemia and can lead to a loss of sodium and water. Water also is lost from the lungs in an attempt to get rid of excess carbon dioxide. The severe dehydration that occurs can lead to hypovolemic shock. Of the problems listed, dehydration is considered first.

A 61-year-old female patient admitted with pneumonia has a total serum calcium level of 13.3 mg/dL (3.3 mmol/L). The nurse will anticipate the need to teach the patient about testing for _____ levels. a. calcitonin b. catecholamine c. thyroid hormone d. parathyroid hormone

D

A nurse is providing discharge instructions to a client who had a unilateral adrenalectomy. Which of the following will be a component of the instructions?

1. The reason for maintaining a diabetic diet *2. Instructions about early signs of a wound infection* 3. Teaching regarding proper application of an ostomy pouch 4. The need for lifelong replacement of all adrenal hormones *rationale* A client who is undergoing a unilateral adrenalectomy will be placed on corticosteroids temporarily to avoid a cortisol deficiency. These medications will be gradually weaned in the postoperative period until they are discontinued. Because of the anti-inflammatory properties of corticosteroids, clients who undergo an adrenalectomy are at increased risk for developing wound infections. Because of this increased risk for infection, it is important for the client to know measures to prevent infection, early signs of infection, and what to do if an infection is present. Options 1, 3, and 4 are incorrect instructions.

A nurse is discussing foot care with a diabetic client and spouse. The nurse includes which of the following during this informational session?

1. There is decreased risk of infection when feet are soaked in hot water. 2. Lanolin should be applied to dry feet, especially the heels and between the toes. *3. The toenails should be cut straight across.* 4. Strong soap should be used to decrease skin bacteria. *rationale* The client should be instructed to cut the toenails straight across. The client should not soak the feet in hot water, to prevent burns. The client should be instructed to wash the feet daily using a mild soap. Moisturizing lotion can be applied to the feet but should not be placed between the toes.

A client with newly diagnosed Cushing's syndrome expresses concern about personal appearance, specifically about the "buffalo hump" that has developed at the base of the neck. When counseling the client about this manifestation, the nurse should incorporate the knowledge that:

1. This is a permanent feature. 2. It can be minimized by wearing tight clothing. *3. It may slowly improve with treatment of the disorder.* 4. It will quickly disappear once medication therapy is started. *rationale* The client with Cushing's syndrome should be reassured that most physical changes resolve over time with treatment. The other options are incorrect.

Glucagon hydrochloride injection would most likely be prescribed for which disorder?

1. Thyroid crisis *2. Type 1 diabetes mellitus* 3. Hypoadrenalism 4. Excess growth hormone secretion *rationale* Glucagon hydrochloride is a medication that can be administered subcutaneously or intramuscularly. It is prescribed to stimulate the liver to release glucose when a client is experiencing hypoglycemia and unable to take oral glucose replacement. It is important to teach a person other than the client how to administer the medication because the client's symptoms may prevent self-injection. Therefore options 1, 3, and 4 are incorrect.

A client with type 1 diabetes mellitus is to begin an exercise program, and the nurse is providing instructions to the client regarding the program. Which of the following should the nurse include in the teaching plan?

1. Try to exercise before mealtime. 2. Administer insulin after exercising. *3. Take a blood glucose test before exercising.* 4. Exercise should be performed during peak times of insulin. *rationale* A blood glucose test performed before exercising provides information to the client regarding the need to eat a snack first. Exercising during the peak times of insulin effect or before mealtime places the client at risk for hypoglycemia. Insulin should be administered as prescribed.

A nurse is preparing to administer an injection of regular insulin. The vial of the regular insulin has been refrigerated. On inspection of the vial, the nurse finds that the medication is frozen. The nurse should:

1. Wait for the insulin to thaw at room temperature. 2. Check the temperature settings of the refrigerator. *3. Discard the insulin and obtain another vial.* 4. Rotate the vial between the hands until the medication becomes liquid. *rationale* Insulin preparations are stable at room temperature for up to 1 month without significant loss of activity. Insulin should not be frozen. If the insulin is frozen, it should be discarded and the nurse should obtain another vial. Options 1, 2, and 4 are incorrect.

A client newly diagnosed with diabetes mellitus is having difficulty learning the technique of blood glucose measurement. The nurse should teach the client to do which of the following to perform the procedure properly?

1. Wash the hands first using cold water. 2. Puncture the center of the finger pad. 3. Puncture the finger as deeply as possible. *4. Let the arm hang dependently and milk the digit.* *rationale* Before doing a fingerstick for blood glucose measurement, the client should first wash the hands. Warm water should be used to stimulate the circulation to the area. The finger is punctured near the side, not the center, because there are fewer nerve endings along the side of the finger. The puncture is only deep enough to obtain an adequately sized drop of blood; excessively deep punctures may lead to pain and bruising. The arm should be allowed to hang dependently, and the finger may be milked to promote obtaining a good-size blood drop.

Which nursing action would be appropriate to implement when a client has a diagnosis of pheochromocytoma?

1. Weigh the client. 2. Test the client's urine for glucose. *3. Monitor the client's blood pressure.* 4. Palpate the client's skin to determine warmth. *rationale* Hypertension is the major symptom that is associated with pheochromocytoma. The blood pressure status is monitored by taking the client's blood pressure. Glycosuria, weight loss, and diaphoresis are also clinical manifestations of pheochromocytoma, but hypertension is the major symptom.

19. Endocrine disorders may be triggered by all of the following except: a. Stress b. Infection c. Chemicals in the food chain and environment d. Cell phone use

19. D: Endocrine function may be influenced by myriad factors. In addition to the above-mentioned, there is evidence that exposure to naturally occurring and man-made endocrine disruptors such as tributyltin, certain bioaccumulating chlorinated compounds, and phytoestrogens is widespread and in susceptible individuals, may trigger endocrine disorders.

what should you do if pt is hypoglycemic?

1st eat simple sugar (4-60z coke, milk, juice)

While collecting data on a client being prepared for an adrenalectomy, the nurse obtains a temperature reading of 100.8° F. The nurse analyzes this temperature reading as:

1. Within normal limits *2. A finding that needs to be reported immediately* 3. An expected finding caused by the operative stress response 4. Slightly abnormal but an insignificant finding *rationale* An adrenalectomy is performed because of excess adrenal gland function. Excess cortisol production impairs the immune response, which puts the client at risk for infection. Because of this, the client needs to be protected from infection, and minor variations in normal vital sign values must be reported so that infections are detected early, before they become overwhelming. In addition, the surgeon may elect to postpone surgery in the event of a fever because it can be indicative of infection. Options 1, 3, and 4 are not correct interpretations.

How to prevent hypoglycemia?

1. eat 2. take insulin regularly 3. know s/s 4. check B/S as prescribed

10. A nurse provides dietary instructions to a client with a diagnosis of hyperparathyroidism. Which statement by the client indicates the need for further instructions? a) I need to drink 3000 ml of fluid per day b) I should drink cranberry juice daily c) I should eat foods high in calcium d) I should eat foods high in fiber

10) C - The client with hyperparathyroidism should consume at least 3000 mL of fluid per day. Measures to prevent dehydration are necessary because dehydration increases serum calcium levels and promotes the formation of renal stones. Cranberry juice and prune juice help make the urine more acidic. A high urinary acidity helps prevent renal stone formation because calcium is more soluble in acidic urine than in alkaline urine. Clients should be on a low-calcium, low-vitamin D diet. High-fiber foods are important to prevent constipation and fecal impaction resulting from the hypercalcemia that occurs with this disorder.

10. All of the following statements about Hashimoto's disease are true except: a. Many patients are entirely asymptomatic b. Not all patients become hypothyroid c. Most cases of obesity are attributable to Hashimoto's disease d. Hypothyroidism may be subclinical

10. C: Although weight gain may be a symptom of Hashimoto's disease, the majority of obese people have normal thyroid function; rarely is thyroid disorder the sole cause of obesity. Other symptoms of Hashimoto's disease include fatigue, cold intolerance, joint pain, myalgias, constipation, dry hair, skin and nails, impaired fertility, slow heart rate, and depression.

16. A nurse is caring for a client with hyperthyroidism and is instructing the client about dietary measures. The nurse tells the client that it is important to eat foods that are: a) high in bulk and fiber b) low in calories c) low in carbohydrates and fats d) high in calories

16) D - The client with hyperthyroidism is usually extremely hungry because of increased metabolism. The client should be instructed to consume a high-calorie diet with six full meals a day. The client should be instructed to eat foods that are nutritious and contain ample amounts of protein, carbohydrates, fats, and minerals. Clients should be discouraged from eating foods that increase peristalsis and thus result in diarrhea, such as highly seasoned, bulky, and fibrous foods.

2. A nurse is admitting a client with a diagnosis of Addison's disease to the hospital. On assessment, the nurse would expect to note which finding that is a manifestation of this disorder? a) peripheral edema b) excessive facial hair c) lower than normal blood glucose level d) high blood pressure

2) C - Blood glucose levels are low in Addison's disease as a result of decreased secretion of glucocorticoids (cortisol). Edema is absent, and aldosterone secretion is decreased so the client develops a deficient fluid volume. Facial hair increases with adrenocortical hyperfunction. Clients with Addison's disease develop hypotension as a result of deficient fluid volume. Options A, B and D are unrelated to Addison's disease.

20. A husband of a client with graves' disease expresses concern regarding his wife's health because during the past 3 months she has been experiencing nervousness, inability to concentrate even on trivial tasks, and outbursts of temper. On the basis of this information, which nursing diagnosis would the nurse identify as appropriate for the client? a) ineffective coping b) disturbed sensory perception c) social isolation d) grieving

20) A - Frequently, family and friends may report that the client with Graves' disease has become more irritable or depressed. The signs and symptoms in the question are supporting data for the nursing diagnosis of Ineffective coping and are not related to options B, C, and D. The question does not provide data to support options B, C, and D.

21. A nurse is caring for a client with hypoparathyroidism. In planning for discharge from the hospital, the nurse identifies which of the following as a potential psychosocial nursing diagnosis? a) impaired comfort related to cold intolerance secondary to decreased metabolic rate b) constipation related to decreased peristaltic action secondary to decreased metabolic rate c) high risk for impaired skin integrity related to edema d) anxiety related to the need for lifelong dietary interventions to control the disease

21) D - Medical management of hypoparathyroidism is aimed at correcting the hypocalcemia. This is accomplished with prescribed medications as well as lifelong compliance to dietary guidelines, which include consumption of foods high in calcium but low in phosphorus. Knowing that the interventions are lifelong can create some anxiety for the client, and this problem needs to be addressed before hospital discharge. The other options are unrelated to this condition and to a psychosocial concern.

If a pregnant woman is over 25 or has a fam hx of diabetes, must screen the mother at ___ wks gestation?

24

27. A nurse provides instructions to a client who is scheduled for a radioactive iodine uptake test. Which statement by the client indicates a need for further instructions? a) the test measures the rate of iodine uptake by my thyroid gland b) I will need to drink a small dose of radioactive iodine before the test c) a 24 hour urine specimen will need to be collected to measure iodine excretion d) I need to minimize close contact with others in my family for a period of 48 hours after the test because of the radioactivity in my system

27) D - The client undergoing a radioactive iodine uptake test needs to be reassured that the amount of radioactive iodine used is very small, that it is not harmful to the client, and that the client will not be radioactive. The other options are correct regarding this diagnostic test.

29. A nurse is caring for a client with Cushing's syndrome who demonstrates withdrawn behavior. The nurse recognizes that this client's behavior is likely related to which nursing diagnosis? a) deficient diversional activity b) powerlessness c) hopelessness d) disturbed body image

29) D - Physical changes in the client's appearance can occur with Cushing's syndrome. Such changes include hirsutism, moon face, buffalo hump, acne, and striae. These changes cause a body image disturbance. Options A, B, and C are not commonly associated with Cushing's syndrome.

3. A nurse is preparing to perform an assessment on a client being admitted to the hospital with a diagnosis of Cushing's syndrome. When performing the assessment, the nurse checks for which significant manifestation of the disorder? a) fluid retention b) stretch marks c) goiter d) melanosis

3) A - Excessive secretion of adrenocortical hormones results in water and sodium reabsorption, causing fluid retention. Stretch marks (striae) are a common feature and can result in a disturbed body image, but are not significant and do not represent a life-threatening situation. Goiter is not a manifestation of Cushing's syndrome. Melanosis is a common manifestation associated with Addison's disease.

30. A client with Addison's disease makes all of the following statements. Which one does the nurse analyze as requiring further discussion? a) I wear a Medic-Alert bracelet at all times b) I need to weigh myself daily and record it c) It is important that I drink enough fluids and increase my salt intake d) my medication doses will not need to be adjusted for any reason

30) D - The client with Addison's disease is experiencing deficits of mineralocorticoids, glucocorticoids, and androgens. Aldosterone deficiency affects the ability of the nephrons to conserve sodium, so the client experiences sodium and fluid volume deficit. The client needs to manage this problem with daily hormone replacement and increased fluid and sodium intake. Clients are instructed to weigh themselves daily as a means of monitoring fluid volume balance. Glucocorticoids and mineralocorticoids are essential components of the stress response. Additional doses of hormone replacement therapy are needed with any type of physical or psychological stressor. This information needs to be conveyed to the client and requires that the client wear a Medic-Alert bracelet so that health care professionals are aware of this problem if the client were to experience a medical emergency.

4. A clinic nurse is performing an assessment on a client who has hypothyroidism. The nurse would expect to note which clinical manifestation? a) complaints of difficulty sleeping b) complaints of diarrhea c) significant weight loss since the last clinic visit d) complaints of intolerance to cold weather

4) D - An insufficient level of thyroid hormone causes a decrease in metabolic rate and heat production. Intolerance to cold would be noted. Options A, B and C are clinical manifestations of hyperthyroidism.

What is the ideal goal for HbA1c for a diabetic?

4-6% or less

. A client has been diagnosed with goiter. The nurse looks for documentation of which of the following in the client's medical record? a) decreased wound healing b) chronic fatigue c) enlarged thyroid gland d) heart damage

6) C - An enlarged thyroid gland occurs in goiter. Decreased wound healing, chronic fatigue, and heart damage are not specifically associated with this condition.

8. A nurse is caring for a client following thyroidectomy and is monitoring for complications. Which of the following if noted in the client, would indicate a need for physician notification? a) surgical pain in the neck area b) voice hoarseness c) numbness and tingling around the mouth d) weakness of the voice

8) C - Hypocalcemia can develop after thyroidectomy if the parathyroid glands are accidentally removed or traumatized during surgery. If the client develops numbness and tingling around the mouth or in the fingertips or toes, muscle spasms, or twitching, the physician should be called immediately. A hoarse or weak voice may occur temporarily if there has been unilateral injury to the laryngeal nerve during surgery. Pain is expected in the postoperative period. Calcium gluconate ampules should be available at the bedside, and the client should have a patent intravenous (IV) line in the event that hypocalcemic tetany occurs

8. Hashimoto's disease is: a. Chronic inflammation of the thyroid gland b. Diagnosed most frequently in Asian-Americans and Pacific Islanders c. A form of hyperthyroidism d. A rare form of hypothyroidism

8. A: Hashimoto's disease is the most common cause of hypothyroidism. It is an autoimmune disease that produces chronic inflammation of the thyroid gland. More women are affected than men and it is generally diagnosed in persons ages 40 to 60. When treatment is indicated, synthetic T4 is administered.

9. A nurse is monitoring a client for complications following thyroidectomy. The nurse notes that the client's voice is very hoarse, and the client is concerned about the hoarseness and asks the nurse about it. The nurse makes which response to alleviate the client's concern? a) hoarseness and weak voice indicate permanent damage to the nerves b) this complication is expected c) this problem is temporary and will probably subside in a few days d) it is best that you not talk at all until the problem is further evaluated

9) C Temporary hoarseness and a weak voice may occur if there has been unilateral injury to the laryngeal nerve during surgery. If hoarseness or a weak voice is present, the client is reassured that the problem will probably subside in a few days. Unnecessary talking is discouraged to minimize hoarseness. The statements in options A, B, and D will not alleviate the client's concern.

9. Persons at increased risk of developing Hashimoto's disease include all of the following except: a. Persons with vitiligo b. Asian-Americans c. Persons with rheumatoid arthritis d. Persons with Addison's disease

9. B: Along with the above-mentioned groups, persons with type 1 diabetes and persons suffering from pernicious anemia (insufficient vitamin b12) are at increased risk of developing Hashimoto's disease. Because it tends to run in families, there is likely a genetic susceptibility as well. Environmental factors such as excessive iodine consumption and selected drugs also have been implicated as potential risk factors.

A 22-year-old patient is being seen in the clinic with increased secretion of the anterior pituitary hormones. The nurse would expect the laboratory results to show a. increased urinary cortisol. b. decreased serum thyroxine. c. elevated serum aldosterone levels. d. low urinary catecholamines excretion.

A

The nurse is caring for a 63-year-old with a possible pituitary tumor who is scheduled for a computed tomography (CT) scan with contrast. Which information about the patient is most important to discuss with the health care provider before the test? a. History of renal insufficiency b. Complains of chronic headache c. Recent bilateral visual field loss d. Blood glucose level of 134 mg/dL

A

Which of the following would be an indication of Androgen Insensitivity Syndrome? Answers: A. A 33 year old woman with a karyotype of XY B. A 16 year old male with reduced kidney function C. Failure to respond to cortisol therapy D. Several pregnancies all of which ended in miscarriages

A Androgen Insensitivity Syndrome is when the body does not respond to androgens such as testosterone. This can result in genetic males being born with the appearance of women.

3. Symptoms of Grave's ophthalmopathy include all of the following except: a. Bulging eyeballs b. Dry, irritated eyes and puffy eyelids c. Cataracts d. Light sensitivity

A Grave's ophthalmopathy is an inflammation of tissue behind the eye causing the eyeballs to bulge. In addition to the above-mentioned symptoms, Grave's ophthalmopathy may cause pressure or pain in the eyes, double vision, and trouble moving the eyes. About one-quarter of persons with Grave's disease develop Grave's ophthalmopathy. The condition is frequently self-limiting, resolving without treatment over the course of a year or two.

In educating a client, the nurse is likely to explain the following is the cause of Hashimoto's disease: Answers: A. Antibodies attacking the thyroid gland B. Inflammation in the kidneys C. An adenocarcinoma in the brain D. Overactivation of the pituitary gland

A Hashimoto's disease is caused by autoimmunity to the thyroid gland, often involving antibodies.

Which statements will the nurse include when teaching a patient who is scheduled for oral glucose tolerance testing in the outpatient clinic (select all that apply)? a. "You will need to avoid smoking before the test." b. "Exercise should be avoided until the testing is complete." c. "Several blood samples will be obtained during the testing." d. "You should follow a low-calorie diet the day before the test." e. "The test requires that you fast for at least 8 hours before testing."

A C E

Which client is at risk for developing thyrotoxicosis?

A client with Graves' disease who is having surgery

The nurse is caring for a client with Addison's disease. The nurse checks the client's vital signs and determines that the client has orthostatic hypotension. The nurse determines that this finding relates to which factor?

A decreased secretion of aldosterone

While collecting data on a client being prepared for an adrenalectomy, the nurse obtains a temperature reading of 100.8° F. The nurse analyzes this temperature reading as which?

A finding that needs to be reported immediately

Which of the following symptoms are expected in the patient with pheochromocytoma? Select all that Apply (SATA): A) Hypertension B) HA C) Hypoglycemia D) Hypermetabolism E) Dry skin

A, B, and D. The five H's: HA, Hypertension, Hyperhydrosis (sweating), Hypermetabolism and Hyperglycemia.

Which of the following would the nurse monitor for as signs of thyroid storm? SATA: A) T 101.5 B) HR 59 C) Constipation D) Edema E) Chest pain

A, D, and E. Thyroid storm is a form of severe hyperthyroidism usually of abrupt onset. S/S are typically characterized by exaggeration of hyperthyroidism. An extreme fever over 101.3 degrees often occurs along with extreme tachycardia (>130) During thyroid storm increased cardiac compromise may occur as evidenced by edema (HF), chest pain, dyspnea, or palpitations.

A patient with acromegaly is treated with transsphenoidal hypophysectomy. Postoperatively the nurse a. ensures that any clear nasal drainage is tested for glucose b. maintains the patient flat in bed to prevent CSF leakage c. assists the patient with toothbrushing every 4 hours to keep the surgical area clean d. encourages deep breathing, coughing, and turning to prevent respiratory complications

A- A transsphenoidal hypophysectomy involves entry into the sella turcica through an incision in the upper lip and gingiva into the floor of the nose and the sphenoid sinuses. Postoperative clear nasal drainage with glucose content indicates CSF leakage from an open connection to the brain, putting the patient at risk for meningitis. After surgery, the patient is positioned with the head of the bed elevated to avoid pressure on the sella turcica. Coughing and straining are avoided to prevent increased ICP and CSF leakage, and although mouth care is required every 4 hours, toothbrushing should not be performed because injury to the suture line may occur.

Preoperative instructions for the patient scheduled for a subtotal thyroidectomy include teaching the patient a. how to support the head with the hands when moving b. that coughing should be avoided to prevent pressure on the incision c. that the head and neck will need to remain immobile until the incision heals d. that any tingling around the lips or in the fingers after surgery is expected and temporary

A- To prevent strain on the suture line postoperatively, the patient's head must be manually supported while turning and moving in bed, but ROM exercises for the head and neck are also taught preoperatively to be gradually implemented after surgery. There is no contraindication for coughing and deep breathing, and these should be carried out postoperatively. Tingling around the lips or fingers is a sign of hypocalcemia, which may occur if the parathyroid glands are inadvertently removed during surgery. This sign should be reported immediately.

During the care of the patient with SIADH, the nurse should a. monitor neurologic status at least every 2 hours b. keep the head of the bed elevated to prevent ADH release c. teach the patient receiving treatment with diuretics to restrict sodium intake d. notify the health care provider if the patient's BP decreases more than 20 mmHg from baseline

A- the patient with syndrome of inappropriate secretion of antidiuretic hormone (SIADH) has marked dilutional hyponatremia and should be monitored for decreased neurologic function and convulsions every 2 hours. ADH release is reduced by keeping the head of the bed flat to increase left atrial filling pressure, and sodium intake is supplemented because of the hyponatremia and sodium loss caused by diuretics. A reduction in BP indicates a reduction in total fluid volume and is an expected outcome of treatment.

12. Nurse Louie is developing a teaching plan for a male client diagnosed with diabetes insipidus. The nurse should include information about which hormone lacking in clients with diabetes insipidus? a. antidiuretic hormone (ADH). b. thyroid-stimulating hormone (TSH). c. follicle-stimulating hormone (FSH). d. luteinizing hormone (LH).

A. ADH is the hormone clients with diabetes insipidus lack. The client's TSH, FSH, and LH levels won't be affected.

19. Jemma, who weighs 210 lb (95 kg) and has been diagnosed with hyperglycemia tells the nurse that her husband sleeps in another room because her snoring keeps him awake. The nurse notices that she has large hands and a hoarse voice. Which of the following would the nurse suspect as a possible cause of the client's hyperglycemia? a. Acromegaly b. Type 1 diabetes mellitus c. Hypothyroidism d. Deficient growth hormone

A. Acromegaly, which is caused by a pituitary tumor that releases excessive growth hormone, is associated with hyperglycemia, hypertension, diaphoresis, peripheral neuropathy, and joint pain. Enlarged hands and feet are related to lateral bone growth, which is seen in adults with this disorder. The accompanying soft tissue swelling causes hoarseness and often sleep apnea. Type 1 diabetes is usually seen in children, and newly diagnosed persons are usually very ill and thin. Hypothyroidism isn't associated with hyperglycemia, nor is growth hormone deficiency.

8. When caring for a male client with diabetes insipidus, nurse Juliet expects to administer: a. vasopressin (Pitressin Synthetic). b. furosemide (Lasix). c. regular insulin. d. 10% dextrose.

A. Because diabetes insipidus results from decreased antidiuretic hormone (vasopressin) production, the nurse should expect to administer synthetic vasopressin for hormone replacement therapy. Furosemide, a diuretic, is contraindicated because a client with diabetes insipidus experiences polyuria. Insulin and dextrose are used to treat diabetes mellitus and its complications, not diabetes insipidus.

18. Which outcome indicates that treatment of a male client with diabetes insipidus has been effective? a. Fluid intake is less than 2,500 ml/day. b. Urine output measures more than 200 ml/hour. c. Blood pressure is 90/50 mm Hg. d. The heart rate is 126 beats/minute.

A. Diabetes insipidus is characterized by polyuria (up to 8 L/day), constant thirst, and an unusually high oral intake of fluids. Treatment with the appropriate drug should decrease both oral fluid intake and urine output. A urine output of 200 ml/hour indicates continuing polyuria. A blood pressure of 90/50 mm Hg and a heart rate of 126 beats/minute indicate compensation for the continued fluid deficit, suggesting that treatment hasn't been effective.

11. Following a unilateral adrenalectomy, nurse Betty would assess for hyperkalemia shown by which of the following? a. Muscle weakness b. Tremors c. Diaphoresis d. Constipation

A. Muscle weakness, bradycardia, nausea, diarrhea, and paresthesia of the hands, feet, tongue, and face are findings associated with hyperkalemia, which is transient and occurs from transient hypoaldosteronism when the adenoma is removed. Tremors, diaphoresis, and constipation aren't seen in hyperkalemia.

A female adult client with a history of chronic hyperparathyroidism admits to being noncompliant. Based on initial assessment findings, nurse Julia formulates the nursing diagnosis of Risk for injury. To complete the nursing diagnosis statement for this client, which "related-to" phrase should the nurse add? a. Related to bone demineralization resulting in pathologic fractures b. Related to exhaustion secondary to an accelerated metabolic rate c. Related to edema and dry skin secondary to fluid infiltration into the interstitial spaces d. Related to tetany secondary to a decreased serum calcium level

A. Poorly controlled hyperparathyroidism may cause an elevated serum calcium level. This, in turn, may diminish calcium stores in the bone, causing bone demineralization and setting the stage for pathologic fractures and a risk for injury. Hyperparathyroidism doesn't accelerate the metabolic rate. A decreased thyroid hormone level, not an increased parathyroid hormone level, may cause edema and dry skin secondary to fluid infiltration into the interstitial spaces. Hyperparathyroidism causes hypercalcemia, not hypocalcemia; therefore, it isn't associated with tetany.

17. A female client has a serum calcium level of 7.2 mg/dl. During the physical examination, nurse Noah expects to assess: a. Trousseau's sign. b. Homans' sign. c. Hegar's sign. d. Goodell's sign.

A. This client's serum calcium level indicates hypocalcemia, an electrolyte imbalance that causes Trousseau's sign (carpopedal spasm induced by inflating the blood pressure cuff above systolic pressure). Homans' sign (pain on dorsiflexion of the foot) indicates deep vein thrombosis. Hegar's sign (softening of the uterine isthmus) and Goodell's sign (cervical softening) are probable signs of pregnancy.

The nurse performs a physical assessment on a 74-year-old woman with possible endocrine dysfunction. The patient's weight was 142 pounds 6 months ago compared to a current weight of 125 pounds. What percent weight change will the nurse document in the patient's health record? A. 12% weight loss B. 17% weight loss C. 25% weight loss D. 74% weight loss

A. 12% weight loss 142 pounds - 125 pounds = 17 pounds; (17/142) × 100 = 12%. Weight change (%) is calculated by dividing the current body weight change by the usual body weight and multiplying the result by 100. Weight change greater than 5% in 1 month, 7.5% in 3 months, or 10% in 6 months is considered severe.

An 18-year-old male patient with a small stature is scheduled for a growth hormone stimulation test. In preparation for the test, the nurse will obtain a. ice in a basin. b. glargine insulin. c. a cardiac monitor. d. 50% dextrose solution.

D

When the nurse assesses the patient that has pancreatitis, what function may be altered related to the endocrine function of the pancreas? A. Blood glucose regulation B. Increased response to stress C. Fluid and electrolyte regulation D. Regulates metabolic rate of cells

A. Blood glucose regulation The endocrine functions of the pancreas are regulated by α cells that produce and secrete glucagon, β cells that produce and secrete insulin and amylin, delta cells that produce and secrete somatostatin, and F cells that secrete pancreatic polypeptide. Glucagon, insulin, and amylin, and somatostatin all affect blood glucose.

When the nurse assesses the patient that has pancreatitis, what function may be altered related to the endocrine function of the pancreas? A. Blood glucose regulation B. Increased response to stress C. Fluid and electrolyte regulation D. Regulates metabolic rate of cells

A. Blood glucose regulation The endocrine functions of the pancreas are regulated by α cells that produce and secrete glucagon, β cells that produce and secrete insulin and amylin, delta cells that produce and secrete somatostatin, and F cells that secrete pancreatic polypeptide. Glucagon, insulin, and amylin, and somatostatin all affect blood glucose. Pancreatic polypeptide regulates appetite. Increased response to stress occurs from epinephrine secreted by the adrenal medulla. Fluid and electrolyte regulation occurs in response to several hormones (mineralocorticoids, antidiuretic hormone, parathyroid hormone, calcitonin) from several organs (adrenal cortex, posterior pituitary, parathyroid, thyroid). The metabolic rate of cells is regulated by triiodothyronine (T3) from the thyroid.

The parathyroid glands play a major role in regulating which substances? A. Calcium and Phosphorus B. Cholride and potassium C. Potassium and calcium D. Sodium and potassium

A. Calcium and Phosphorus

The surgeon was unable to save a patient's parathyroid gland during a radical thyroidectomy. The nurse should consequently pay particular attention to which laboratory value? A. Calcium levels B. Potassium levels C. Blood glucose levels D. Sodium and chloride levels

A. Calcium levels The parathyroid gland plays a key role in maintaining calcium levels.

The surgeon was unable to save a patient's parathyroid gland during a radical thyroidectomy. The nurse should consequently pay particular attention to which laboratory value? A. Calcium levels B. Potassium levels C. Blood glucose levels D. Sodium and chloride levels

A. Calcium levels The parathyroid gland plays a key role in maintaining calcium levels. Potassium, sodium, glucose, and chloride are not directly influenced by the loss of the parathyroid gland.

The patient has been feeling tired lately and has gained weight; reports thickened, dry skin and increased cold sensitivity even though it is now summer. Which endocrine diagnostic test should be done first? A. Free thyroxine (FT4) B. Serum growth hormone (GH) C. Follicle stimulating hormone (FSH) D. Magnetic resonance imaging (MRI) of the head

A. Free thyroxine (FT4)

The patient has been feeling tired lately and has gained weight; reports thickened, dry skin and increased cold sensitivity even though it is now summer. Which endocrine diagnostic test should be done first? A. Free thyroxine (FT4) B. Serum growth hormone (GH) C. Follicle stimulating hormone (FSH) D. Magnetic resonance imaging (MRI) of the head

A. Free thyroxine (FT4) The manifestations the patient is experiencing could be related to hypothyroidism. Free thyroxine (FT4) is considered a better indicator of thyroid function than total T4 and could be done to evaluate the patient for hypothyroidism. Growth hormone excess could cause thick, leathery, oily skin but does not demonstrate the other manifestations. FSH is manifest with menstrual irregularity and would be useful in distinguishing primary gonadal problems from pituitary insufficiency. MRI is the examination of choice for radiologic evaluation of the pituitary gland and the hypothalamus but would not be the first diagnostic study to further explore the basis of these manifestations.

A patient has sought care because of a loss of 25 lb over the past 6 months, during which the patient claims to have made no significant dietary changes. What potential problem should the nurse assess the patient for? A. Thyroid disorders B. Diabetes insipidus C. Pituitary dysfunction D. Parathyroid dysfunction

A. Thyroid disorders Hyperthyroidism is associated with weight loss.

Which statement by a 50-year-old female patient indicates to the nurse that further assessment of thyroid function may be necessary? a. "I notice my breasts are tender lately." b. "I am so thirsty that I drink all day long." c. "I get up several times at night to urinate." d. "I feel a lump in my throat when I swallow."

D

The nurse is providing preoperative teaching to the patient about to undergo a thyroidectomy. Which statement, if made by the patient, indicates the need for further teaching? A) I need to eat super healthy before the surgery and should limit my calories to a healthy amount B) Before the surgery I should practice raising my elbows and placing both hands behind my neck to provide support C) Well, I will have to quit drinking my coffee and alcohol for now D) I will be sure increase my intake of protein and carbohydrates

A. a high calorie diet should be encouraged preoperatively because of the increased metabolic needs of the body. Practicing supporting the neck before the surgery is a good way to remember what to do after the surgery. The patient should avoid consuming any stimulants including tea, coffee, and cola. The patient should increase intake of protein and carbohydrates preoperatively because of increased metabolic needs.

A patient has sought care because of a loss of 25 lb over the past 6 months, during which the patient claims to have made no significant dietary changes. What potential problem should the nurse assess the patient for? A. Thyroid disorders B. Diabetes insipidus C. Pituitary dysfunction D. Parathyroid dysfunction

A.Thyroid disorders Hyperthyroidism is associated with weight loss. Alterations in pituitary function, such as diabetes insipidus, and parathyroid dysfunction are not commonly associated with this phenomenon.

Which question during the assessment of a diabetic patient will help the nurse identify autonomic neuropathy? a. "Do you feel bloated after eating?" b. "Have you seen any skin changes?" c. "Do you need to increase your insulin dosage when you are stressed?" d. "Have you noticed any painful new ulcerations or sores on your feet?"

ANS: A Autonomic neuropathy can cause delayed gastric emptying, which results in a bloated feeling for the patient. The other questions are also appropriate to ask but would not help in identifying autonomic neuropathy.

In order to assist an older diabetic patient to engage in moderate daily exercise, which action is most important for the nurse to take? a. Determine what type of activities the patient enjoys. b. Remind the patient that exercise will improve self-esteem. c. Teach the patient about the effects of exercise on glucose level. d. Give the patient a list of activities that are moderate in intensity.

ANS: A Because consistency with exercise is important, assessment for the types of exercise that the patient finds enjoyable is the most important action by the nurse in ensuring adherence to an exercise program. The other actions will also be implemented but are not the most important in improving compliance.

Which patient action indicates a good understanding of the nurse's teaching about the use of an insulin pump? a. The patient programs the pump for an insulin bolus after eating. b. The patient changes the location of the insertion site every week. c. The patient takes the pump off at bedtime and starts it again each morning. d. The patient plans for a diet that is less flexible when using the insulin pump

ANS: A In addition to the basal rate of insulin infusion, the patient will adjust the pump to administer a bolus after each meal, with the dosage depending on the oral intake. The insertion site should be changed every 2 or 3 days. There is more flexibility in diet and exercise when an insulin pump is used. The pump will deliver a basal insulin rate 24 hours a day.

A female patient is scheduled for an oral glucose tolerance test. Which information from the patient's health history is most important for the nurse to communicate to the health care provider? a. The patient uses oral contraceptives. b. The patient runs several days a week. c. The patient has been pregnant three times. d. The patient has a family history of diabetes.

ANS: A Oral contraceptive use may falsely elevate oral glucose tolerance test (OGTT) values. Exercise and a family history of diabetes both can affect blood glucose but will not lead to misleading information from the OGTT. History of previous pregnancies may provide informational about gestational glucose tolerance, but will not lead to misleading information from the OGTT.

A 32-year-old patient with diabetes is starting on intensive insulin therapy. Which type of insulin will the nurse discuss using for mealtime coverage? a. Lispro (Humalog) b. Glargine (Lantus) c. Detemir (Levemir) d. NPH (Humulin N)

ANS: A Rapid- or short-acting insulin is used for mealtime coverage for patients receiving intensive insulin therapy. NPH, glargine, or detemir will be used as the basal insulin.

Which statement by the patient indicates a need for additional instruction in administering insulin? a. "I need to rotate injection sites among my arms, legs, and abdomen each day." b. "I can buy the 0.5 mL syringes because the line markings will be easier to see." c. "I should draw up the regular insulin first after injecting air into the NPH bottle." d. "I do not need to aspirate the plunger to check for blood before injecting insulin."

ANS: A Rotating sites is no longer recommended because there is more consistent insulin absorption when the same site is used consistently. The other patient statements are accurate and indicate that no additional instruction is needed.

The nurse has administered 4 oz of orange juice to an alert patient whose blood glucose was 62 mg/dL. Fifteen minutes later, the blood glucose is 67 mg/dL. Which action should the nurse take next? a. Give the patient 4 to 6 oz more orange juice. b. Administer the PRN glucagon (Glucagon) 1 mg IM. c. Have the patient eat some peanut butter with crackers. d. Notify the health care provider about the hypoglycemia.

ANS: A The "rule of 15" indicates that administration of quickly acting carbohydrates should be done 2 to 3 times for a conscious patient whose glucose remains less than 70 mg/dL before notifying the health care provider. More complex carbohydrates and fats may be used once the glucose has stabilized. Glucagon should be used if the patient's level of consciousness decreases so that oral carbohydrates can no longer be given.

A 38-year-old patient who has type 1 diabetes plans to swim laps daily at 1:00 PM. The clinic nurse will plan to teach the patient to a. check glucose level before, during, and after swimming. b. delay eating the noon meal until after the swimming class. c. increase the morning dose of neutral protamine Hagedorn (NPH) insulin. d. time the morning insulin injection so that the peak occurs while swimming.

ANS: A The change in exercise will affect blood glucose, and the patient will need to monitor glucose carefully to determine the need for changes in diet and insulin administration. Because exercise tends to decrease blood glucose, patients are advised to eat before exercising. Increasing the morning NPH or timing the insulin to peak during exercise may lead to hypoglycemia, especially with the increased exercise.

A 55-year-old female patient with type 2 diabetes has a nursing diagnosis of imbalanced nutrition: more than body requirements. Which goal is most important for this patient? a. The patient will reach a glycosylated hemoglobin level of less than 7%. b. The patient will follow a diet and exercise plan that results in weight loss. c. The patient will choose a diet that distributes calories throughout the day. d. The patient will state the reasons for eliminating simple sugars in the diet.

ANS: A The complications of diabetes are related to elevated blood glucose, and the most important patient outcome is the reduction of glucose to near-normal levels. The other outcomes also are appropriate but are not as high in priority.

Which information will the nurse include in teaching a female patient who has peripheral arterial disease, type 2 diabetes, and sensory neuropathy of the feet and legs? a. Choose flat-soled leather shoes. b. Set heating pads on a low temperature. c. Use callus remover for corns or calluses. d. Soak feet in warm water for an hour each day.

ANS: A The patient is taught to avoid high heels and that leather shoes are preferred. The feet should be washed, but not soaked, in warm water daily. Heating pad use should be avoided. Commercial callus and corn removers should be avoided. The patient should see a specialist to treat these problems

A patient receives aspart (NovoLog) insulin at 8:00 AM. Which time will it be most important for the nurse to monitor for symptoms of hypoglycemia? a. 10:00 AM b. 12:00 AM c. 2:00 PM d. 4:00 PM

ANS: A The rapid-acting insulins peak in 1 to 3 hours. The patient is not at a high risk for hypoglycemia at the other listed times, although hypoglycemia may occur.

Which prescribed medication should the nurse administer first to a 60-year-old patient admitted to the emergency department in thyroid storm? a. Propranolol (Inderal) b. Propylthiouracil (PTU) c. Methimazole (Tapazole) d. Iodine (Lugol's solution)

ANS: A Adrenergic blockers work rapidly to decrease the cardiovascular manifestations of thyroid storm. The other medications take days to weeks to have an impact on thyroid function.

Which finding for a patient who has hypothyroidism and hypertension indicates that the nurse should contact the health care provider before administering levothyroxine (Synthroid)? a. Increased thyroxine (T4) level b. Blood pressure 112/62 mm Hg c. Distant and difficult to hear heart sounds d. Elevated thyroid stimulating hormone level

ANS: A An increased thyroxine level indicates the levothyroxine dose needs to be decreased. The other data are consistent with hypothyroidism and the nurse should administer the levothyroxine.

Which finding by the nurse when assessing a patient with Hashimoto's thyroiditis and a goiter will require the most immediate action? a. New-onset changes in the patient's voice b. Apical pulse rate at rest 112 beats/minute c. Elevation in the patient's T3 and T4 levels d. Bruit audible bilaterally over the thyroid gland

ANS: A Changes in the patient's voice indicate that the goiter is compressing the laryngeal nerve and may lead to airway compression. The other findings will also be reported but are expected with Hashimoto's thyroiditis and do not require immediate action.

Which finding indicates to the nurse that the current therapies are effective for a patient with acute adrenal insufficiency? a. Increasing serum sodium levels b. Decreasing blood glucose levels c. Decreasing serum chloride levels d. Increasing serum potassium levels

ANS: A Clinical manifestations of Addison's disease include hyponatremia and an increase in sodium level indicates improvement. The other values indicate that treatment has not been effective.

A 56-year-old female patient has an adrenocortical adenoma, causing hyperaldosteronism. The nurse providing care should a. monitor the blood pressure every 4 hours. b. elevate the patient's legs to relieve edema. c. monitor blood glucose level every 4 hours. d. order the patient a potassium-restricted diet.

ANS: A Hypertension caused by sodium retention is a common complication of hyperaldosteronism. Hyperaldosteronism does not cause an elevation in blood glucose. The patient will be hypokalemic and require potassium supplementation before surgery. Edema does not usually occur with hyperaldosteronism.

The nurse determines that additional instruction is needed for a 60-year-old patient with chronic syndrome of inappropriate antidiuretic hormone (SIADH) when the patient says which of the following? a. "I need to shop for foods low in sodium and avoid adding salt to food." b. "I should weigh myself daily and report any sudden weight loss or gain." c. "I need to limit my fluid intake to no more than 1 quart of liquids a day." d. "I will eat foods high in potassium because diuretics cause potassium loss."

ANS: A Patients with SIADH are at risk for hyponatremia, and a sodium supplement may be prescribed. The other patient statements are correct and indicate successful teaching has occurred.

In explaining the condition to a client, a nurse would say that Cushing's syndrome is caused primarily by: Answers: A. Low levels of glucocorticoids B. Excess secretion of sodium C. Autoimmunity in the pancreas D. Elevated levels of cortisol

D Cushing's syndrome is caused by elevated levels of cortisol. Glucocorticoids tend to cause this.

The nurse is caring for a patient admitted with diabetes insipidus (DI). Which information is most important to report to the health care provider? a. The patient is confused and lethargic. b. The patient reports a recent head injury. c. The patient has a urine output of 400 mL/hr. d. The patient's urine specific gravity is 1.003.

ANS: A The patient's confusion and lethargy may indicate hypernatremia and should be addressed quickly. In addition, patients with DI compensate for fluid losses by drinking copious amounts of fluids, but a patient who is lethargic will be unable to drink enough fluids and will become hypovolemic. A high urine output, low urine specific gravity, and history of a recent head injury are consistent with diabetes insipidus, but they do not require immediate nursing action to avoid life-threatening complications.

A patient who was admitted with myxedema coma and diagnosed with hypothyroidism is improving and expected to be discharged in 2 days. Which teaching strategy will be best for the nurse to use? a. Provide written reminders of self-care information. b. Offer multiple options for management of therapies. c. Ensure privacy for teaching by asking visitors to leave. d. Delay teaching until patient discharge date is confirmed.

ANS: A Written instructions will be helpful to the patient because initially the hypothyroid patient may be unable to remember to take medications and other aspects of self-care. Because the treatment regimen is somewhat complex, teaching should be initiated well before discharge. Family members or friends should be included in teaching because the hypothyroid patient is likely to forget some aspects of the treatment plan. A simpler regimen will be easier to understand until the patient is euthyroid.

In which order will the nurse take these steps to prepare NPH 20 units and regular insulin 2 units using the same syringe? (Put a comma and a space between each answer choice [A, B, C, D, E]). a. Rotate NPH vial. b. Withdraw regular insulin. c. Withdraw 20 units of NPH. d. Inject 20 units of air into NPH vial. e. Inject 2 units of air into regular insulin vial.

ANS: A, D, E, B, C When mixing regular insulin with NPH, it is important to avoid contact between the regular insulin and the additives in the NPH that slow the onset, peak, and duration of activity in the longer-acting insulin.

A 34-year-old has a new diagnosis of type 2 diabetes. The nurse will discuss the need to schedule a dilated eye exam a. every 2 years. b. as soon as possible. c. when the patient is 39 years old. d. within the first year after diagnosis.

ANS: B Because many patients have some diabetic retinopathy when they are first diagnosed with type 2 diabetes, a dilated eye exam is recommended at the time of diagnosis and annually thereafter. Patients with type 1 diabetes should have dilated eye exams starting 5 years after they are diagnosed and then annually.

After change-of-shift report, which patient should the nurse assess first? a. 19-year-old with type 1 diabetes who has a hemoglobin A1C of 12% b. 23-year-old with type 1 diabetes who has a blood glucose of 40 mg/dL c. 40-year-old who is pregnant and whose oral glucose tolerance test is 202 mg/dL d. 50-year-old who uses exenatide (Byetta) and is complaining of acute abdominal pain

ANS: B Because the brain requires glucose to function, untreated hypoglycemia can cause unconsciousness, seizures, and death. The nurse will rapidly assess and treat the patient with low blood glucose. The other patients also have symptoms that require assessments and/or interventions, but they are not at immediate risk for life-threatening complications.

The nurse is preparing to teach a 43-year-old man who is newly diagnosed with type 2 diabetes about home management of the disease. Which action should the nurse take first? a. Ask the patient's family to participate in the diabetes education program. b. Assess the patient's perception of what it means to have diabetes mellitus. c. Demonstrate how to check glucose using capillary blood glucose monitoring. d. Discuss the need for the patient to actively participate in diabetes management.

ANS: B Before planning teaching, the nurse should assess the patient's interest in and ability to self-manage the diabetes. After assessing the patient, the other nursing actions may be appropriate, but planning needs to be individualized to each patient.

Which patient action indicates good understanding of the nurse's teaching about administration of aspart (NovoLog) insulin? a. The patient avoids injecting the insulin into the upper abdominal area. b. The patient cleans the skin with soap and water before insulin administration. c. The patient stores the insulin in the freezer after administering the prescribed dose. d. The patient pushes the plunger down while removing the syringe from the injection site.

ANS: B Cleaning the skin with soap and water or with alcohol is acceptable. Insulin should not be frozen. The patient should leave the syringe in place for about 5 seconds after injection to be sure that all the insulin has been injected. The upper abdominal area is one of the preferred areas for insulin injection.

An unresponsive patient with type 2 diabetes is brought to the emergency department and diagnosed with hyperosmolar hyperglycemic syndrome (HHS). The nurse will anticipate the need to a. give a bolus of 50% dextrose. b. insert a large-bore IV catheter. c. initiate oxygen by nasal cannula. d. administer glargine (Lantus) insulin.

ANS: B HHS is initially treated with large volumes of IV fluids to correct hypovolemia. Regular insulin is administered, not a long-acting insulin. There is no indication that the patient requires oxygen. Dextrose solutions will increase the patient's blood glucose and would be contraindicated.

Which action should the nurse take after a 36-year-old patient treated with intramuscular glucagon for hypoglycemia regains consciousness? a. Assess the patient for symptoms of hyperglycemia. b. Give the patient a snack of peanut butter and crackers. c. Have the patient drink a glass of orange juice or nonfat milk. d. Administer a continuous infusion of 5% dextrose for 24 hours.

ANS: B Rebound hypoglycemia can occur after glucagon administration, but having a meal containing complex carbohydrates plus protein and fat will help prevent hypoglycemia. Orange juice and nonfat milk will elevate blood glucose rapidly, but the cheese and crackers will stabilize blood glucose. Administration of IV glucose might be used in patients who were unable to take in nutrition orally. The patient should be assessed for symptoms of hypoglycemia after glucagon administration.

A client is admitted for removal of a goiter. Which nursing intervention should receive priority during the post-operative period? A. Maintaining fluid and electrolyte balance B. Assessing the client's airway C. Providing needed nutrition and fluids D. Providing pain relief with narcotic analgesics

Answer B is correct. A goiter is hyperplasia of the thyroid gland. Removal of a goiter can result in laryngeal spasms and airway occlusion. The other answers are lesser in priority.

Which laboratory value reported to the nurse by the unlicensed assistive personnel (UAP) indicates the most urgent need for the nurse's assessment of the patient? a. Bedtime glucose of 140 mg/dL b. Noon blood glucose of 52 mg/dL c. Fasting blood glucose of 130 mg/dL d. 2-hr postprandial glucose of 220 mg/dL

ANS: B The nurse should assess the patient with a blood glucose level of 52 mg/dL for symptoms of hypoglycemia and give the patient a carbohydrate-containing beverage such as orange juice. The other values are within an acceptable range or not immediately dangerous for a diabetic patient.

Which information will the nurse include when teaching a 50-year-old patient who has type 2 diabetes about glyburide (Micronase, DiaBeta, Glynase)? a. Glyburide decreases glucagon secretion from the pancreas. b. Glyburide stimulates insulin production and release from the pancreas. c. Glyburide should be taken even if the morning blood glucose level is low. d. Glyburide should not be used for 48 hours after receiving IV contrast media.

ANS: B The sulfonylureas stimulate the production and release of insulin from the pancreas. If the glucose level is low, the patient should contact the health care provider before taking the glyburide, because hypoglycemia can occur with this class of medication. Metformin should be held for 48 hours after administration of IV contrast media, but this is not necessary for glyburide. Glucagon secretion is not affected by glyburide.

An active 28-year-old male with type 1 diabetes is being seen in the endocrine clinic. Which finding may indicate the need for a change in therapy? a. Hemoglobin A1C level 6.2% b. Blood pressure 146/88 mmHg c. Heart rate at rest 58 beats/minute d. High density lipoprotein (HDL) level 65 mg/dL

ANS: B To decrease the incidence of macrovascular and microvascular problems in patients with diabetes, the goal blood pressure is usually 130/80. An A1C less than 6.5%, a low resting heart rate (consistent with regular aerobic exercise in a young adult), and an HDL level of 65 mg/dL all indicate that the patient's diabetes and risk factors for vascular disease are well controlled.

A diabetic patient who has reported burning foot pain at night receives a new prescription. Which information should the nurse teach the patient about amitriptyline (Elavil)? a. Amitriptyline decreases the depression caused by your foot pain. b. Amitriptyline helps prevent transmission of pain impulses to the brain. c. Amitriptyline corrects some of the blood vessel changes that cause pain. d. Amitriptyline improves sleep and makes you less aware of nighttime pain.

ANS: B Tricyclic antidepressants decrease the transmission of pain impulses to the spinal cord and brain. Tricyclic antidepressants also improve sleep quality and are used for depression, but that is not the major purpose for their use in diabetic neuropathy. The blood vessel changes that contribute to neuropathy are not affected by tricyclic antidepressants.

A 45-year-old male patient with suspected acromegaly is seen at the clinic. To assist in making the diagnosis, which question should the nurse ask? a. "Have you had a recent head injury?" b. "Do you have to wear larger shoes now?" c. "Is there a family history of acromegaly?" d. "Are you experiencing tremors or anxiety?"

ANS: B Acromegaly causes an enlargement of the hands and feet. Head injury and family history are not risk factors for acromegaly. Tremors and anxiety are not clinical manifestations of acromegaly.

The nurse is planning postoperative care for a patient who is being admitted to the surgical unit form the recovery room after transsphenoidal resection of a pituitary tumor. Which nursing action should be included? a. Palpate extremities for edema. b. Measure urine volume every hour. c. Check hematocrit every 2 hours for 8 hours. d. Monitor continuous pulse oximetry for 24 hours.

ANS: B After pituitary surgery, the patient is at risk for diabetes insipidus caused by cerebral edema. Monitoring of urine output and urine specific gravity is essential. Hemorrhage is not a common problem. There is no need to check the hematocrit hourly. The patient is at risk for dehydration, not volume overload. The patient is not at high risk for problems with oxygenation, and continuous pulse oximetry is not needed.

The nurse is assessing a 41-year-old African American male patient diagnosed with a pituitary tumor causing panhypopituitarism. Assessment findings consistent with panhypopituitarism include a. high blood pressure. b. decreased facial hair. c. elevated blood glucose. d. tachycardia and cardiac palpitations.

ANS: B Changes in male secondary sex characteristics such as decreased facial hair, testicular atrophy, diminished spermatogenesis, loss of libido, impotence, and decreased muscle mass are associated with decreases in follicle stimulating hormone (FSH) and luteinizing hormone (LH). Fasting hypoglycemia and hypotension occur in panhypopituitarism as a result of decreases in adrenocorticotropic hormone (ACTH) and cortisol. Bradycardia is likely due to the decrease in thyroid stimulating hormone (TSH) and thyroid hormones associated with panhypopituitarism.

The nurse determines that demeclocycline (Declomycin) is effective for a patient with syndrome of inappropriate antidiuretic hormone (SIADH) based on finding that the patient's a. weight has increased. b. urinary output is increased. c. peripheral edema is decreased. d. urine specific gravity is increased.

ANS: B Demeclocycline blocks the action of antidiuretic hormone (ADH) on the renal tubules and increases urine output. An increase in weight or an increase in urine specific gravity indicates that the SIADH is not corrected. Peripheral edema does not occur with SIADH. A sudden weight gain without edema is a common clinical manifestation of this disorder.

The cardiac telemetry unit charge nurse receives status reports from other nursing units about four patients who need cardiac monitoring. Which patient should be transferred to the cardiac unit first? a. Patient with Hashimoto's thyroiditis and a heart rate of 102 b. Patient with tetany who has a new order for IV calcium chloride c. Patient with Cushing syndrome and a blood glucose of 140 mg/dL d. Patient with Addison's disease who takes hydrocortisone twice daily

ANS: B Emergency treatment of tetany requires IV administration of calcium; ECG monitoring will be required because cardiac arrest may occur if high calcium levels result from too-rapid administration. The information about the other patients indicates that they are more stable than the patient with tetany.

Which nursing assessment of a 69-year-old patient is most important to make during initiation of thyroid replacement with levothyroxine (Synthroid)? a. Fluid balance b. Apical pulse rate c. Nutritional intake d. Orientation and alertness

ANS: B In older patients, initiation of levothyroxine therapy can increase myocardial oxygen demand and cause angina or dysrhythmias. The medication also is expected to improve mental status and fluid balance and will increase metabolic rate and nutritional needs, but these changes will not result in potentially life-threatening complications.

After receiving change-of-shift report about the following four patients, which patient should the nurse assess first? a. A 31-year-old female with Cushing syndrome and a blood glucose level of 244 mg/dL b. A 70-year-old female taking levothyroxine (Synthroid) who has an irregular pulse of 134 c. A 53-year-old male who has Addison's disease and is due for a scheduled dose of hydrocortisone (Solu-Cortef). d. A 22-year-old male admitted with syndrome of inappropriate antidiuretic hormone (SIADH) who has a serum sodium level of 130 mEq/L

ANS: B Initiation of thyroid replacement in older adults may cause angina and cardiac dysrhythmias. The patient's high pulse rate needs rapid investigation by the nurse to assess for and intervene with any cardiac problems. The other patients also require nursing assessment and/or actions but are not at risk for life-threatening complications.

Which question will the nurse in the endocrine clinic ask to help determine a patient's risk factors for goiter? a. "How much milk do you drink?" b. "What medications are you taking?" c. "Are your immunizations up to date?" d. "Have you had any recent neck injuries?"

ANS: B Medications that contain thyroid-inhibiting substances can cause goiter. Milk intake, neck injury, and immunization history are not risk factors for goiter.

A patient who had radical neck surgery to remove a malignant tumor developed hypoparathyroidism. The nurse should plan to teach the patient about a. bisphosphonates to reduce bone demineralization. b. calcium supplements to normalize serum calcium levels. c. increasing fluid intake to decrease risk for nephrolithiasis. d. including whole grains in the diet to prevent constipation.

ANS: B Oral calcium supplements are used to maintain the serum calcium in normal range and prevent the complications of hypocalcemia. Whole grain foods decrease calcium absorption and will not be recommended. Bisphosphonates will lower serum calcium levels further by preventing calcium from being reabsorbed from bone. Kidney stones are not a complication of hypoparathyroidism and low calcium levels.

Which information will the nurse include when teaching a 50-year-old male patient about somatropin (Genotropin)? a. The medication will be needed for 3 to 6 months. b. Inject the medication subcutaneously every day. c. Blood glucose levels may decrease when taking the medication. d. Stop taking the medication if swelling of the hands or feet occurs.

ANS: B Somatropin is injected subcutaneously on a daily basis, preferably in the evening. The patient will need to continue on somatropin for life. If swelling or other common adverse effects occur, the health care provider should be notified. Growth hormone will increase blood glucose levels.

Which intervention will the nurse include in the plan of care for a 52-year-old male patient with syndrome of inappropriate antidiuretic hormone (SIADH)? a. Monitor for peripheral edema. b. Offer patient hard candies to suck on. c. Encourage fluids to 2 to 3 liters per day. d. Keep head of bed elevated to 30 degrees.

ANS: B Sucking on hard candies decreases thirst for a patient on fluid restriction. Patients with SIADH are on fluid restrictions of 800 to 1000 mL/day. Peripheral edema is not seen with SIADH. The head of the bed is elevated no more than 10 degrees to increase left atrial filling pressure and decrease antidiuretic hormone (ADH) release.

The nurse will plan to monitor a patient diagnosed with a pheochromocytoma for a. flushing. b. headache. c. bradycardia. d. hypoglycemia.

ANS: B The classic clinical manifestations of pheochromocytoma are hypertension, tachycardia, severe headache, diaphoresis, and abdominal or chest pain. Elevated blood glucose may also occur because of sympathetic nervous system stimulation. Bradycardia and flushing would not be expected.

A 37-year-old patient has just arrived in the postanesthesia recovery unit (PACU) after a thyroidectomy. Which information is most important to communicate to the surgeon? a. The patient reports 7/10 incisional pain. b. The patient has increasing neck swelling. c. The patient is sleepy and difficult to arouse. d. The patient's cardiac rate is 112 beats/minute.

ANS: B The neck swelling may lead to respiratory difficulty, and rapid intervention is needed to prevent airway obstruction. The incisional pain should be treated but is not unusual after surgery. A heart rate of 112 is not unusual in a patient who has been hyperthyroid and has just arrived in the PACU from surgery. Sleepiness in the immediate postoperative period is expected.

A 38-year-old male patient is admitted to the hospital in Addisonian crisis. Which patient statement supports a nursing diagnosis of ineffective self-health management related to lack of knowledge about management of Addison's disease? a. "I frequently eat at restaurants, and my food has a lot of added salt." b. "I had the stomach flu earlier this week, so I couldn't take the hydrocortisone." c. "I always double my dose of hydrocortisone on the days that I go for a long run." d. "I take twice as much hydrocortisone in the morning dose as I do in the afternoon."

ANS: B The need for hydrocortisone replacement is increased with stressors such as illness, and the patient needs to be taught to call the health care provider because medication and IV fluids and electrolytes may need to be given. The other patient statements indicate appropriate management of the Addison's disease.

Which nursing action will be included in the plan of care for a 55-year-old patient with Graves' disease who has exophthalmos? a. Place cold packs on the eyes to relieve pain and swelling. b. Elevate the head of the patient's bed to reduce periorbital fluid. c. Apply alternating eye patches to protect the corneas from irritation. d. Teach the patient to blink every few seconds to lubricate the corneas.

ANS: B The patient should sit upright as much as possible to promote fluid drainage from the periorbital area. With exophthalmos, the patient is unable to close the eyes completely to blink. Lubrication of the eyes, rather than eye patches, will protect the eyes from developing corneal scarring. The swelling of the eye is not caused by excessive blood flow to the eye, so cold packs will not be helpful.

A 63-year-old patient with primary hyperparathyroidism has a serum phosphorus level of 1.7 mg/dL (0.55 mmol/L) and calcium of 14 mg/dL (3.5 mmol/L). Which nursing action should be included in the plan of care? a. Restrict the patient to bed rest. b. Encourage 4000 mL of fluids daily. c. Institute routine seizure precautions. d. Assess for positive Chvostek's sign.

ANS: B The patient with hypercalcemia is at risk for kidney stones, which may be prevented by a high fluid intake. Seizure precautions and monitoring for Chvostek's or Trousseau's sign are appropriate for hypocalcemic patients. The patient should engage in weight-bearing exercise to decrease calcium loss from bone.

A patient who had a subtotal thyroidectomy earlier today develops laryngeal stridor and a cramp in the right hand upon returning to the surgical nursing unit. Which collaborative action will the nurse anticipate next? a. Suction the patient's airway. b. Administer IV calcium gluconate. c. Plan for emergency tracheostomy. d. Prepare for endotracheal intubation.

ANS: B The patient's clinical manifestations of stridor and cramping are consistent with tetany caused by hypocalcemia resulting from damage to the parathyroid glands during surgery. Endotracheal intubation or tracheostomy may be needed if the calcium does not resolve the stridor. Suctioning will not correct the stridor.

A 56-year-old patient who is disoriented and reports a headache and muscle cramps is hospitalized with possible syndrome of inappropriate antidiuretic hormone (SIADH). The nurse would expect the initial laboratory results to include a(n) a. elevated hematocrit. b. decreased serum sodium. c. low urine specific gravity. d. increased serum chloride.

ANS: B When water is retained, the serum sodium level will drop below normal, causing the clinical manifestations reported by the patient. The hematocrit will decrease because of the dilution caused by water retention. Urine will be more concentrated with a higher specific gravity. The serum chloride level will usually decrease along with the sodium level.

To monitor for complications in a patient with type 2 diabetes, which tests will the nurse in the diabetic clinic schedule at least annually (select all that apply)? a. Chest x-ray b. Blood pressure c. Serum creatinine d. Urine for microalbuminuria e. Complete blood count (CBC) f. Monofilament testing of the foot

ANS: B, C, D, F Blood pressure, serum creatinine, urine testing for microalbuminuria, and monofilament testing of the foot are recommended at least annually to screen for possible microvascular and macrovascular complications of diabetes. Chest x-ray and CBC might be ordered if the diabetic patient presents with symptoms of respiratory or infectious problems but are not routinely included in screening.

Which statement by a nurse to a patient newly diagnosed with type 2 diabetes is correct? a. Insulin is not used to control blood glucose in patients with type 2 diabetes. b. Complications of type 2 diabetes are less serious than those of type 1 diabetes. c. Changes in diet and exercise may control blood glucose levels in type 2 diabetes. d. Type 2 diabetes is usually diagnosed when the patient is admitted with a hyperglycemic coma.

ANS: C For some patients with type 2 diabetes, changes in lifestyle are sufficient to achieve blood glucose control. Insulin is frequently used for type 2 diabetes, complications are equally severe as for type 1 diabetes, and type 2 diabetes is usually diagnosed with routine laboratory testing or after a patient develops complications such as frequent yeast infections.

When a patient who takes metformin (Glucophage) to manage type 2 diabetes develops an allergic rash from an unknown cause, the health care provider prescribes prednisone (Deltasone). The nurse will anticipate that the patient may a. need a diet higher in calories while receiving prednisone. b. develop acute hypoglycemia while taking the prednisone. c. require administration of insulin while taking prednisone. d. have rashes caused by metformin-prednisone interactions.

ANS: C Glucose levels increase when patients are taking corticosteroids, and insulin may be required to control blood glucose. Hypoglycemia is not a side effect of prednisone. Rashes are not an adverse effect caused by taking metformin and prednisone simultaneously. The patient may have an increased appetite when taking prednisone, but will not need a diet that is higher in calories.

The health care provider suspects the Somogyi effect in a 50-year-old patient whose 6:00 AM blood glucose is 230 mg/dL. Which action will the nurse teach the patient to take? a. Avoid snacking at bedtime. b. Increase the rapid-acting insulin dose. c. Check the blood glucose during the night d. Administer a larger dose of long-acting insulin.

ANS: C If the Somogyi effect is causing the patient's increased morning glucose level, the patient will experience hypoglycemia between 2:00 and 4:00 AM. The dose of insulin will be reduced, rather than increased. A bedtime snack is used to prevent hypoglycemic episodes during the night.

A 26-year-old female with type 1 diabetes develops a sore throat and runny nose after caring for her sick toddler. The patient calls the clinic for advice about her symptoms and a blood glucose level of 210 mg/dL despite taking her usual glargine (Lantus) and lispro (Humalog) insulin. The nurse advises the patient to a. use only the lispro insulin until the symptoms are resolved. b. limit intake of calories until the glucose is less than 120 mg/dL. c. monitor blood glucose every 4 hours and notify the clinic if it continues to rise. d. decrease intake of carbohydrates until glycosylated hemoglobin is less than 7%.

ANS: C Infection and other stressors increase blood glucose levels and the patient will need to test blood glucose frequently, treat elevations appropriately with lispro insulin, and call the health care provider if glucose levels continue to be elevated. Discontinuing the glargine will contribute to hyperglycemia and may lead to diabetic ketoacidosis (DKA). Decreasing carbohydrate or caloric intake is not appropriate because the patient will need more calories when ill. Glycosylated hemoglobin testing is not used to evaluate short-term alterations in blood glucose.

The nurse determines a need for additional instruction when the patient with newly diagnosed type 1 diabetes says which of the following? a. "I can have an occasional alcoholic drink if I include it in my meal plan." b. "I will need a bedtime snack because I take an evening dose of NPH insulin." c. "I can choose any foods, as long as I use enough insulin to cover the calories." d. "I will eat something at meal times to prevent hypoglycemia, even if I am not hungry."

ANS: C Most patients with type 1 diabetes need to plan diet choices very carefully. Patients who are using intensified insulin therapy have considerable flexibility in diet choices but still should restrict dietary intake of items such as fat, protein, and alcohol. The other patient statements are correct and indicate good understanding of the diet instruction.

A 26-year-old patient with diabetes rides a bicycle to and from work every day. Which site should the nurse teach the patient to administer the morning insulin? a. thigh. b. buttock. c. abdomen. d. upper arm.

ANS: C Patients should be taught not to administer insulin into a site that will be exercised because exercise will increase the rate of absorption. The thigh, buttock, and arm are all exercised by riding a bicycle.

A client with Addison's disease will most likely exhibit which symptom? A. Hypertension B. Bronze pigmentation C. Hirsutism D. Purple striae

Answer B is correct. Answer B is correct because a bronze pigmentation is a sign of Addison's disease. Answers A, C, and D are symptoms of Cushing's syndrome, making them incorrect.

A few weeks after an 82-year-old with a new diagnosis of type 2 diabetes has been placed on metformin (Glucophage) therapy and taught about appropriate diet and exercise, the home health nurse makes a visit. Which finding by the nurse is most important to discuss with the health care provider? a. Hemoglobin A1C level is 7.9%. b. Last eye exam was 18 months ago. c. Glomerular filtration rate is decreased. d. Patient has questions about the prescribed diet.

ANS: C The decrease in renal function may indicate a need to adjust the dose of metformin or change to a different medication. In older patients, the goal for A1C may be higher in order to avoid complications associated with hypoglycemia. The nurse will plan on scheduling the patient for an eye exam and addressing the questions about diet, but the biggest concern is the patient's decreased renal function.

A patient who was admitted with diabetic ketoacidosis secondary to a urinary tract infection has been weaned off an insulin drip 30 minutes ago. The patient reports feeling lightheaded and sweaty. Which action should the nurse take first? a. Infuse dextrose 50% by slow IV push. b. Administer 1 mg glucagon subcutaneously. c. Obtain a glucose reading using a finger stick. d. Have the patient drink 4 ounces of orange juice.

ANS: C The patient's clinical manifestations are consistent with hypoglycemia and the initial action should be to check the patient's glucose with a finger stick or order a stat blood glucose. If the glucose is low, the patient should ingest a rapid-acting carbohydrate, such as orange juice. Glucagon or dextrose 50% might be given if the patient's symptoms become worse or if the patient is unconscious.

After change-of-shift report, which patient will the nurse assess first? a. 19-year-old with type 1 diabetes who was admitted with possible dawn phenomenon b. 35-year-old with type 1 diabetes whose most recent blood glucose reading was 230 mg/dL c. 60-year-old with hyperosmolar hyperglycemic syndrome who has poor skin turgor and dry oral mucosa d. 68-year-old with type 2 diabetes who has severe peripheral neuropathy and complains of burning foot pain

ANS: C The patient's diagnosis of HHS and signs of dehydration indicate that the nurse should rapidly assess for signs of shock and determine whether increased fluid infusion is needed. The other patients also need assessment and intervention but do not have life-threatening complications.

A 48-year-old male patient screened for diabetes at a clinic has a fasting plasma glucose level of 120 mg/dL (6.7 mmol/L). The nurse will plan to teach the patient about a. self-monitoring of blood glucose. b. using low doses of regular insulin. c. lifestyle changes to lower blood glucose. d. effects of oral hypoglycemic medications.

ANS: C The patient's impaired fasting glucose indicates prediabetes, and the patient should be counseled about lifestyle changes to prevent the development of type 2 diabetes. The patient with prediabetes does not require insulin or oral hypoglycemics for glucose control and does not need to self-monitor blood glucose.

Which information is most important for the nurse to report to the health care provider before a patient with type 2 diabetes is prepared for a coronary angiogram? a. The patient's most recent HbA1C was 6.5%. b. The patient's admission blood glucose is 128 mg/dL. c. The patient took the prescribed metformin (Glucophage) today. d. The patient took the prescribed captopril (Capoten) this morning.

ANS: C To avoid lactic acidosis, metformin should be discontinued a day or 2 before the coronary arteriogram and should not be used for 48 hours after IV contrast media are administered. The other patient data will also be reported but do not indicate any need to reschedule the procedure.

The nurse is assessing a 22-year-old patient experiencing the onset of symptoms of type 1 diabetes. Which question is most appropriate for the nurse to ask? a. "Are you anorexic?" b. "Is your urine dark colored?" c. "Have you lost weight lately?" d. "Do you crave sugary drinks?"

ANS: C Weight loss occurs because the body is no longer able to absorb glucose and starts to break down protein and fat for energy. The patient is thirsty but does not necessarily crave sugar-containing fluids. Increased appetite is a classic symptom of type 1 diabetes. With the classic symptom of polyuria, urine will be very dilute.

A 29-year-old woman with systemic lupus erythematosus has been prescribed 2 weeks of high-dose prednisone therapy. Which information about the prednisone is most important for the nurse to include? a. "Weigh yourself daily to monitor for weight gain caused by increased appetite." b. "A weight-bearing exercise program will help minimize the risk for osteoporosis." c. "The prednisone dose should be decreased gradually rather than stopped suddenly." d. "Call the health care provider if you experience mood alterations with the prednisone."

ANS: C Acute adrenal insufficiency may occur if exogenous corticosteroids are suddenly stopped. Mood alterations and weight gain are possible adverse effects of corticosteroid use, but these are not life-threatening effects. Osteoporosis occurs when patients take corticosteroids for longer periods.

The nurse is caring for a patient following an adrenalectomy. The highest priority in the immediate postoperative period is to a. protect the patient's skin. b. monitor for signs of infection. c. balance fluids and electrolytes. d. prevent emotional disturbances.

ANS: C After adrenalectomy, the patient is at risk for circulatory instability caused by fluctuating hormone levels, and the focus of care is to assess and maintain fluid and electrolyte status through the use of IV fluids and corticosteroids. The other goals are also important for the patient but are not as immediately life threatening as the circulatory collapse that can occur with fluid and electrolyte disturbances.

A patient has just arrived on the unit after a thyroidectomy. Which action should the nurse take first? a. Observe the dressing for bleeding. b. Check the blood pressure and pulse. c. Assess the patient's respiratory effort. d. Support the patient's head with pillows.

ANS: C Airway obstruction is a possible complication after thyroidectomy because of swelling or bleeding at the site or tetany. The priority nursing action is to assess the airway. The other actions are also part of the standard nursing care postthyroidectomy but are not as high of a priority.

After a 22-year-old female patient with a pituitary adenoma has had a hypophysectomy, the nurse will teach about the need for a. sodium restriction to prevent fluid retention. b. insulin to maintain normal blood glucose levels. c. oral corticosteroids to replace endogenous cortisol. d. chemotherapy to prevent malignant tumor recurrence.

ANS: C Antidiuretic hormone (ADH), cortisol, and thyroid hormone replacement will be needed for life after hypophysectomy. Without the effects of adrenocorticotropic hormone (ACTH) and cortisol, the blood glucose and serum sodium will be low unless cortisol is replaced. An adenoma is a benign tumor, and chemotherapy will not be needed.

Which information will the nurse teach a 48-year-old patient who has been newly diagnosed with Graves' disease? a. Exercise is contraindicated to avoid increasing metabolic rate. b. Restriction of iodine intake is needed to reduce thyroid activity. c. Antithyroid medications may take several months for full effect. d. Surgery will eventually be required to remove the thyroid gland.

ANS: C Medications used to block the synthesis of thyroid hormones may take 2 to 3 months before the full effect is seen. Large doses of iodine are used to inhibit the synthesis of thyroid hormones. Exercise using large muscle groups is encouraged to decrease the irritability and hyperactivity associated with high levels of thyroid hormones. Radioactive iodine is the most common treatment for Graves' disease although surgery may be used.

An expected nursing diagnosis for a 30-year-old patient admitted to the hospital with symptoms of diabetes insipidus is a. excess fluid volume related to intake greater than output. b. impaired gas exchange related to fluid retention in lungs. c. sleep pattern disturbance related to frequent waking to void. d. risk for impaired skin integrity related to generalized edema.

ANS: C Nocturia occurs as a result of the polyuria caused by diabetes insipidus. Edema, excess fluid volume, and fluid retention are not expected.

A 37-year-old patient is being admitted with a diagnosis of Cushing syndrome. Which findings will the nurse expect during the assessment? a. Chronically low blood pressure b. Bronzed appearance of the skin c. Purplish streaks on the abdomen d. Decreased axillary and pubic hair

ANS: C Purplish-red striae on the abdomen are a common clinical manifestation of Cushing syndrome. Hypotension and bronzed-appearing skin are manifestations of Addison's disease. Decreased axillary and pubic hair occur with androgen deficiency.

Which information obtained by the nurse in the endocrine clinic about a patient who has been taking prednisone (Deltasone) 40 mg daily for 3 weeks is most important to report to the health care provider? a. Patient's blood pressure is 148/94 mm Hg. b. Patient has bilateral 2+ pitting ankle edema. c. Patient stopped taking the medication 2 days ago. d. Patient has not been taking the prescribed vitamin D.

ANS: C Sudden cessation of corticosteroids after taking the medication for a week or more can lead to adrenal insufficiency, with problems such as severe hypotension and hypoglycemia. The patient will need immediate evaluation by the health care provider to prevent and/or treat adrenal insufficiency. The other information will also be reported, but does not require rapid treatment.

Which assessment finding of a 42-year-old patient who had a bilateral adrenalectomy requires the most rapid action by the nurse? a. The blood glucose is 176 mg/dL. b. The lungs have bibasilar crackles. c. The blood pressure (BP) is 88/50 mm Hg. d. The patient reports 5/10 incisional pain.

ANS: C The decreased BP indicates possible adrenal insufficiency. The nurse should immediately notify the health care provider so that corticosteroid medications can be administered. The nurse should also address the elevated glucose, incisional pain, and crackles with appropriate collaborative or nursing actions, but prevention and treatment of acute adrenal insufficiency is the priority after adrenalectomy.

A patient develops carpopedal spasms and tingling of the lips following a parathyroidectomy. Which action should the nurse take first? a. Administer the ordered muscle relaxant. b. Give the ordered oral calcium supplement. c. Have the patient rebreathe from a paper bag. d. Start the PRN oxygen at 2 L/min per cannula.

ANS: C The patient's symptoms suggest mild hypocalcemia. The symptoms of hypocalcemia will be temporarily reduced by having the patient breathe into a paper bag, which will raise the PaCO2 and create a more acidic pH. The muscle relaxant will have no impact on the ionized calcium level. Although severe hypocalcemia can cause laryngeal stridor, there is no indication that this patient is experiencing laryngeal stridor or needs oxygen. Calcium supplements will be given to normalize calcium levels quickly, but oral supplements will take time to be absorbed.

Which assessment finding for a 33-year-old female patient admitted with Graves' disease requires the most rapid intervention by the nurse? a. Bilateral exophthalmos b. Heart rate 136 beats/minute c. Temperature 103.8° F (40.4° C) d. Blood pressure 166/100 mm Hg

ANS: C The patient's temperature indicates that the patient may have thyrotoxic crisis and that interventions to lower the temperature are needed immediately. The other findings also require intervention but do not indicate potentially life-threatening complications.

A 62-year-old patient with hyperthyroidism is to be treated with radioactive iodine (RAI). The nurse instructs the patient a. about radioactive precautions to take with all body secretions. b. that symptoms of hyperthyroidism should be relieved in about a week. c. that symptoms of hypothyroidism may occur as the RAI therapy takes effect. d. to discontinue the antithyroid medications taken before the radioactive therapy.

ANS: C There is a high incidence of postradiation hypothyroidism after RAI, and the patient should be monitored for symptoms of hypothyroidism. RAI has a delayed response, with the maximum effect not seen for 2 to 3 months, and the patient will continue to take antithyroid medications during this time. The therapeutic dose of radioactive iodine is low enough that no radiation safety precautions are needed.

A 42-year-old female patient is scheduled for transsphenoidal hypophysectomy to treat a pituitary adenoma. During preoperative teaching, the nurse instructs the patient about the need to a. cough and deep breathe every 2 hours postoperatively. b. remain on bed rest for the first 48 hours after the surgery. c. avoid brushing teeth for at least 10 days after the surgery. d. be positioned flat with sandbags at the head postoperatively.

ANS: C To avoid disruption of the suture line, the patient should avoid brushing the teeth for 10 days after surgery. It is not necessary to remain on bed rest after this surgery. Coughing is discouraged because it may cause leakage of cerebrospinal fluid (CSF) from the suture line. The head of the bed should be elevated 30 degrees to reduce pressure on the sella turcica and decrease the risk for headaches.

An 82-year-old patient in a long-term care facility has several medications prescribed. After the patient is newly diagnosed with hypothyroidism, the nurse will need to consult with the health care provider before administering a. docusate (Colace). b. ibuprofen (Motrin). c. diazepam (Valium). d. cefoxitin (Mefoxin).

ANS: C Worsening of mental status and myxedema coma can be precipitated by the use of sedatives, especially in older adults. The nurse should discuss the use of diazepam with the health care provider before administration. The other medications may be given safely to the patient.

A hospitalized diabetic patient received 38 U of NPH insulin at 7:00 AM. At 1:00 PM, the patient has been away from the nursing unit for 2 hours, missing the lunch delivery while awaiting a chest x-ray. To prevent hypoglycemia, the best action by the nurse is to a. save the lunch tray for the patient's later return to the unit. b. ask that diagnostic testing area staff to start a 5% dextrose IV. c. send a glass of milk or orange juice to the patient in the diagnostic testing area. d. request that if testing is further delayed, the patient be returned to the unit to eat.

ANS: D Consistency for mealtimes assists with regulation of blood glucose, so the best option is for the patient to have lunch at the usual time. Waiting to eat until after the procedure is likely to cause hypoglycemia. Administration of an IV solution is unnecessarily invasive for the patient. A glass of milk or juice will keep the patient from becoming hypoglycemic but will cause a rapid rise in blood glucose because of the rapid absorption of the simple carbohydrate in these items.

Which action by a patient indicates that the home health nurse's teaching about glargine and regular insulin has been successful? a. The patient administers the glargine 30 minutes before each meal. b. The patient's family prefills the syringes with the mix of insulins weekly. c. The patient draws up the regular insulin and then the glargine in the same syringe. d. The patient disposes of the open vials of glargine and regular insulin after 4 weeks.

ANS: D Insulin can be stored at room temperature for 4 weeks. Glargine should not be mixed with other insulins or prefilled and stored. Short-acting regular insulin is administered before meals, while glargine is given once daily.

Which finding indicates a need to contact the health care provider before the nurse administers metformin (Glucophage)? a. The patient's blood glucose level is 174 mg/dL. b. The patient has gained 2 lb (0.9 kg) since yesterday. c. The patient is scheduled for a chest x-ray in an hour. d. The patient's blood urea nitrogen (BUN) level is 52 mg/dL.

ANS: D The BUN indicates possible renal failure, and metformin should not be used in patients with renal failure. The other findings are not contraindications to the use of metformin.

A patient with type 2 diabetes is scheduled for a follow-up visit in the clinic several months from now. Which test will the nurse schedule to evaluate the effectiveness of treatment for the patient? a. Urine dipstick for glucose b. Oral glucose tolerance test c. Fasting blood glucose level d. Glycosylated hemoglobin level

ANS: D The glycosylated hemoglobin (A1C or HbA1C) test shows the overall control of glucose over 90 to 120 days. A fasting blood level indicates only the glucose level at one time. Urine glucose testing is not an accurate reflection of blood glucose level and does not reflect the glucose over a prolonged time. Oral glucose tolerance testing is done to diagnose diabetes, but is not used for monitoring glucose control once diabetes has been diagnosed.

The nurse has been teaching a patient with type 2 diabetes about managing blood glucose levels and taking glipizide (Glucotrol). Which patient statement indicates a need for additional teaching? a. "If I overeat at a meal, I will still take the usual dose of medication." b. "Other medications besides the Glucotrol may affect my blood sugar." c. "When I am ill, I may have to take insulin to control my blood sugar." d. "My diabetes won't cause complications because I don't need insulin."

ANS: D The patient should understand that type 2 diabetes places the patient at risk for many complications and that good glucose control is as important when taking oral agents as when using insulin. The other statements are accurate and indicate good understanding of the use of glipizide.

A 28-year-old male patient with type 1 diabetes reports how he manages his exercise and glucose control. Which behavior indicates that the nurse should implement additional teaching? a. The patient always carries hard candies when engaging in exercise. b. The patient goes for a vigorous walk when his glucose is 200 mg/dL. c. The patient has a peanut butter sandwich before going for a bicycle ride. d. The patient increases daily exercise when ketones are present in the urine.

ANS: D When the patient is ketotic, exercise may result in an increase in blood glucose level. Type 1 diabetic patients should be taught to avoid exercise when ketosis is present. The other statements are correct.

Which information is most important for the nurse to communicate rapidly to the health care provider about a patient admitted with possible syndrome of inappropriate antidiuretic hormone (SIADH)? a. The patient has a recent weight gain of 9 lb. b. The patient complains of dyspnea with activity. c. The patient has a urine specific gravity of 1.025. d. The patient has a serum sodium level of 118 mEq/L.

ANS: D A serum sodium of less than 120 mEq/L increases the risk for complications such as seizures and needs rapid correction. The other data are not unusual for a patient with SIADH and do not indicate the need for rapid action

A 23-year-old patient is admitted with diabetes insipidus. Which action will be most appropriate for the registered nurse (RN) to delegate to an experienced licensed practical/vocational nurse (LPN/LVN)? a. Titrate the infusion of 5% dextrose in water. b. Teach the patient how to use desmopressin (DDAVP) nasal spray. c. Assess the patient's hydration status every 8 hours. d. Administer subcutaneous DDAVP.

ANS: D Administration of medications is included in LPN/LVN education and scope of practice. Assessments, patient teaching, and titrating fluid infusions are more complex skills and should be done by the RN.

Which finding by the nurse when assessing a patient with a large pituitary adenoma is most important to report to the health care provider? a. Changes in visual field b. Milk leaking from breasts c. Blood glucose 150 mg/dL d. Nausea and projectile vomiting

ANS: D Nausea and projectile vomiting may indicate increased intracranial pressure, which will require rapid actions for diagnosis and treatment. Changes in the visual field, elevated blood glucose, and galactorrhea are common with pituitary adenoma, but these do not require rapid action to prevent life-threatening complications.

Which vitamin is directly involved in the metabolism of the hormones secreted by the parathyroid? A. Vitamin C B. Vitamin D C. Vitamin K D. Vitamin B9

Answer B is correct. Vitamin D is related to absorption of calcium and phosphorus. A, C, and D are incorrect because they are not related to the absorption of calcium and phosphorus.

A client is admitted for treatment of hypoparathyroidism. Based on the client's diagnosis, the nurse would anticipate an order for: A. Potassium B. Magnesium C. Calcium D. Iron

Answer C is correct. The parathyroid is responsible for calcium and phosphorus absorption. Clients with hypoparathyroidism have hypocalcemia. Answers A, B, and D are not associated with hypoparathyroidism therefore they are incorrect.

A 44-year-old female patient with Cushing syndrome is admitted for adrenalectomy. Which intervention by the nurse will be most helpful for a nursing diagnosis of disturbed body image related to changes in appearance? a. Reassure the patient that the physical changes are very common in patients with Cushing syndrome. b. Discuss the use of diet and exercise in controlling the weight gain associated with Cushing syndrome. c. Teach the patient that the metabolic impact of Cushing syndrome is of more importance than appearance. d. Remind the patient that most of the physical changes caused by Cushing syndrome will resolve after surgery.

ANS: D The most reassuring communication to the patient is that the physical and emotional changes caused by the Cushing syndrome will resolve after hormone levels return to normal postoperatively. Reassurance that the physical changes are expected or that there are more serious physiologic problems associated with Cushing syndrome are not therapeutic responses. The patient's physiological changes are caused by the high hormone levels, not by the patient's diet or exercise choices.

A 27-year-old patient admitted with diabetic ketoacidosis (DKA) has a serum glucose level of 732 mg/dL and serum potassium level of 3.1 mEq/L. Which action prescribed by the health care provider should the nurse take first? a. Place the patient on a cardiac monitor. b. Administer IV potassium supplements. c. Obtain urine glucose and ketone levels. d.Start an insulin infusion at 0.1 units/kg/hr.

ANS:A Hypokalemia can lead to potentially fatal dysrhythmias such as ventricular tachycardia and ventricular fibrillation, which would be detected with electrocardiogram (ECG) monitoring. Because potassium must be infused over at least 1 hour, the nurse should initiate cardiac monitoring before infusion of potassium. Insulin should not be administered without cardiac monitoring because insulin infusion will further decrease potassium levels. Urine glucose and ketone levels are not urgently needed to manage the patient's care.

lack of aldosterone results in what disease

Addison's

ACTH stimulation test is to assess for

Addison's disease

Primary hyperparathyroidism is caused by:

Adenoma (benign tumor) in the parathyroid gland.

A client has just been admitted with a diagnosis of myxedema coma. If all of the following interventions were prescribed, the nurse should place highest priority on completing which action first?

Administering oxygen

How is Addisonian crisis managed?

Aggressive shock management. - high dose of hydrocortisone replacement - large vol. IV fluid to reverse hypotension & F/E imbalance

What's the function of aldosterone?

Aldosterone is a potent mineralocorticoid that maintains extracellular fluid volume

What are the effects of mineralcorticoids?

Aldosterone: retain Na+ and H2O, lose K+ Too much Aldosterone: FVE - K+ decreases Little Aldosterone: FVD- K+ increases

A client with acromegaly will most likely experience which symptom? A. Bone pain B. Frequent infections C. Fatigue D. Weight loss

Answer A is correct. Acromegaly is an increase in secretion of growth hormone. The growth hormones cause expansion and elongation of the bones. Answers B, C, and D are not directly associated with acromegaly, so they are incorrect.

The client with a suspected pituitary tumor will most likely exhibit symptoms of: A. Alteration in visual acuity B. Frequent diarrhea C. Alterations in blood glucose D. Urticaria

Answer A is correct. The pituitary is located in the middle of the skull adjacent to the optic nerve and brain. Pressure on the optic nerve can cause an increase in intracranial pressure. Clients frequently complain of headache, nausea, vomiting, and decreasing visual acuity as the intracranial pressure increases. B, C, and D are incorrect because they are not associated with a pituitary tumor.

A client with Cushing's syndrome should be instructed to: A. Avoid alcoholic beverages B. Limit the sodium in her diet C. Increase servings of dark green vegetables D. Limit the amount of protein in her diet

Answer B is correct. A client with Cushing's syndrome has adrenocortical hypersecretion, so she retains sodium and water. The client may drink alcohol in moderation, so answer A is incorrect, and there is no need to eat more green vegetables or limit protein, so answers C and D are incorrect.

Which item should be kept at the bedside of a client who has just returned from having a thyroidectomy? A. A padded tongue B. An endotracheal tube C. An airway D. A tracheostomy set

Answer D is correct. Laryngeal swelling is not uncommon in clients following a thyroidectomy. A tracheostomy tray should be kept available. The ventilator is not necessary, so answer A is incorrect. The endotracheal tube is very difficult, if not impossible, to intubate if swelling has already occurred, so answer B is incorrect. The airway will do no good because the swelling is in the trachea, so answer C is incorrect.

Acromegaly is associated w/ which endocrine gland?

Anterior pituitary

The nurse is reviewing the postoperative prescriptions for a client who had a transsphenoidal hypophysectomy. Which health care provider's prescription noted on the record indicates the need for clarification?

Apply a loose dressing if any clear drainage is noted.

A 30-year-old patient seen in the emergency department for severe headache and acute confusion is found to have a serum sodium level of 118 mEq/L. The nurse will anticipate the need for which diagnostic test? a. Urinary 17-ketosteroids b. Antidiuretic hormone level c. Growth hormone stimulation test d. Adrenocorticotropic hormone level

B

A 35-year-old female patient with a possible pituitary adenoma is scheduled for a computed tomography (CT) scan with contrast media. Which patient information is most important for the nurse to communicate to the health care provider before the test? a. Bilateral poor peripheral vision b. Allergies to iodine and shellfish c. Recent weight loss of 20 pounds d. Complaint of ongoing headaches

B

A 40-year-old male patient has been newly diagnosed with type 2 diabetes mellitus. Which information about the patient will be most useful to the nurse who is helping the patient develop strategies for successful adaptation to this disease? a. Ideal weight b. Value system c. Activity level d. Visual changes

B

A nurse will teach a patient who is scheduled to complete a 24-hour urine collection for 17-ketosteroids to a. insert and maintain a retention catheter. b. keep the specimen refrigerated or on ice. c. drink at least 3 L of fluid during the 24 hours. d. void and save that specimen to start the collection.

B

During the physical examination of a 36-year-old female, the nurse finds that the patient's thyroid gland cannot be palpated. The most appropriate action by the nurse is to a. palpate the patient's neck more deeply. b. document that the thyroid was nonpalpable. c. notify the health care provider immediately. d. teach the patient about thyroid hormone testing.

B

The nurse is caring for a 45-year-old male patient during a water deprivation test. Which finding is most important for the nurse to communicate to the health care provider? a. The patient complains of intense thirst. b. The patient has a 5-lb (2.3 kg) weight loss. c. The patient's urine osmolality does not increase. d. The patient feels dizzy when sitting on the edge of the bed.

B

Which action by a new registered nurse (RN) caring for a patient with a goiter and possible hyperthyroidism indicates that the charge nurse needs to do more teaching? a. The RN checks the blood pressure on both arms. b. The RN palpates the neck thoroughly to check thyroid size. c. The RN lowers the thermostat to decrease the temperature in the room. d. The RN orders nonmedicated eye drops to lubricate the patient's bulging eyes.

B

Which information about a 30-year-old patient who is scheduled for an oral glucose tolerance test should be reported to the health care provider before starting the test? a. The patient reports having occasional orthostatic dizziness. b. The patient takes oral corticosteroids for rheumatoid arthritis. c. The patient has had a 10-pound weight gain in the last month. d. The patient drank several glasses of water an hour previously.

B

11. The most common benign tumor of the pituitary gland is a: a. Glioma b Prolactinoma c. Carcinoid tumor d. Islet cell tumor

B: Prolactinomas can cause symptoms by releasing excessive amounts of prolactin into the blood or mechanically by pressing on surrounding tissues. In women, symptoms may include menstrual irregularities and infertility; in men erectile dysfunction and libido may be impaired.

When caring for a patient with nephrogenic diabetes insipidus, the nurse would expect treatment to include a. fluid restriction b. thiazide diuretics c. a high sodium diet d. chlorprpamide (Diabinese)

B- In nephrogenic diabetes insipidus, the kidney is unable to respond to ADH, so vasopressin or hormone analogs are not effective. Thiazide diuretics slow the glomerular filtration rate in the kidney and produce a decrease in urine output. Low sodium diets (<3g/day) are also thought to decrease urine output. Fluids are not restricted because the patient could easily become dehydrated

A patient with SIADH is treated with water restriction and administration of IV fluids. The nurse evaluates that treatment has been effective when the patient experiences a. increased urine output, decreased serum sodium, and increased urine specific gravity b. increased urine output, increased serum sodium, and decreased urine specific gravity c. decreased urine output, increased serum sodium, and decreased urine specific gravity d. decreased urine output, decreased serum sodium, and increased urine specific gravity

B- The patient with SIADH has water retention with hyponatremia, decreased urine output, and concentrated urine with high specific gravity. Improvement in the patient's condition is reflected by increased urine output, normalization of serum sodium, and more water in the urine, decreasing specific gravity.

13. Early this morning, a female client had a subtotal thyroidectomy. During evening rounds, nurse Tina assesses the client, who now has nausea, a temperature of 105° F (40.5° C), tachycardia, and extreme restlessness. What is the most likely cause of these signs? a. Diabetic ketoacidosis b. Thyroid crisis c. Hypoglycemia d. Tetany

B. Thyroid crisis usually occurs in the first 12 hours after thyroidectomy and causes exaggerated signs of hyperthyroidism, such as high fever, tachycardia, and extreme restlessness. Diabetic ketoacidosis is more likely to produce polyuria, polydipsia, and polyphagia; hypoglycemia, to produce weakness, tremors, profuse perspiration, and hunger. Tetany typically causes uncontrollable muscle spasms, stridor, cyanosis, and possibly asphyxia.

in patients with pheochromocytoma there is a release of

high levels of epinephrine and norepinephrine

5. Nurse Oliver should expect a client with hypothyroidism to report which health concerns? a. Increased appetite and weight loss b. Puffiness of the face and hands c. Nervousness and tremors d. Thyroid gland swelling

B. Hypothyroidism (myxedema) causes facial puffiness, extremity edema, and weight gain. Signs and symptoms of hyperthyroidism (Graves' disease) include an increased appetite, weight loss, nervousness, tremors, and thyroid gland enlargement (goiter).

You are the nursing student and you are taking care of the patient recently diagnosed with hyperthyroidism. You can visibly see the thryroid gland enlarged. What is the next thing you should assess? A) Assess for a bruit or a thrill B) Listen to respiratory rate and look at SpO2 C) Check for peripheral pulses D) Poke the neck to see if it is soft or hard

B. Because an enlarged thyroid gland can constrict the neck (often caused by a goiter) it is very important to monitor the patient's respiratory status. After that a bruit or thrill may be felt for over the distended thyroid gland. Palpating to see if the neck is soft or hard may also be done, as a hard nodule may be indicative thyroid cancer.

Which of the following would the nurse most expect to find in the patient with hypoparathyroidism? A) Hypertension and cramps in extremities B) Dysphagia and stiffness in hands or feet C) Positive Trousseau's sign and profuse sweating D) Increased Phosphorous levels and tremors

B. Dysphagia related to laryngeal spasms and stiffness or camping in extremities would be expected signs and symptoms of hypoparathyroidism. Hypertension, profuse sweating, and increased phosphorous levels would not be expected with this condition.

Which assessment parameter is of highest priority when caring for a patient undergoing a water deprivation test? A. Serum glucose B. Patient weight C. Arterial blood gases D. Patient temperature

B. Patient weight A patient is at risk for severe dehydration during a water deprivation test. The test should be discontinued and the patient rehydrated if the patient's weight drops more than 2 kg at any time.

Which assessment parameter is of highest priority when caring for a patient undergoing a water deprivation test? A. Serum glucose B. Patient weight C. Arterial blood gases D. Patient temperature

B. Patient weight A patient is at risk for severe dehydration during a water deprivation test. The test should be discontinued and the patient rehydrated if the patient's weight drops more than 2 kg at any time. The other assessment parameters do not assess fluid balance.

The nurse interviews a 50-year-old man with a history of type 2 diabetes mellitus, chronic bronchitis, and osteoarthritis who has a fasting blood glucose of 154 mg/dL. Which medications, if taken by the patient, may raise blood glucose levels? A. Glargine (Lantus) B. Prednisone (Deltasone) C. Metformin (Glucophage) D. Acetaminophen (Tylenol)

B. Prednisone (Deltasone) Prednisone is a corticosteroid that may cause glucose intolerance in susceptible patients by increasing gluconeogenesis and insulin resistance. Insulin (e.g., glargine) and metformin (an oral hypoglycemic agent) decrease blood glucose levels. Acetaminophen has a glucose-lowering effect.

Uh oh, the patient records got scrambled up. You need to go in and see each of the patients in the clinic to see which problem they have. Which of the following patients does the nurse suspect to be the patient with hyperparathyroidism? A) The patient with shakiness in his hands and c/o numbness and tingling in fingers and toes B) The patient c/o back pain and abdominal cramping with nausea C) The patient with a BP of 167/87 and HR 110 with profuse sweating D) The patient c/o of fatigue and depression with bronze skin pigmentation

B. Remember, the patient with hyperparathyroidism is experiencing symptoms r/t too much calcium in the bloodstream. That means they will have issues with Stones (Renal), Bones (bone pain, joint pain, osteoporosis), Moans (GI upset, abdominal cramps), and psychic moans (mental changes). Patient A might be a patient with hypoparathyroidism. Patient C might be a patient with Pheochromocytoma. Patient D might be a patient with Addison's disease

The nurse is counseling the patient with hypoparathyroidism about nutrition choices r/t her disease process. Which of the following trays demonstrates correct understanding of the teaching? A) Yogurt, cream of chicken soup, and apple sauce with sweet tea B) Fried salmon, green beans, and a white wheat bread roll C) Sausage and egg sandwich with whole wheat bread and coffee with creamer D) Turkey sandwich with lettuce, tomato, fruit cup and a glass of milk

B. The recommended diet for patients with hypoparathyroidism includes foods that are high in calcium yet low in phosphorous (remember the balancing act) Fish, green beans, and white wheat are recommended foods because they are low in phosphorous. Milk and dairy products, while are high in calcium are also high in phosphorous and are thus restricted. Hypoparathyroidism is as much an issue with too much phosphorus as calcium (although s/s are r/t hypocalcemia) Other restricted foods include eggs and spinach. Other foods encouraged include broccoli, cucumbers, and nondairy creamer.

14. All of the following organs may be affected by multiple endocrine neoplasia type 1 except: a. Parathyroid glands b. Kidneys c. Pancreas and Duodenum d. Pituitary gland

B: Kidneys Multiple endocrine neoplasia type 1, also known as Werner's syndrome, is a heritable disorder that causes tumors in endocrine glands and the duodenum. Although the tumors associated with multiple endocrine neoplasia type 1 are generally benign, they can produce symptoms chemically by releasing excessive amounts of hormones or mechanically by pressing on adjacent tissue.

diet requirements for Addison's patient

high salt, carbs, and protein with low potassium

4. An ACTH stimulation test is commonly used to diagnose: a. Grave's disease b. Adrenal insufficiency and Addison's disease c. Cystic fibrosis d. Hashimoto's disease

B: The ACTH stimulation test measures blood and urine cortisol before and after injection of ACTH. Persons with chronic adrenal insufficiency or Addison's disease generally do not respond with the expected increase in cortisol levels. An abnormal ACTH stimulation test may be followed with a CRH stimulation test to pinpoint the cause of adrenal insufficiency.

A client is admitted with a diagnosis of pheochromocytoma. The nurse should monitor which parameter to detect the most common sign of pheochromocytoma?

Blood pressure elevation

The nurse is caring for a client after a thyroidectomy and monitoring for signs of thyroid storm. The nurse determines that which sign/symptom is indicative that a thyroid storm may be occurring?

Blood pressure of 80/60 mmHg

A client with a pituitary tumor will undergo transsphenoidal hypophysectomy. The nurse reinforces which information in the preoperative teaching plan for the client?

Blowing the nose following surgery is prohibited.

A 29-year-old patient in the outpatient clinic will be scheduled for blood cortisol testing. Which instruction will the nurse provide? a. "Avoid adding any salt to your foods for 24 hours before the test." b. "You will need to lie down for 30 minutes before the blood is drawn." c. "Come to the laboratory to have the blood drawn early in the morning." d. "Do not have anything to eat or drink before the blood test is obtained."

C

A 44-year-old patient is admitted with tetany. Which laboratory value should the nurse monitor? a. Total protein b. Blood glucose c. Ionized calcium d. Serum phosphate

C

A 60-year-old patient is taking spironolactone (Aldactone), a drug that blocks the action of aldosterone on the kidney, for hypertension. The nurse will monitor for a. increased serum sodium. b. decreased urinary output. c. elevated serum potassium. d. evidence of fluid overload.

C

The nurse reviews a patient's glycosylated hemoglobin (Hb A1C) results to evaluate a. fasting preprandial glucose levels. b. glucose levels 2 hours after a meal. c. glucose control over the past 90 days. d. hypoglycemic episodes in the past 3 months.

C

The nurse will teach a patient to plan to minimize physical and emotional stress while the patient is undergoing a. a water deprivation test. b. testing for serum T3 and T4 levels. c. a 24-hour urine test for free cortisol. d. a radioactive iodine (I-131) uptake test.

C

Which additional information will the nurse need to consider when reviewing the laboratory results for a patient's total calcium level? a. The blood glucose is elevated. b. The phosphate level is normal. c. The serum albumin level is low. d. The magnesium level is normal.

C

Which laboratory value should the nurse review to determine whether a patient's hypothyroidism is caused by a problem with the anterior pituitary gland or with the thyroid gland? a. Thyroxine (T4) level b. Triiodothyronine (T3) level c. Thyroid-stimulating hormone (TSH) level d. Thyrotropin-releasing hormone (TRH) level

C

Which question will provide the most useful information to a nurse who is interviewing a patient about a possible thyroid disorder? a. "What methods do you use to help cope with stress?" b. "Have you experienced any blurring or double vision?" c. "Have you had a recent unplanned weight gain or loss?" d. "Do you have to get up at night to empty your bladder?"

C

A client presents with hypocalcemia, hyperphosphatemia, muscle cramps, and positive Trosseau's sign. What diagnosis does this support? Answers: A. Diabetes insipidus B. Conn's syndrome C. Hypoparathyroidism D. Acromegaly

C Hypoparathyroidism often leads to the symptoms mentioned. Conn's syndrome is an aldosterone-producing adenoma.

Which laboratory value should the nurse review to determine whether a patient's hypothyroidism is caused by a problem with the anterior pituitary gland or with the thyroid gland? a. Thyroxine (T4) level b. Triiodothyronine (T3) level c. Thyroid-stimulating hormone (TSH) level d. Thyrotropin-releasing hormone (TRH) level

c. Thyroid-stimulating hormone (TSH) level

Majority of calories for a diabetic diet is from?

complex carbs, fats, proteins

glycosylated Hgb (Hgb A1c) levels indicate

complicance since the life span of a RBC is 4 mo

thyroid storm can quickly cause

congestive heart failure and pulmonary edema

People with hypothryoidism tend to have what?

CAD, constipation, heavy, moon face. Need to increase their fiber

What is the role of calcitonin?

Calcitonin _decreases_ serum Ca+ levesl by taking calcium out of _blood_ and pushing it back into _bones_?

What electrolyte does the parathyroid secrete?

Calcium

For a laryngospasm secondary to hypocalcemia, a nurse could administer:

Calcium gluconate

What atmosphere is most appropriate for a pt w/ thyroid storm?

Calm, quiet room. - eye care - HOB ^ and dark glassess

A patient is admitted to the hospital in thyrotoxic crisis. On physical assessment of the patient, the nurse would expect to find a. hoarseness and laryngeal stridor b. bulging eyeballs and dysrhythmias c. elevated temperature and signs of heart failure d. lethargy progressing suddenly to impairment of consciousness

C- A hyperthyroid crisis results in marked manifestations of hyperthyroidism, with severe tachycardia, heart failure, shock, hyperthermia, agitation, nausea, vomiting, diarrhea, delirium, and coma. Although exopthalmus may be present in the patient with Grave's disease, it is not a significant factor in hyperthyroid crisis. Hoarseness and laryngeal stridor are characteristic of tetany of hypoparathyroidism, and lethargy progressing to coma is characteristic of myxedema coma, a complication of hypothyroidism.

expected finding in new diagnosis of cushing's disease

hirsutism due to increased androgen production

A patient with diabetes insipidus is treated with nasal demospressin (DDAVP). The nurse determines that the drug is not having an adequate therapeutic effect when the patient experiences a. headache and weight gain b nasal irritation and nausea c. urine specific gravity of 1.002 d. an oral intake greater than urinary output

C- Normal urine specific gravity of 1.003 to 1.030, and urine with a specific gravity of 1.002 is very dilute, indicating that there continues to be excessive water loss and that treatment of diabetes insipidus is inadequate. Headache, weight gain, and oral intake greater than urinary output are signs of volume excess that occur with over medication. Nasal irritation and nausea may also indicate overdosage

During assessment of the patient with acromegaly, the nurse would expect the patient to report a. infertility b. dry, irritated skin c. undesirable changes in appearance d. an increase in height of 2 to 3 inches a year

C- the increased production of GH in acromegaly causes an increase in thickness and width of bones and enlargement of soft tissues, resulted in marked changes in facial features, oily and coarse skin, and speech difficulties. Height is not increased in adults with GH excess because the epiphyses of the bones are closed; infertility is not a common finding because GH is usually the only pituitary hormone involved in acromegaly

acute adrenal insufficiency requires treatment with

hydrocortisone sodium succinate

Synthroid requires careful monitoring of which patients?

Cardiac: - monitor apical heart rate (for dysrhythmias) - pulse

Acromegaly is caused by? And characterized by?

Cause: overproduction of GH S/S: Enlargement of hands, feet, bony & soft tissues of the head.

6. A female client with hypothyroidism (myxedema) is receiving levothyroxine (Synthroid), 25 mcg P.O. daily. Which finding should nurse Hans recognize as an adverse drug effect? a. Dysuria b. Leg cramps c. Tachycardia d. Blurred vision

C. Levothyroxine, a synthetic thyroid hormone, is given to a client with hypothyroidism to simulate the effects of thyroxine. Adverse effects of this agent include tachycardia. The other options aren't associated with levothyroxine.

21. An incoherent female client with a history of hypothyroidism is brought to the emergency department by the rescue squad. Physical and laboratory findings reveal hypothermia, hypoventilation, respiratory acidosis, bradycardia, hypotension, and nonpitting edema of the face and pretibial area. Knowing that these findings suggest severe hypothyroidism, nurse Libby prepares to take emergency action to prevent the potential complication of: a. Thyroid storm. b. Cretinism. c. myxedema coma. d. Hashimoto's thyroiditis.

C. Severe hypothyroidism may result in myxedema coma, in which a drastic drop in the metabolic rate causes decreased vital signs, hypoventilation (possibly leading to respiratory acidosis), and nonpitting edema. Thyroid storm is an acute complication of hyperthyroidism. Cretinism is a form of hypothyroidism that occurs in infants. Hashimoto's thyroiditis is a common chronic inflammatory disease of the thyroid gland in which autoimmune factors play a prominent role.

Following hypophysectomy, a client complains of being very thirsty and having to urinate frequently. Which is the initial nursing action?

Check the urine specific gravity.

Is the myxedema patient sensitive to heat or coolness?

coolness (they are always cold), give them blankets, but NEVER a heating pad because they can't tell if they are too hot)

ACTH adrenocorticotropic hormone measures

corticol response to to ACTH

16. A male client is admitted for treatment of the syndrome of inappropriate antidiuretic hormone (SIADH). Which nursing intervention is appropriate? a. Infusing I.V. fluids rapidly as ordered b. Encouraging increased oral intake c. Restricting fluids d. Administering glucose-containing I.V. fluids as ordered

C. To reduce water retention in a client with the SIADH, the nurse should restrict fluids. Administering fluids by any route would further increase the client's already heightened fluid load.

A nursing student is studying for a test on care of the client with endocrine disorders. Which of the following statements demonstrates an understanding of the difference between hyperthyroidism and hypothyroidism? a. "Deficient amounts of TH cause abnormalities in lipid metabolism, with decreased serum cholesterol and triglyceride levels." b. "Graves' disease is the most common cause of hypothyroidism." c. "Decreased renal blood flow and glomerular filtration rate reduces the kidney's ability to excrete water, which may cause hyponatremia." d. "Increased amounts of TH cause a decrease in cardiac output and peripheral blood flow."

C. "Decreased renal blood flow and glomerular filtration rate reduces the kidney's ability to excrete water, which may cause hyponatremia." Rationale: # 1 is incorrect because deficient amounts of TH cause abnormalities in lipid metabolism with elevated serum cholesterol and triglyceride levels. # 2 is incorrect because Graves' disease is the most common cause of hyperthyroidism, not hypothyroidism. # 4 is incorrect because increased amounts of TH cause an increase in cardiac output and peripheral blood flow.

When caring for the patient with a traumatic brain injury (TBI), the nurse knows that damage to which endocrine gland can affect the hormone secretion from some of the other endocrine glands? A. 1 (Pineal) B. 2 (Pituitary) C. 3 (Parathyroids) D. 4 (Thyroid)

C. 3 (Parathyroids) With a TBI, the anterior pituitary is likely to be damaged. The anterior pituitary gland secrets tropic hormones that control the secretion of hormones by other endocrine glands (the thyroid, adrenal cortex, and reproductive organs). The parathyroids secrete parathyroid hormone that regulates serum calcium level by acting on bone, the kidneys, and indirectly the gastrointestinal tract. The pineal gland secretes melatonin that helps regulate circadian rhythm and reproduction. The thyroid glands secrete thyroxine (T4), triiodothyronine (T3) that regulates the cell processes of cell growth and tissue differentiation, and calcitonin that affects bone tissue to regulate serum calcium and phosphorus levels.

You the nurse are taking care of the patient with hypoparathyroidism. Based on your knowledge of the disease process. Which of the following do you expect to be your priority in planning care for this patient? A) Acute Pain B) Fluid Volume Deficit C) Ineffective Airway Maintenance D) Impaired Cerebral Perfusion

C. Due to bronchospasm that can be caused by decreased calcium levels, it is important for the nurse to assess airway and breathing pattern. Signs associated with hypoparathyroidism are d/t hypocalcemia and the number one sign is tetany.

The nurse is caring for a group of older patients in a long-term care setting. Which physical changes in the patients should the nurse investigate as signs of possible endocrine dysfunction? A. Absent reflexes, diarrhea, and hearing loss B. Hypoglycemia, delirium, and incontinence C. Fatigue, constipation, and mental impairment D. Hypotension, heat intolerance, and bradycardia

C. Fatigue, constipation, and mental impairment Changes of aging often mimic clinical manifestations of endocrine disorders. Clinical manifestations of endocrine dysfunction such as fatigue, constipation, or mental impairment in the older adult are often missed because they are attributed solely to aging.

The patient with Addison's disease presents to the clinic saying "I've gained five pounds in the past week!" You note some edema in both lower extremities. What does the nurse suspect to be the problem? A) Precipitation of Addisonian Crisis B) Heart Failure d/t end stage Addison's disease C) Excessive hormone replacement dose D) Adrenocarcinoma development

C. Signs of too high a corticosteroid dose include edema and weight gain. S/S of an Addisonian crisis include hypotension, tachycardia, cyanosis, pallor, and tachypnea. Heart failure is not associated with Addison's disease. There is no evidence of a tumor in the adrenal glands based on these symptoms.

When instructing a patient regarding a urine study for free cortisol, what is most important for the nurse to tell the patient? A. Save the first voided urine in the morning. B. Maintain a high-sodium diet 3 days before collection. C. Try to avoid stressful situations during the collection period. D. Complete at least 30 minutes of exercise before collecting the urine sample.

C. Try to avoid stressful situations during the collection period. A urine study for free cortisol requires a 24-hour urine collection. The patient should be instructed to avoid stressful situations and excessive physical exercise that could unduly increase cortisol levels. The patient should also maintain a low-sodium diet before and during the urine collection period.

When instructing a patient regarding a urine study for free cortisol, what is most important for the nurse to tell the patient? A. Save the first voided urine in the morning. B. Maintain a high-sodium diet 3 days before collection. C. Try to avoid stressful situations during the collection period. D. Complete at least 30 minutes of exercise before collecting the urine sample.

C. Try to avoid stressful situations during the collection period. A urine study for free cortisol requires a 24-hour urine collection. The patient should be instructed to avoid stressful situations and excessive physical exercise that could unduly increase cortisol levels. The patient should also maintain a low-sodium diet before and during the urine collection period.

The nurse is caring for a 36-year-old woman with possible hypoparathyroidism after a thyroidectomy. It is most appropriate for the nurse to assess for which clinical manifestations? A. Polyuria, polydipsia, and weight loss B. Cardiac dysrhythmias and hypertension C. Muscle spasms and hyperactive deep tendon reflexes D. Hyperpigmentation, skin ulcers, and peripheral edema

C.Muscle spasms and hyperactive deep tendon reflexes Common assessment abnormalities associated with hypoparathyroidism include tetany (muscle spasms) and increased deep tendon reflexes. Hyperpigmentation is associated with Addison's disease. Skin ulcers occur in patient with diabetes. Edema is associated with hypothyroidism. Polyuria and polydipsia occur in patients with diabetes mellitus or diabetes insipidus. Weight loss occurs in hyperthyroidism or diabetic ketoacidosis. Hypertension and cardiac dysrhythmias may be caused by hyperthyroidism, hyperparathyroidism, or pheochromocytoma.

2. Grave's disease is: a. The most common cause of hypothyroidism b. The most common cause of hyperparathyroidism c. The most common cause of hyperthyroidism d. The most common cause of adrenal insufficiency

C: Grave's disease is an autoimmune disorder characterized by an enlarged thyroid gland and overproduction of thyroid hormones producing symptoms of hyperthyroidism such as rapid heartbeat, heat intolerance, agitation or irritability, weight loss, and trouble sleeping. It usually presents in persons age 20 to 40 and it is much more common in women than in men.

15. What is the treatment for hyperparathyroidism? a. Synthetic thyroid hormone b. Desiccated thyroid hormone c. Surgical removal of the glands d. Calcium and phosphate

C: When hyperparathyroidism requires treatment, surgery is the treatment of choice and is considered curative for 95% of cases. Because untreated hyperparathyroidism may elevate blood and urine levels of calcium and deplete phosphorus, bones and teeth may lose the minerals needed to remain strong.

Clients should take levothyroxine (Synthroid)

Clients should take this medication in the morning with a full glass of water. Levothyroxine should be taken on an empty stomach, at least one hour before any other medications or vitamins.

The nurse working on an endocrine nursing unit understands that which correct concept is used in planning care?

Clients who have hyperparathyroidism should be protected against falls.

Endocrine disorders often go unrecognized in the older adult because: a. symptoms are often attributed to aging b. older adults rarely have identifiable symptoms c. endocrine disorders are relatively rare in the older adult d. older adults usually have subclinical endocrine disorders that minimize symptoms

Correct answer: a Rationale: Assessment of the effects of aging on the endocrine system is difficult because the subtle changes of aging often mimic manifestations of endocrine disorders.

A patient has a serum sodium level of 152 mEq/L. The normal hormonal response to this situation is: A. release of ADH B. release of ACTH C. secretion of aldosterone D. secretion of corticotropin-releasing hormone

Correct answer: a Rationale: The most important stimulus of antidiuretic hormone (ADH) secretion is plasma osmolality, which is a measure of solute concentration in circulating blood. Plasma osmolality increases when there is a decrease in extracellular fluid or an increase in solute concentration. The increased plasma osmolality activates osmoreceptors, which are extremely sensitive, specialized neurons in the hypothalamus. These activated osmoreceptors stimulate ADH release. When ADH is released, the renal tubules reabsorb water, which causes urine to be more concentrated.

When obtaining subjective data from a patient during assessment of the endocrine system, the nurse asks specifically about: a. energy level b. intake of vitamin C C. employment history d. frequency of sexual intercourse

Correct answer: a Rationale: The nurse should ask about energy levels, particularly in comparison with the patient's past energy level. Fatigue and hyperactivity are two common problems associated with endocrine problems.

A characteristic common to all hormones is that they: A.circulate in the blood bound to plasma proteins B. influence cellular activity of specific target tissues C. accelerate the metabolic processes of all body cells D. enter a cell to alter the cell's metabolism or gene expression

Correct answer: b Rationale: A hormone is a chemical substance synthesized and secreted by a specific organ or tissue. Most hormones have common characteristics, including (1) secretion in small amounts at variable but predictable rates, (2) circulation through the blood, and (3) binding to specific cell receptors in the cell membrane or within the cell.

A patient is receiving radiation therapy for cancer of the kidney. The nurse monitors the patient for signs and symptoms of damage to the A. pancreas B. thyroid gland C. adrenal glands D. poster pituitary gland

Correct answer: c Rationale: The adrenal glands are small, paired, highly vascularized glands located on the upper portion of each kidney.

All cells in the body are believed to have intracellular receptors for: a. insulin b. glucagon c. growth hormone d. thyroid hormone

Correct answer: d Rationale: There are two types of receptors: those that are within the cell (e.g., steroid and thyroid hormone receptors) and those that are on the cell membrane (e.g., water-soluble hormone receptors). Thyroid hormone receptors are located inside the cell. Because these hormones are lipid soluble, they pass through the target cell membrane by passive diffusion and bind to receptor sites located in the cytoplasm or nucleus of the target cell.

An abnormal finding by the nurse during an endocrine assessment would be: (select all that apply) a. blood pressure of 100/70 mm Hg b. excessive facial hair on a woman c. soft, formed stool every other day d. 3-lb weight gain over last 6 months e. hyperpigmented coloration in the lower legs

Correct answers: b, e Rationale: Hirsutism (i.e., excessive facial hair on women) may indicate Cushing syndrome or prolactinoma, a pituitary tumor. Hyperpigmentation (i.e., darkening of the skin, particularly in creases and skin folds) may indicate Addison's disease, which is caused by increased secretion of melanocyte-stimulating hormone, or it may indicate acanthosis nigricans.

hyperparathyroidism is associated with

hypercalcemia

The doctor walks into the patient room and says. "You have pheochromocytoma" and leaves. The patient begins to cry and says, "What does that mean?" What is the best response by the nurse? A) It is a tumor in your pituitary gland. You are probably going to die. B) You seem upset. Tell me how you are feeling. C) Pheochromocytoma is a disorder of the adrenal glands where too little steroids are released into the blood stream D) This is a caused by a benign tumor in certain glands in your body located above your kidneys

D This is a caused by a benign tumor in certain glands in your body located above your kidneys

What is the word for myxedema that is present at birth

cretinism, this is very dangerous and can lead to slow mental and physical development if undetected

If you do not have any Parathormone in your body, your serum calcium level will be ____?

Decrease

A patient suspected of having acromegaly has an elevated plasma growth hormone (GH) level. In acromegaly, the nurse would also expect the patient's diagnostic results to include a. hyperinsulinemia b. a plasma glucose of < 70 mg/dL c. decreased GH levels with an oral glucose challenge test d. elevated serum somatomedin C (insulin like growth factor-1 (IGF-1))

D- A normal response to growth hormone (GH) secretion is stimulation of the liver to produce somatomedin C, or IGF-1, which stimulates growth of bones and soft tissues. The increased levels of somatomedin C normally inhibit GH, but in acromegaly, the pituitary gland secretes GH despite elevated IGF-1 levels. When both GH and IGF-1 levels are increased, overproduction of GH is confirmed. GH also causes elevation of blood glucose, and normally GH levels fall during an oral glucose challenge but not in acromegaly.

In a patient with central diabetes insipidus, administration of ADH during a water deprivation test will result in a(n) a. decrease in body weight b. increase in urinary output c. decrease in blood pressure d. increase in urine osmolality

D- A patient with diabetes insipidus has a deficiency of ADH with excessive loss of water from the kidney, hypovolemia, hypernatremia, and dilute urine with low specific gravity. When a vasopressin is administered, the symptoms are reversed, with water retention, decreased urinary output that increases urine osmolality, and an increase in BP

Causes of primary hypothyroidism in adults include a. malignant or benign thyroid nodules b. surgical removal or failure of pituitary gland c. surgical removal or radiation of thyroid gland d. autoimmune induced atrophy of the thryoid gland.

D- Both Grave's disease and Hashimoto's thyroiditis are autoimmune disorders that eventually destroy the thyroid gland, leading to primary hypothyroidism. Thyroid tumors most often result in hyperthyroidism. Secondary hyperthyroidism occurs as a result of pituitary failure, and iatrogenic hypothyroidism results from thyroidectomy or radiation of thyroid gland.

Causes of primary hypothyroidism in adults include a. malignant or benign thyroid nodules b. surgical removal or failure of pituitary gland c. surgical removal or radiation of thyroid gland d. autoimmune induced atrophy of the thryoid gland.

D- Both Grave's disease and Hashimoto's thyroiditis are autoimmune disorders that eventually destroy the thyroid gland, leading to primary hypothyroidism. Thyroid tumors most often result in hyperthyroidism. Secondary hyperthyroidism occurs as a result of pituitary failure, and iatrogenic hypothyroidism results from thyroidectomy or radiation of thyroid gland.

A patient with Grave's disease asks the nurse what caused the disorder. The best response by the nurse is, a. "The cause of Grave's disease is not known, although it is thought to be genetic" b. "It is usually associated with goiter formation from an iodine deficiency over a long period of time" c. "Antibodies develop against thyroid tissue and destroy it, causing a deficiency of thyroid hormones" d. "In genetically susceptible persons, antibodies are formed that cause excessive thyroid hormone secretion"

D- In Grave's disease, antibodies to the TSH receptor are formed, attach to the receptors, and stimulate the thyroid gland to release triiodothyronin (T3), thyroxine (T4), or both, creating hyperthyroidism. The disease is not directly genetic, but individuals appear to have a genetic susceptibility to develop autoimmune antibodies. Goiter formation from insufficient iodine intake is usually associated with hypothyroidism.

When providing discharge instructions to a patient following a subtotal thyroidectomy, the nurse advises the patient to a. never miss a daily dose of thyroid replacement therapy b. avoid regular exercises until thyroid function is normalized c. use warm saltwater gargles several times a day to relieve throat pain d. reduce caloric intake by at least half the amount taken before surgery

D- With the decrease in thyroid hormone postoperatively, calories need to be reduced substantially to prevent weight gain. When a patient has had a subtotal thyroidectomy, thyroid replacement therapy is not given because exogenous hormone inhibits pituitary production of TSH and delays or prevents the restoration of thyroid tissue regeneration. Regular exercise stimulates the thyroid gland and is encouraged. Saltwater gargles are use for dryness or irritation of the mouth and throat following radioactive iodine therapy.

9. The nurse is aware that the following is the most common cause of hyperaldosteronism? a. Excessive sodium intake b. A pituitary adenoma c. Deficient potassium intake d. An adrenal adenoma

D. An autonomous aldosterone-producing adenoma is the most common cause of hyperaldosteronism. Hyperplasia is the second most frequent cause. Aldosterone secretion is independent of sodium and potassium intake as well as of pituitary stimulation.

15. When assessing a male client with pheochromocytoma, a tumor of the adrenal medulla that secretes excessive catecholamine, nurse April is most likely to detect: a. a blood pressure of 130/70 mm Hg. b. a blood glucose level of 130 mg/dl. c. bradycardia. d. a blood pressure of 176/88 mm Hg.

D. Pheochromocytoma, a tumor of the adrenal medulla that secretes excessive catecholamine, causes hypertension, tachycardia, hyperglycemia, hypermetabolism, and weight loss. It isn't associated with the other options.

what is the diet for cushings pre-treatment?

Increase K+, Ca+, protein, Decrease Na+

If you have too much Parathormone in your body, your serum calcium level will be ____?

Increased

Does treatment of hyperthyroidism correct eye or vision problems?

NO

in a patient with addison's disease, they lose ___ and ___ and retain ___

Na and H2O ; potassium

normally aldosterone makes you retain __ and ___ and lose ___

Na and H2O ; potassium

25. During preoperative teaching for a female client who will undergo subtotal thyroidectomy, the nurse should include which statement? a. "The head of your bed must remain flat for 24 hours after surgery." b. "You should avoid deep breathing and coughing after surgery." c. "You won't be able to swallow for the first day or two." d. "You must avoid hyperextending your neck after surgery."

D. To prevent undue pressure on the surgical incision after subtotal thyroidectomy, the nurse should advise the client to avoid hyperextending the neck. The client may elevate the head of the bed as desired and should perform deep breathing and coughing to help prevent pneumonia. Subtotal thyroidectomy doesn't affect swallowing.

7. A 67-year-old male client has been complaining of sleeping more, increased urination, anorexia, weakness, irritability, depression, and bone pain that interferes with her going outdoors. Based on these assessment findings, nurse Richard would suspect which of the following disorders? a. Diabetes mellitus b. Diabetes insipidus c. Hypoparathyroidism d. Hyperparathyroidism

D. Hyperparathyroidism is most common in older women and is characterized by bone pain and weakness from excess parathyroid hormone (PTH). Clients also exhibit hypercaliuria-causing polyuria. While clients with diabetes mellitus and diabetes insipidus also have polyuria, they don't have bone pain and increased sleeping. Hypoparathyroidism is characterized by urinary frequency rather than polyuria.

What happens with SIADH?

Na falls, resulting in: - vomitting - muscle pain - twitching - seizures - cerebral edema - HA - coma

Central DI is caused by:

Neurogenic: - head trauma, brain surgery or tumor, CNS infection

Can Diabetes type 1 patients take oral hypoglycemic agents?

No b/c they have no insulin to produce

The hypothalamus secretes releasing hormones and inhibiting hormones. What is the target tissue of these releasing hormones and inhibiting hormones? A. Pineal B. Adrenal cortex C. Posterior pituitary D. Anterior pituitary

D. Anterior pituitary The anterior pituitary is the target tissue of the releasing hormones (corticotropin releasing hormone, thyrotropin releasing hormone, growth hormone releasing factor, gonadotropin releasing hormone, prolactin releasing factor) and the inhibiting hormones (somatostatin, prolactin inhibiting factor). The posterior pituitary releases antidiuretic hormone (ADH) in response to plasma osmolality changes that is not directly affected by the hypothalamus hormones.

Will client with HHNK have kussmaul respirations?

No they are not acidic!

When should a diabetic pt exercise?

exercise same time & amount daily, after eatting when blood sugar is at its highest.

The hypothalamus secretes releasing hormones and inhibiting hormones. What is the target tissue of these releasing hormones and inhibiting hormones? A. Pineal B. Adrenal cortex C. Posterior pituitary D. Anterior pituitary

D. Anterior pituitary The anterior pituitary is the target tissue of the releasing hormones (corticotropin releasing hormone, thyrotropin releasing hormone, growth hormone releasing factor, gonadotropin releasing hormone, prolactin releasing factor) and the inhibiting hormones (somatostatin, prolactin inhibiting factor). These hormones release or inhibit other hormones that affect the thyroid, adrenal cortex, pancreas, reproductive organs, and all body cells. The pineal gland is not directly affected by the releasing and inhibiting hormones from the hypothalamus. The posterior pituitary releases antidiuretic hormone (ADH) in response to plasma osmolality changes that is not directly affected by the hypothalamus hormones.

The patient diagnosed with SIADH (Syndrome of inappropriate antidiuretic hormone) has a sodium level of 129 mEq/L. He is receiving IV 3% saline at 100 mL/hr. Which of the following is the nurse most concerned about? A) The patient complains of cramps in his hands B) There is trace edema in the lower extremities C) HR 110 D) The patient is suddenly states "Who are you?"

D. Decreased LOC is a sign of CMP (Central Pontine Myelinlysis) An extremely life-threatening condition that can occur with patient receiving high levels of sodium rich IV fluids. It is a medical emergency and the infusion should be stopped immediately. Cramps in the hands and trace edema are being treated with the infusion of 3% saline solution. HR 110 is also important, but not as emergent as sudden confusion.

You are providing discharge teaching to the client with primary Addison's disease. Which statement, if made by the patient, indicates the need for further teaching? A) I will have to take my Florinef and prednisone for the rest of my life B) During hot weather I will be sure to eat lots of salty foods C) I should continue to monitor my blood pressure for any drops D) Because I can't tolerate the heat I will cancel my trip to the Bahamas but I can still go skiing

D. Pt. should avoid cold weather in order to prevent precipitation of an Addisonian crisis which can be caused by stress on the body. The patient should be taught that medication will be taken for the rest of the patient's life. Salt intake should be increased during hot weather d/t hyponatremia. A drop in blood pressure could be a sign of an impending Addisonian crisis

A patient's recent medical history is indicative of diabetes insipidus. The nurse would perform patient teaching related to which diagnostic test? A. Thyroid scan B. Fasting glucose test C. Oral glucose tolerance D. Water deprivation test

D. Water deprivation test A water deprivation test is used to diagnose the polyuria that accompanies diabetes insipidus.

A patient's recent medical history is indicative of diabetes insipidus. The nurse would perform patient teaching related to which diagnostic test? A. Thyroid scan B. Fasting glucose test C. Oral glucose tolerance D. Water deprivation test

D. Water deprivation test A water deprivation test is used to diagnose the polyuria that accompanies diabetes insipidus. Glucose tests and thyroid tests are not directly related to the diagnosis of diabetes insipidus.

An 18-year-old male patient is undergoing a growth hormone stimulation test. The nurse should monitor the patient for A. hypothermia. B. hypertension. C. hyperreflexia. D. hypoglycemia.

D. hypoglycemia. Insulin or arginine (agent that stimulates insulin secretion) is administered for a growth hormone stimulation test. The nurse should monitor the patient closely for hypoglycemia. Hypothermia and hypertension are not expected in response to insulin or arginine. Hyperreflexia is an autonomic complication of spinal cord injury.

5. All of the following are symptoms of Cushing's syndrome except: a. Severe fatigue and weakness b. Hypertension and elevated blood glucose c. A protruding hump between the shoulders d. Hair loss

D: Cushing's syndrome also may cause fragile, thin skin prone to bruises and stretch marks on the abdomen and thighs as well as excessive thirst and urination and mood changes such as depression and anxiety. Women who suffer from high levels of cortisol often have irregular menstrual cycles or amenorrhea and present with hair on their faces, necks, chests, abdomens, and thighs.

6. Which of the following conditions is caused by long-term exposure to high levels of cortisol? a. Addison's disease b. Crohn's disease c. Adrenal insufficiency d. Cushing's syndrome

D: Cushing's syndrome is a form of hypercortisolism. Risk factors for Cushing's syndrome are obesity, diabetes, and hypertension. Cushing's syndrome is most frequently diagnosed in persons ages 20 to 50 who have characteristic round faces, upper body obesity, large necks, and relatively thin limbs.

Major complication of Type 1 --> think

DKA

What does fruity breath describe?

DKA

when a patient with type 1 diabetes mellitus has a glucose greater than 300

DKA

What is myxedema coma?

Decompensated hypothyroidism: - not nec. precipitated by lower thyroid markers - changes in personality and hypothermia - very depressed physiologic state

what is the only type of insulin that can be given via IV?

Regular

The nurse is collecting data on a client with a diagnosis of hypothyroidism. Which of these behaviors, if present in the client's history, should the nurse determine as being likely related to the symptoms of this disorder?

Depression

The nurse is caring for a child with a diagnosis of diabetes insipidus. The nurse anticipates that the health care provider will prescribe which medications?

Desmopressin acetate (DDAVP)

The nurse is caring for a client diagnosed with hyperparathyroidism who is prescribed furosemide (Lasix). The nurse reinforces dietary instructions to the client. Which is an appropriate instruction?

Drink at least 2 to 3 L of fluid daily.

The nurse educator is asking the nursing student to recall the signs/symptoms of hypothyroidism. The nurse educator determines that the student understands this disorder if which are included in the student's response? Select all that apply

Dry skin Constipation Cold intolerance

How do you position pt post thyroidectomy?

Elevate HOB

Cushings is caused by:

Excess cortiocosteroids

When caring for a client diagnosed with pheochromocytoma, which information should the nurse know when assisting with planning care?

Excessive catecholamines are released.

After several diagnostic tests, a client is diagnosed with diabetes insipidus. The nurse understands that which signs/symptoms are indicative of this disorder?

Excessive thirst and urine output

When drawing up regular and NPH, which gets drawn first?

Regular = Clear -> Cloudy

hyponatremia, as serum sodium drops

extra water enters body cells and causes them to swell. Convulsions, shock, coma and death may occur with cerebral edema and increased brain cell volume.

The nurse is monitoring a client with Graves' disease for signs of thyrotoxic crisis (thyroid storm). Which signs and symptoms noted in the client should alert the nurse to the presence of this crisis? Select all that apply.

Fever Sweating Agitation

The nurse is caring for a client following an adrenalectomy and is monitoring for signs of adrenal insufficiency. Which are signs and symptoms related to adrenal insufficiency? Select all that apply.

Fever Weakness Hypotension Mental status changes

What mineralcorticoid drug is prescribed for addison's disease?

Fludrocortisone : aldosterone

What's the primary tx for SIADH? What else?

Fluid restriction - < 1,000mL/day - HOB flat to 10* - seizure precautions - oral hygiene - distractions to help tx thirst

What is radioactive Iodine and how do you give it?

Given PO in 1 dose, destroys thyroid cells, radioactive precautions includes staying away from babies for 24 hrs & don't kiss anyone for 24 hrs because they kill thyroid cells.

What are examples of oral anti-diabetic meds?

Glizipizide (Glucotrol) Metformin (Glucophage) Pioglitazone (Actos)

The nurse is caring for a client with pheochromocytoma. The client asks for a snack and something warm to drink. Which is the appropriate choice for this client to meet nutritional needs?

Graham crackers and warm milk

What is another way to describe hyperthyroidism (TOO MUCH ENERGY)?

Graves disease

s/s Cushing's

Growth arrest, skinny arms/legs because steroids break down fat & protein, thin skin ^ risk for infection, hyperglycemia, psychosis to depression, moon face, truncal obesity, buffalo hump, oily skin, woman with male traits, libido, high B/P, CHF, Wt gain, FVE

Major complications of Type II --> think

HHNK

s/s of pheochromocytoma?

High B/P, HR, Pulse; flushing & diaphoresis

The nurse is caring for a client following a thyroidectomy. The client tells the nurse that she is concerned because of voice hoarseness. The client asks the nurse whether the hoarseness will subside. Which statement by the nurse regarding the hoarseness is accurate?

The hoarseness is normal and will gradually subside.

Addison's disease (primary insufficiency) of the adrenal gland results from

failure to produce adequate levels of cortisol and aldosterone

The nurse is preparing to reinforce instructions to a client with Addison's disease regarding diet therapy. The nurse understands that which diet should be prescribed for this client?

High-sodium, high-carbohydrate diet

Assess for recurrent laryngeal nerve damage by listening for_____?

Hoareness and weak voice

Which signs/symptoms should the nurse expect to note when collecting data on a client with Addison's disease?

Hypotension and vomiting

Another word for myxedema

Hypothyroid

A nurse is reviewing discharge teaching with a client you has Cushing's syndrome. Which statement made by the client indicates that the instructions related to dietary management were understood

I can eat foods that contain potassium

A community health nurse visit a client at home. Prednisone 10 mg orally daily has been prescribed for a client and the nurse reinforces teaching for the client about the medication. What statement made by the client indicates that further teaching is necessary

I can take aspirin or my antihistamine if I need it

Thyroid storm should be managed in what setting? What should the nurse monitor?

ICU setting - dysrhythmias - decompensation - O2 status - IV fluid replacement

Treatment for hypoparathyroidism

IV Ca+ or phosphate binding drugs

A client is diagnosed with hyperparathyroidism. The nurse teaching the client about dietary alterations to manage the disorder tells the client to limit which food in the diet?

Ice cream

To treat a conscious adult with a serum glucose below 50 mg/dL, the nurse should prepare to administer 1 mg glucagon.

If the person is conscious and alert, the nurse can offer 15-20 g of carbohydrates (4 ounces of juice or regular soda, or 1 tablespoon honey) to the client. Glucagon is used if the client is unconscious.

Acromegaly is NOT the result of excess growth hormone secretion in children, IT IS IN ADULTS

In children, too much growth hormone causes gigantism, resulting in an abnormal increase in height and bone growth. Acromegaly occurs in adulthood.

When and how are steroids, like Prednisone, administered?

In the AM w/ food to mimic normal secretion.

Increased ACTH means?

Increased cortisol levels

The nurse is collecting data on a client admitted to the hospital with a diagnosis of myxedema. Which data collection technique would provide data necessary to support the admitting diagnosis?

Inspection of facial features

The nurse should expect to note which interventions in the plan of care for a client with hypothyroidism? Select all that apply.

Instruct the client about thyroid replacement therapy. Encourage the client to consume fluids and high-fiber foods in the diet. Instruct the client to contact the health care provider if episodes of chest pain occur.

The nurse is reinforcing discharge instructions to a client who had a unilateral adrenalectomy. Which information should be a component of the instructions?

Instructions about early signs of a wound infection

A client has an endocrine system dysfunction of the pancreas. The nurse anticipates that the client will exhibit impaired secretion of which substance?

Insulin

when worrying about DKA, what other electrolyte do we worry about (not glucose)?

K because insulin drives glucose & K+ out of the vascular space into the cell.

A nurses caring for a postoperative parathyroidectomy client. Which of the following would require the nurses immediate attention

Laryngeal strider

The nurse is caring for a postoperative parathyroidectomy client. Which would require the nurse's immediate attention?

Laryngeal stridor

What are some meds taken for myxedema?

Levothyroxine, Thyrglobulin, Liothyronine

Nephrogenic DI can be related to what drug tx?

Lithium (salt)

A client has been diagnosed with hypoparathyroidism. Which food groups should be included in the diet?

Low in phosphorus and high in calcium

What condition do we worry about with diabetes?

Metabolic acidosis

A client with hypoparathyroidism has hypocalcemia. The nurse avoids giving the client the prescribed vitamin and calcium supplement with which liquid?

Milk

After receiving furosemide (Lasix) 40 mg slow intravenous push for chest pain related to shortness of breath and generalized edema, the client responds poorly. The client has no relief of the chest pain, shortness of breath, or edema and only minimal urine output (less than 40 mL of urine). The health care provider is notified, and after reviewing the chart, suspects the client has syndrome of inappropriate antidiuretic hormone (SIADH). Which findings would lead to this specific diagnosis? Refer to chart.

Minimal responsiveness to furosemide (Lasix) and small cell lung cancer

If corticosteroids are taken long term, what should be monitored?

Monitor glucose

The nurse reviews a plan of care for a postoperative client following a thyroidectomy and notes that the client is at risk for breathing difficulty. Which nursing intervention should the nurse include in the plan of care?

Monitor neck circumference frequently.

The nurse is reviewing a plan of care for a client with Addison's disease. The nurse notes that the client is at risk for dehydration and suggests nursing interventions that will prevent this occurrence. Which nursing intervention is an appropriate component of the plan of care? Select all that apply.

Monitoring intake and output Monitoring for changes in mental status Encouraging fluid intake of at least 3000 mL/day

What regulates the blood level of calcium?

Parathyroid hormone or PTH, is secreted by the parathyroid glands in response to low serum calcium levels. This causes resorption of calcium from the bones, kidneys and GI tract.

Calcium has an inverse relationship with ______?

Phosphorous

The nurse is collecting data from a client who is being admitted to the hospital for a diagnostic workup for primary hyperparathyroidism. The nurse understands that which client complaint would be characteristic of this disorder?

Polyuria

The nurse is caring for a client with a diagnosis of hypoparathyroidism. The nurse reviews the client's laboratory results and notes that the calcium level is extremely low. The nurse should expect to note which sign/symptom on data collection?

Positive Trousseau's sign

SIADH is a disorder of what endocrine gland?

Posterior pituitary

What is pheochromocytoma?

Rare condition - tumor of adrenal medulla - produces excessive catcholamines (epi/norepi)

The nurse is collecting data regarding a client after a thyroidectomy and notes that the client has developed hoarseness and a weak voice. Which nursing action is appropriate?

Reassure the client that this is usually a temporary condition.

What is a major possible complication from radioactive iodine?

Rebound effect --> Thyroid storm (hyperthyroidism multiplied by 100)-> careful of MI

Graves is characterized by what pattern?

Remissions and exacerbations

Untreated hypothyroidism increases susceptibility to:

Sedatives

What is addisonian crisis?

Severe hypotension and vascular collapse

A health care provider has prescribed propylthiouracil (PTU) for a client with hyperthyroidism, and the nurse assists in developing a plan of care for the client. Which nursing measure would be included in the plan regarding this medication?

Signs and symptoms of hypothyroidism

The nurse is caring for a client with hypothyroidism who is overweight. Which food items should the nurse suggest to include in the plan?

Skim milk, apples, whole-grain bread, and cereal

Cushings syndrome is another word for too many__

Steroids

Why are diabetics prone to CAD?

Sugar destroys vessels just like fat

Myxedema is characterized by:

Swelling of the affected area r/t mucopolysaccharide deposits in the dermis. - pretibial is characteristic of Graves - mostly mysedema is assoc. w/ hypothyroid

What hormones does the thyroid produce?

T3, T4, calcitonin

Myxedema coma diagnostic tests

TSH, Free T4, cortisol

Post op care for a thyroidectomy

Teach pt how to support neck & put personal items close

S/S of Addsion's

anorexia/nausea hyperpigmentation- bronzing of skin, vitiligo- white patchy skin hypotension, decrease Na+, increased K+ hypoglycemic,

The nurse identifies a need for additional teaching when the patient who is self-monitoring blood glucose a. washes the puncture site using warm water and soap. b. chooses a puncture site in the center of the finger pad. c. hangs the arm down for a minute before puncturing the site. d. says the result of 120 mg indicates good blood sugar control.

The nurse identifies a need for additional teaching when the patient who is self-monitoring blood glucose a. washes the puncture site using warm water and soap. b. chooses a puncture site in the center of the finger pad. c. hangs the arm down for a minute before puncturing the site. d. says the result of 120 mg indicates good blood sugar control. ANS: B The patient is taught to choose a puncture site at the side of the finger pad because there are fewer nerve endings along the side of the finger pad. The other patient actions indicate that teaching has been effective.

An older client with a history of hyperparathyroidism and severe osteoporosis is hospitalized. The nurse caring for the client plans to address which problem first?

The possibility of injury

What do Potassium Iodine (SSKI) and Strong Iodine solution (Lugol's solution) do?

These are Iodine compounds, they decrease the size and vascularity of the gland

What are toxic nodular goiters?

Thyroid secreting nodules functioning independently of TSH stimulation. - assoc. w/ hyperthyroid

The nurse is monitoring a client following a thyroidectomy for signs/symptoms of hypocalcemia. Which sign/symptom noted in the client indicates the presence of hypocalcemia?

Tingling around the mouth

The nurse caring for a client scheduled for a transsphenoidal hypophysectomy to remove a tumor in the pituitary gland assists in developing a plan of care for the client. The nurse suggests including which specific information in the preoperative teaching plan?

Toothbrushing will not be permitted for at least 2 weeks following surgery.

What's the tx for Acromegaly? Post-op treatment? Meds?

Transphenoidal hypophysectomy: - HOB at 30* - test nasal drainage for glucose Meds: - analgesics for HA - hormone replacement for life

Treatment for SIADH will focus on what? And nursing interventions will include?

Tx: fluid restriction (to dec. dilution of urine) - daily weight - I&Os

what is the diagnosis for pheochromocytoma?

VMA (vanillylmandelic acid) test: 24 Hr urine looking for increase epi & norepi (catecholamines) - throw out 1st urine, keeping last void

The nurse has reinforced dietary instructions to a client with a diagnosis of hypoparathyroidism. The nurse instructs the client to include which item in the diet?

Vegetables

An ophthamlic complication of hyperthydroidism is:

Vision loss - dry corneal surface from retracted eyelids

A preoperative client is scheduled for adrenalectomy to remove a pheochromocytoma. The nurse should most closely monitor which assessment in the preoperative period?

Vital signs

What is the most important thing to assess for addison patient when adjusting their medication?

Weight

when is calcium likely to be impaired

as a complication of thyroid removal

What patients should NEVER be given Beta blockers?

asthmatics or diaetics because they mask hypoglycemia

Hyperparathyroidsims= _________ = _________

_Hypercalcemia_ = _Hypophosphatemia_

Hyperparathyroidism means serum calcium is ____ and serum phosphate is ___

_high_, _low_

trousseau sign is seen with

a carpal spasm occurs when the upper arm is compressed, as by a tourniquet or a blood pressure cuff.

Hypopituitarism

a condition in which the pituitary gland does not produce normal amounts of some or all of its hormones (ACTH, ADH, FSH, GH, LH, oxytocin, prolactin, TSH).

Hypoparathyroidism

a disorder in which the parathyroid glands do not produce enough parathyroid hormone (PTH). PTH helps control serum levels of calcium, phosphorus, and vitamin D.

Cushing syndrome

a disorder that occurs when the body is exposed to high levels of the hormone cortisol.

An African American woman with a history of breast cancer has panhypopituitarism from radiation therapy for primary pituitary tumors. Which medications should the nurse teach her about needing for the rest of her life (select all that apply)? a. Cortisol b. Vasopressin c. Sex hormones d. Levothyroxine (Synthroid) e. Growth hormone (somatropin [Omnitrope]) f. Dopamine agonists (bromocriptine [Parlodel])

a, b, d, e. With panhypopituitarism, lifetime hormone replacement is needed for cortisol, vasopressin, thyroid, and GH. Sex hormones will not be replaced because of the patient's history of breast cancer. Dopamine agonists will not be used because they reduce secretion of GH, which has already been achieved with the radiation.

Which characteristics describe the use of RAI (select all that apply)? a. Often causes hypothyroidism over time b. Decreases release of thyroid hormones c. Blocks peripheral conversion of T4 to T3 d. Treatment of choice in nonpregnant adults e. Decreases thyroid secretion by damaging thyroid gland f. Often used with iodine to produce euthyroid before surgery

a, d, e. RAI causes hypothyroidism over time by damaging thyroid tissue and is the treatment of choice for nonpregnant adults. Potassium iodide decreases the release of thyroid hormones and decreases the size of the thyroid gland preoperatively. Propylthiouracil (PTU) blocks peripheral conversion of T4 to T3 and may be used with iodine to produce a euthyroid state before surgery.

As a precaution for vocal cord paralysis from damage to the recurrent laryngeal nerve during thyroidectomy surgery, what equipment should be in the room in case it is needed for this emergency situation? a. Tracheostomy tray c. IV calcium gluconate b. Oxygen equipment d. Paper and pencil for communication

a. A tracheostomy tray is in the room to use if vocal cord paralysis occurs from recurrent laryngeal nerve damage or for laryngeal stridor from tetany. The oxygen equipment may be useful but will not improve oxygenation with vocal cord paralysis without a tracheostomy. IV calcium salts will be used if hypocalcemia occurs from parathyroid damage. The paper and pencil for communication may be helpful, especially if a tracheostomy is performed, but will not aid in emergency oxygenation of the patient.

A patient with acromegaly is treated with a transsphenoidal hypophysectomy. What should the nurse do postoperatively? a. Ensure that any clear nasal drainage is tested for glucose. b. Maintain the patient flat in bed to prevent cerebrospinal fluid (CSF) leakage. c. Assist the patient with toothbrushing every 4 hours to keep the surgical area clean. d. Encourage deep breathing, coughing, and turning to prevent respiratory complications.

a. A transsphenoidal hypophysectomy involves entry into the sella turcica through an incision in the upper lip and gingiva into the floor of the nose and the sphenoid sinuses. Postoperative clear nasal drainage with glucose content indicates cerebrospinal fluid (CSF) leakage from an open connection to the brain, putting the patient at risk for meningitis. After surgery, the patient is positioned with the head elevated to avoid pressure on the sella turcica. Coughing and straining are avoided to prevent increased intracranial pressure and CSF leakage. Although mouth care is required every 4 hours, toothbrushing should not be performed because injury to the suture line may occur.

Which statement accurately describes Graves' disease? a. Exophthalmos occurs in Graves' disease. b. It is an uncommon form of hyperthyroidism. c. Manifestations of hyperthyroidism occur from tissue desensitization to the sympathetic nervous system. d. Diagnostic testing in the patient with Graves' disease will reveal an increased thyroid-stimulating hormone (TSH) level.

a. Exophthalmos or protrusion of the eyeballs may occur in Graves' disease from increased fat deposits and fluid in the orbital tissues and ocular muscles, forcing the eyeballs outward. Graves' disease is the most common form of hyperthyroidism. Increased metabolic rate and sensitivity of the sympathetic nervous system lead to the clinical manifestations. Thyroid-stimulating hormone (TSH) level is decreased in Graves' disease.

The nurse is caring for a 63-year-old with a possible pituitary tumor who is scheduled for a computed tomography (CT) scan with contrast. Which information about the patient is most important to discuss with the health care provider before the test? a. History of renal insufficiency b. Complains of chronic headache c. Recent bilateral visual field loss d. Blood glucose level of 134 mg/dL

a. History of renal insufficiency

When caring for a patient with primary hyperaldosteronism, the nurse would question a health care provider's prescription for which drug? a. Furosemide (Lasix) c. Spironolactone (Aldactone) b. Amiloride (Midamor) d. Aminoglutethimide (Cytadren)

a. Hyperaldosteronism is an excess of aldosterone, which is manifested by sodium and water retention and potassium excretion. Furosemide is a potassium-wasting diuretic that would increase the potassium deficiency. Aminoglutethimide blocks aldosterone synthesis. Spironolactone and amiloride are potassium-sparing diuretics.

The nurse is caring for a client who is about to undergo an adrenalectomy. Which of the following Preoperative interventions is most appropriate for this client? a. Maintain careful use of medical and surgical asepsis when providing care and treatments. b. Teach the client about a diet high in sodium to correct any potential sodium imbalances preoperatively. c. Explain to the client that electrolytes and glucose levels will be measured postoperatively. d. Teach the client how to effectively cough and deep breathe once surgery is complete.

a. Maintain careful use of medical and surgical asepsis when providing care and treatments. Rationale: Use careful medical and surgical asepsis when providing care and treatments since Cortisol excess increases the risk of infection. # 2 is incorrect. Nutrition should be addressed preoperatively. Request a dietary consultation to discuss with the client about a diet high in vitamins and proteins. If hypokalemia exists, include foods high in potassium. Glucocorticoid excess increases catabolism. Vitamins and proteins are necessary for tissue repair and wound healing following surgery. # 3 is incorrect. Monitor the results of laboratory tests of electrolytes and glucose levels. Electrolyte and glucose imbalances are corrected

A nurse on a general medical-surgical unit is caring for a client with Cushing's syndrome. Which of the following statements is correct about the medication regimen for Cushing's syndrome? a. Mitotane is used to treat metastatic adrenal cancer. b. Aminogluthimide may be administered to clients with ectopic ACTH-secreting tumors before surgery is performed. c. Ketoconazole increases cortisol synthesis by the adrenal cortex. d. Somatostatin analog increases ACTH secretion in some clients.

a. Mitotane is used to treat metastatic adrenal cancer. Rationale: Mitotane directly suppresses activity of the adrenal cortex and decreases peripheral metabolism of corticosteroids. It is used to treat metastatic adrenal cancer. # 2 is incorrect because aminogluthimide may be administered to clients with ectopic ACTH-secreting tumors that cannot be surgically removed. # 3 is incorrect because ketoconazole inhibits, not increases, cortisol synthesis by the adrenal cortex. # 4 is incorrect because somatostatin suppresses, not increases, ACTH secretion.

The nurse is caring for a client with pheochromocytoma. Which of the following must be included in planning the nursing care for this client ? a. Monitor blood pressure frequently, assessing for hypertension. b. Assess only for physical stressors present. c. Collect a random urine sample. d. Prepare the client for chemotherapy to shrink the tumor.

a. Monitor blood pressure frequently, assessing for hypertension. Rationale: Pheochromocytomas are tumors of chromaffin tissues in the adrenal medulla. These tumors which are usually benign produce catecholamines (epinephrine or norepinephrine) that stimulate the sympathetic nervous system. Although many organs are affected, the most dangerous effects are peripheral vasoconstriction and increased cardiac rate and contractility with resultant paroxysmal hypertension. Systolic blood pressure may rise to 200 to 300 mmHg, the diastolic to 150 to 175 mmHg. # 1 is correct because the careful monitoring of blood pressure is essential. Attacks are often precipitated by physical, emotional, or environmental stimuli, so # 2 is incorrect because more than physical stressors are considered. This condition is life threatening and is usually treated with surgery as the preferred treatment. # 3 is incorrect because it is a random sample and not a 24 hour urine collection. Because catecholamine secretion is episodic, a 24-hour urine is a better surveillance method than serum catecholamines. (Pagana & Pagana, 2002). Surgical removal of the tumor(s) by adrenalectomy is the treatment of choice. # 4 is incorrect because surgery would be the treatment usually completed.

A patient is admitted to the hospital with a diagnosis of Cushing syndrome. On physical assessment of the patient, what should the nurse expect to find? a. Hypertension, peripheral edema, and petechiae b. Weight loss, buffalo hump, and moon face with acne c. Abdominal and buttock striae, truncal obesity, and hypotension d. Anorexia, signs of dehydration, and hyperpigmentation of the skin

a. The effects of adrenocortical hormone excess, especially glucocorticoid excess, include weight gain from accumulation and redistribution of adipose tissue, sodium and water retention, glucose intolerance, protein wasting, loss of bone structure, loss of collagen, and capillary fragility leading to petechiae. Clinical manifestations of adrenocortical hormone deficiency include hypotension, dehydration, weight loss, and hyperpigmentation of the skin.

During care of the patient with SIADH, what should the nurse do? a. Monitor neurologic status at least every 2 hours. b. Teach the patient receiving treatment with diuretics to restrict sodium intake. c. Keep the head of the bed elevated to prevent antidiuretic hormone (ADH) release. d. Notify the health care provider if the patient's blood pressure decreases more than 20 mm Hg from baseline.

a. The patient with syndrome of inappropriate antidiuretic hormone (SIADH) has marked dilutional hyponatremia and should be monitored for decreased neurologic function and seizures every 2 hours. Sodium intake is supplemented because of the hyponatremia and sodium loss caused by diuretics. ADH release is reduced by keeping the head of the bed flat to increase left atrial filling pressure. A reduction in blood pressure (BP) indicates a reduction in total fluid volume and is an expected outcome of treatment.

What preoperative instruction should the nurse give to the patient scheduled for a subtotal thyroidectomy? a. How to support the head with the hands when turning in bed b. Coughing should be avoided to prevent pressure on the incisionm c. Head and neck will need to remain immobile until the incision heals d. Any tingling around the lips or in the fingers after surgery is expected and temporary

a. To prevent strain on the suture line postoperatively, the patient's head must be manually supported while turning and moving in bed but range-of-motion exercises for the head and neck are also taught preoperatively to be gradually implemented after surgery. There is no contraindication for coughing and deep breathing and these should be carried out postoperatively. Tingling around the lips or fingers is a sign of hypocalcemia, which may occur if the parathyroid glands are inadvertently removed during surgery. This sign should be reported immediately.

Identify the rationale for having the following items immediately available in the patient's room following thyroidectomy a. Tracheostomy tray b. Calcium salts for IV administration c. Oxygen equipment

a. To use in case airway obstruction occurs because of vocal cord paralysis from recurrent laryngeal nerve damage during surgery or laryngeal stridor occurs with tetany b. needed in case hypocalcemia occurs form parathyroid gland removal or damage during surgery, resulting in tetany c. in case of airway obstruction, laryngeal stridor or edema around trachea

Identify the rationale for having the following items immediately available in the patient's room following thyroidectomy a. Tracheostomy tray b. Calcium salts for IV administration c. Oxygen equipment

a. To use in case airway obstruction occurs because of vocal cord paralysis from recurrent laryngeal nerve damage during surgery or laryngeal stridor occurs with tetany b. needed in case hypocalcemia occurs form parathyroid gland removal or damage during surgery, resulting in tetany c. in case of airway obstruction, laryngeal stridor or edema around trachea

A 22-year-old patient is being seen in the clinic with increased secretion of the anterior pituitary hormones. The nurse would expect the laboratory results to show a. increased urinary cortisol. b. decreased serum thyroxine. c. elevated serum aldosterone levels. d. low urinary catecholamines excretion.

a. increased urinary cortisol.

A nurse is preparing a diet plan for a 50 yr old with simple goiter. Which of the following should be included in the clients diet to decrease the enlargement of he thyroid gland? a. iodine b. sodium c. potassium d. calcium

a. iodine

A patient who is on corticosteroid therapy for treatment of an autoimmune disorder has the following additional drugs ordered. How is the need for these drugs related to the effects of corticosteroids a. Furosemide (Lasix) b. Pantoprazole (Protonix) c. Alendronate (Fosamax) d. Insulin e. Potassium

a. sodium and fluid retention because of mineralcorticoid effect b. gastrointestinal irritation with an increase in secretion of pepsin and hydrochloric acid c. corticosteroid induced osteoperosis

chvosteks sigh is seen with

abnormal facial twitch when touching the facial nerve in front of the ear

Diabetes Type 1 appears__

abruptly, 1st sign may be DKA

In patients with primary adrenal insufficiency will have this result from ACTH stimulation test

absent or very decreased

Aldosterone is produced by the

adrenal glands

what is the treatment for cushing's syndrome?

adrenalectomy, quiet environment

Diabetes need a high ____ diet to slow down glucose absorption in intestines

fiber

A patient has been diagnosed with hypoparathyroidism. What manifestations should the nurse expect to observe (select all that apply)? a. Skeletal pain b. Dry, scaly skin c. Personality changes d. Abdominal cramping e. Cardiac dysrhythmias f. Muscle spasms and stiffness

b, c, d, e, f. In hypoparathyroidism the patient has inadequate circulating parathyroid hormone (PTH) that leads to hypocalcemia from the inability to maintain serum calcium levels. With hypocalcemia there is muscle stiffness and spasms, which can lead to cardiac dysrhythmias and abdominal cramps. There can also be personality and visual changes and dry, scaly skin.

A client newly diagnosed with Addison's disease is giving a return explanation of teaching done by the primary nurse. Which of the following statements indicates that further teaching is necessary? a. "I need to increase how much I drink each day." b. "I need to weigh myself if I think I am losing or gaining weight." c. "I need to maintain a diet high in sodium and low in potassium." d. "I need to take my medications each day."

b. "I need to weigh myself if I think I am losing or gaining weight." The client is at risk for ineffective therapeutic regimen management. Clients with Addison's disease must learn to provide lifelong self-care that involves varied components: medications, diet, and recognizing and responding to stress. Changes in lifestyle are difficult to maintain permanently. The client needs to take the medications on a daily basis. The client needs to perform daily weights to monitor for signs of dehydration. The client needs to maintain a diet high in sodium and low in potassium, as well as maintain an increased fluid intake. # 2 is incorrect because daily weights need to be performed instead of weighing when a problem is suspected.

What is an appropriate nursing intervention for the patient with hyperparathyroidism? a. Pad side rails as a seizure precaution. b. Increase fluid intake to 3000 to 4000 mL daily. c. Maintain bed rest to prevent pathologic fractures. d. Monitor the patient for Trousseau's and Chvostek's signs.

b. A high fluid intake is indicated in hyperparathyroidism to dilute the hypercalcemia and flush the kidneys so that calcium stone formation is reduced. Seizures are not associated with hyperparathyroidism. Impending tetany of hypoparathyroidism after parathyroidectomy can be noted with Trousseau's and Chvostek's signs. The patient with hyperparathyroidism is at risk for pathologic fractures resulting from decreased bone density but mobility is encouraged to promote bone calcification.

During discharge teaching for the patient with Addison's disease, which statement by the patient indicates that the nurse needs to do additional teaching? a. "I should always call the doctor if I develop vomiting or diarrhea." b. "If my weight goes down, my dosage of steroid is probably too high." c. "I should double or triple my steroid dose if I undergo rigorous physical exercise." d. "I need to carry an emergency kit with injectable hydrocortisone in case I can't take my medication by mouth."

b. A weight reduction in the patient with Addison's disease may indicate a fluid loss and a dose of replacement therapy that is too low rather than too high. Because vomiting and diarrhea are early signs of crisis and because fluid and electrolytes must be replaced, patients should notify their health care provider if these symptoms occur. Patients with Addison's disease are taught to take two to three times their usual dose of steroids if they become ill, have teeth extracted, or engage in rigorous physical activity and should always have injectable hydrocortisone available if oral doses cannot be taken.

Tx DKA

find cause, do HRly blood sugars & K+ checks ECG, HRly outputs IV insulin ABGs (watch metabolic acidosis) IVFs- start with NS then switch to D5W to prevent hypoglycemia anticipate Potassium in IV at some point

A somogyi effect happens when there is a is swing in insulin level

from very low during the night to very high in the am

During discharge teaching for the patient with Addison's disease, which statement by the patient indicates that the nurse needs to do additional teaching? ma. "I should always call the doctor if I develop vomiting or diarrhea." b. "If my weight goes down, my dosage of steroid is probably too high." c. "I should double or triple my steroid dose if I undergo rigorous physical exercise." d. "I need to carry an emergency kit with injectable hydrocortisone in case I can't take my medication by mouth."

b. A weight reduction in the patient with Addison's disease may indicate a fluid loss and a dose of replacement therapy that is too low rather than too high. Because vomiting and diarrhea are early signs of crisis and because fluid and electrolytes must be replaced, patients should notify their health care provider if these symptoms occur. Patients with Addison's disease are taught to take two to three times their usual dose of steroids if they become ill, have teeth extracted, or engage in rigorous physical activity and should always have injectable hydrocortisone available if oral doses cannot be taken.

A 35-year-old female patient with a possible pituitary adenoma is scheduled for a computed tomography (CT) scan with contrast media. Which patient information is most important for the nurse to communicate to the health care provider before the test? a. Bilateral poor peripheral vision b. Allergies to iodine and shellfish c. Recent weight loss of 20 pounds d. Complaint of ongoing headaches

b. Allergies to iodine and shellfish

A 30-year-old patient seen in the emergency department for severe headache and acute confusion is found to have a serum sodium level of 118 mEq/L. The nurse will anticipate the need for which diagnostic test? a. Urinary 17-ketosteroids b. Antidiuretic hormone level c. Growth hormone stimulation test d. Adrenocorticotropic hormone level

b. Antidiuretic hormone level

A patient with hypothyroidism is treated with levothyroxine (Synthroid). What should the nurse include when teaching the patient about this therapy? a. Explain that alternate-day dosage may be used if side effects occur. b. Provide written instruction for all information related to the drug therapy. c. Assure the patient that a return to normal function will occur with replacement therapy. d. Inform the patient that the drug must be taken until the hormone balance is reestablished.

b. Because of the mental sluggishness, inattentiveness, and memory loss that occur with hypothyroidism, it is important to provide written instructions and repeat information when teaching the patient. Replacement therapy must be taken for life and alternate-day dosing is not therapeutic. Although most patients return to a normal state with treatment, cardiovascular conditions and psychoses may persist.

The nurse has identified the nursing diagnosis of fatigue for a patient who is hypothyroid. What should the nurse do while caring for this patient? a. Monitor for changes in orientation, cognition, and behavior. b. Monitor for vital signs and cardiac rhythm response to activity. c. Monitor bowel movement frequency, consistency, shape, volume, and color. d. Assist in developing well-balanced meal plans consistent with level of energy expenditure.

b. Cardiorespiratory response to activity is important to monitor in this patient to determine the effect of activities and plan activity increases. Monitoring changes in orientation, cognition, and behavior are interventions for impaired memory. Monitoring bowels is needed to plan care for the patient with constipation. Assisting with meal planning will help the patient with imbalanced nutrition: more than body requirements to lose weight if needed.

When caring for a patient with nephrogenic diabetes insipidus, what should the nurse expect the treatment to include? a. Fluid restriction b. Thiazide diuretics c. A high-sodium diet d. Chlorpropamide (Diabinese)

b. In nephrogenic diabetes insipidus, the kidney is unable to respond to ADH, so vasopressin or hormone analogs are not effective. Thiazide diuretics slow the glomerular filtration rate (GFR) in the kidney and produce a decrease in urine output. Low-sodium diets (<3 g/day) are also thought to decrease urine output. Fluids are not restricted because the patient could easily become dehydrated.

Priority Decision: What is the priority nursing intervention during the management of the patient with pheochromocytoma? a. Administering IV fluids b. Monitoring blood pressure c. Administering β-adrenergic blockers d. Monitoring intake and output and daily weights

b. Pheochromocytoma is a catecholamine-producing tumor of the adrenal medulla, which may cause severe, episodic hypertension; severe, pounding headache; and profuse sweating. Monitoring for a dangerously high BP before surgery is critical, as is monitoring for BP fluctuations during medical and surgical treatment.

Priority Decision: What is the priority nursing intervention during the management of the patient with pheochromocytoma? a. Administering IV fluids c. Administering β-adrenergic blockers b. Monitoring blood pressure d. Monitoring intake and output and daily weights

b. Pheochromocytoma is a catecholamine-producing tumor of the adrenal medulla, which may cause severe, episodic hypertension; severe, pounding headache; and profuse sweating. Monitoring for a dangerously high BP before surgery is critical, as is monitoring for BP fluctuations during medical and surgical treatment.

When the patient with parathyroid disease experiences symptoms of hypocalcemia, what is a measure that can be used to temporarily raise serum calcium levels? a. Administer IV normal saline. b. Have patient rebreathe in a paper bag. c. Administer furosemide (Lasix) as ordered. d. Administer oral phosphorus supplements.

b. Rebreathing in a paper bag promotes carbon dioxide retention in the blood, which lowers pH and creates an acidosis. An acidemia enhances the solubility and ionization of calcium, increasing the proportion of total body calcium available in physiologically active form and relieving the symptoms of hypocalcemia. Saline promotes calcium excretion, as does furosemide. Phosphate levels in the blood are reciprocal to calcium and an increase in phosphate promotes calcium excretion.

When the patient with parathyroid disease experiences symptoms of hypocalcemia, what is a measure that can be used to temporarily raise serum calcium levels? a. Administer IV normal saline. c. Administer furosemide (Lasix) as ordered. b. Have patient rebreathe in a paper bag. d. Administer oral phosphorus supplements.

b. Rebreathing in a paper bag promotes carbon dioxide retention in the blood, which lowers pH and creates an acidosis. An acidemia enhances the solubility and ionization of calcium, increasing the proportion of total body calcium available in physiologically active form and relieving the symptoms of hypocalcemia. Saline promotes calcium excretion, as does furosemide. Phosphate levels in the blood are reciprocal to calcium and an increase in phosphate promotes calcium excretion.

A client presents to the emergency room with a history of Graves' disease. The client reports having symptoms for a few days, but has not previously sought or received any additional treatment. The client also reports having had a cold a few days back. Which of the following interventions would be appropriate to implement for this client, based on the history and current symptoms? Select all that apply. a. Administer aspirin b. Replace intravenous fluids c. Induce shivering d. Relieve respiratory distress e. Administer a cooling blanket

b. Replace intravenous fluids c. Induce shivering d. Relieve respiratory distress e. Administer a cooling blanket Rationale: Thyroid storm (also called thyroid crisis) is an extreme state of hyperthyroidism that is rare today because of improved diagnosis and treatment methods (Porth, 2005). When it does occur, those affected are usually people with untreated hyperthyroidism (most often Graves' disease) and people with hyperthyroidism who have experienced a stressor, such as an infection, trauma. The rapid increase in metabolic rate that results from the excessive TH causes the manifestations of thyroid storm. The manifestations include hyperthermia, with body temperatures ranging from 102°F (39°C) to 106°F (41°C); tachycardia; systolic hypertension; and gastrointestinal symptoms (abdominal pain, vomiting, diarrhea). Agitation, restlessness, and tremors are common, progressing to confusion, psychosis, delirium, and seizures. The mortality rate is high. Rapid treatment of thyroid storm is essential to preserve life. Treatment includes cooling without aspirin (which increases free TH) or inducing shivering, replacing fluids, glucose, and electrolytes, relieving respiratory distress, stabilizing cardiovascular function, and reducing TH synthesis and secretion. #1 is incorrect because cooling happens without the use of aspirin. All of the other choices are correct.

Which action by a new registered nurse (RN) caring for a patient with a goiter and possible hyperthyroidism indicates that the charge nurse needs to do more teaching? a. The RN checks the blood pressure on both arms. b. The RN palpates the neck thoroughly to check thyroid size. c. The RN lowers the thermostat to decrease the temperature in the room. d. The RN orders nonmedicated eye drops to lubricate the patient's bulging eyes.

b. The RN palpates the neck thoroughly to check thyroid size.

The nurse is caring for a 45-year-old male patient during a water deprivation test. Which finding is most important for the nurse to communicate to the health care provider? a. The patient complains of intense thirst. b. The patient has a 5-lb (2.3 kg) weight loss. c. The patient's urine osmolality does not increase. d. The patient feels dizzy when sitting on the edge of the bed.

b. The patient has a 5-lb (2.3 kg) weight loss.

A patient with SIADH is treated with water restriction. What does the patient experience when the nurse determines that treatment has been effective? a. Increased urine output, decreased serum sodium, and increased urine specific gravity b. Increased urine output, increased serum sodium, and decreased urine specific gravity c. Decreased urine output, increased serum sodium, and decreased urine specific gravity d. Decreased urine output, decreased serum sodium, and increased urine specific gravity

b. The patient with SIADH has water retention with hyponatremia, decreased urine output, and concentrated urine with high specific gravity. Improvement in the patient's condition is reflected by increased urine output, normalization of serum sodium, and more water in the urine, thus decreasing the specific gravity.

What might you find in the cushings pt's urine?

glucose & ketones

What happens to blood sugar when you are sick or stressed?

goes up - increase insulin to avoid DKA

pheochromocytoma symptoms are

headache, sweating, fast heartbeat, with markedly high blood pressure

What medication is used with thyrotoxicosis to block the effects of the sympathetic nervous stimulation of the thyroid hormones? a. Potassium iodide b. Atenolol (Tenormin) c. Propylthiouracil (PTU) d. Radioactive iodine (RAI)

b. The β-adrenergic blocker atenolol is used to block the sympathetic nervous system stimulation by thyroid hormones. Potassium iodide is used to prepare the patient for thyroidectomy or for treatment of thyrotoxic crisis to inhibit the synthesis of thyroid hormones. Antithyroid medications inhibit the synthesis of thyroid hormones. Radioactive iodine (RAI) therapy destroys thyroid tissue, which limits thyroid hormone secretion.

What medication is used with thyrotoxicosis to block the effects of the sympathetic nervous stimulation of the thyroid hormones? a. Potassium iodide c. Propylthiouracil (PTU) b. Atenolol (Tenormin) d. Radioactive iodine (RAI)

b. The β-adrenergic blocker atenolol is used to block the sympathetic nervous system stimulation by thyroid hormones. Potassium iodide is used to prepare the patient for thyroidectomy or for treatment of thyrotoxic crisis to inhibit the synthesis of thyroid hormones. Antithyroid medications inhibit the synthesis of thyroid hormones. Radioactive iodine (RAI) therapy destroys thyroid tissue, which limits thyroid hormone secretion.

A patient with Addison's disease comes to the emergency department with complaints of nausea, vomiting, diarrhea, and fever. What collaborative care should the nurse expect? a. IV administration of vasopressors b. IV administration of hydrocortisone c. IV administration of D5W with 20 mEq KCl d. Parenteral injections of adrenocorticotropic hormone (ACTH)

b. Vomiting and diarrhea are early indicators of Addisonian crisis and fever indicates an infection, which is causing additional stress for the patient. Treatment of a crisis requires immediate glucocorticoid replacement and IV hydrocortisone, fluids, sodium, and glucose are necessary for 24 hours. Addison's disease is a primary insufficiency of the adrenal gland and adrenocorticotropic hormone (ACTH) is not effective, nor would vasopressors be effective with the fluid deficiency of Addison's disease. Potassium levels are increased in Addison's disease and KCl would be contraindicated.

The patient is diagnosed with syndrome of inappropriate antidiuretic hormone (SIADH). What manifestation should the nurse expect to find? a. Decreased body weight b. Decreased urinary output c. Increased plasma osmolality d. Increased serum sodium levels

b. With increased antidiuretic hormone (ADH), the permeability of the renal distal tubules is increased, so water is reabsorbed into circulation. Decreased output of concentrated urine with increased urine osmolality and specific gravity occur. In addition, fluid retention with weight gain, serum hypoosmolality, dilutional hyponatremia, and hypochloremia occur.

The patient is diagnosed with syndrome of inappropriate antidiuretic hormone (SIADH). What manifestation should the nurse expect to find? a. Decreased body weight c. Increased plasma osmolality b. Decreased urinary output d. Increased serum sodium levels

b. With increased antidiuretic hormone (ADH), the permeability of the renal distal tubules is increased, so water is reabsorbed into circulation. Decreased output of concentrated urine with increased urine osmolality and specific gravity occur. In addition, fluid retention with weight gain, serum hypoosmolality, dilutional hyponatremia, and hypochloremia occur.

During the physical examination of a 36-year-old female, the nurse finds that the patient's thyroid gland cannot be palpated. The most appropriate action by the nurse is to a. palpate the patient's neck more deeply. b. document that the thyroid was nonpalpable. c. notify the health care provider immediately. d. teach the patient about thyroid hormone testing.

b. document that the thyroid was nonpalpable.

A nurse will teach a patient who is scheduled to complete a 24-hour urine collection for 17-ketosteroids to a. insert and maintain a retention catheter. b. keep the specimen refrigerated or on ice. c. drink at least 3 L of fluid during the 24 hours. d. void and save that specimen to start the collection.

b. keep the specimen refrigerated or on ice.

patient getting Levothyroxine, be alert for problems where?

heart, check apical pulse before giving

function of aldosterone is to

helps maintain BP, help the kidney maintain the water/sodium balance and excrete potassium.

why do diabetics have problem with infections

bacteria loves sugar

how does steroids affect bones

bad - osteoporosis because calcium is pulled from the bone

where should you check for bleeding with thyroid surgery?

behind the neck

in DKA ph and this are low

bicarb

a lab finding of metabolic ketoacidosis

bicarb less than 15

Glycosylated Hemoglobin (Hb A1c) is a ____ test, its gives an average of what your blood sugar has been over the past ___?

blood, 3 months

Diabetes type 2 is usually found ____

by accident, usually go to MD for a wound that wont heal or reptead vaginal infections

A 29-year-old patient in the outpatient clinic will be scheduled for blood cortisol testing. Which instruction will the nurse provide? a. "Avoid adding any salt to your foods for 24 hours before the test." b. "You will need to lie down for 30 minutes before the blood is drawn." c. "Come to the laboratory to have the blood drawn early in the morning." d. "Do not have anything to eat or drink before the blood test is obtained."

c. "Come to the laboratory to have the blood drawn early in the morning."

Which question will provide the most useful information to a nurse who is interviewing a patient about a possible thyroid disorder? a. "What methods do you use to help cope with stress?" b. "Have you experienced any blurring or double vision?" c. "Have you had a recent unplanned weight gain or loss?" d. "Do you have to get up at night to empty your bladder?"

c. "Have you had a recent unplanned weight gain or loss?"

A patient is admitted to the hospital with thyrotoxicosis. On physical assessment of the patient, what should the nurse expect to find? a. Hoarseness and laryngeal stridor b. Bulging eyeballs and dysrhythmias c. Elevated temperature and signs of heart failure d. Lethargy progressing suddenly to impairment of consciousness

c. A hyperthyroid crisis results in marked manifestations of hyperthyroidism, with severe tachycardia, heart failure, shock, hyperthermia, restlessness, irritability, abdominal pain, vomiting, diarrhea, delirium, and coma. Although exophthalmos may be present in the patient with Graves' disease, it is not a significant factor in hyperthyroid crisis. Hoarseness and laryngeal stridor are characteristic of the tetany of hypoparathyroidism and lethargy progressing to coma is characteristic of myxedema coma, a complication of hypothyroidism.

A patient who is on corticosteroid therapy for treatment of an autoimmune disorder has the following additional drugs ordered. Which one is used to prevent corticosteroid-induced osteoporosis? a. Potassium b. Furosemide (Lasix) c. Alendronate (Fosamax) d. Pantoprazole (Protonix)

c. Alendronate (Fosamax) is used to prevent corticosteroid- induced osteoporosis. Potassium is used to prevent the mineralocorticoid effect of hypokalemia. Furosemide (Lasix) is used to decrease sodium and fluid retention from the mineralocorticoid effect. Pantoprazole (Protonix) is used to prevent gastrointestinal (GI) irritation from an increase in secretion of pepsin and hydrochloric acid.

A patient who is on corticosteroid therapy for treatment of an autoimmune disorder has the following additional drugs ordered. Which one is used to prevent corticosteroid-induced osteoporosis? a. Potassium c. Alendronate (Fosamax) b. Furosemide (Lasix) d. Pantoprazole (Protonix)

c. Alendronate (Fosamax) is used to prevent corticosteroid- induced osteoporosis. Potassium is used to prevent the mineralocorticoid effect of hypokalemia. Furosemide (Lasix) is used to decrease sodium and fluid retention from the mineralocorticoid effect. Pantoprazole (Protonix) is used to prevent gastrointestinal (GI) irritation from an increase in secretion of pepsin and hydrochloric acid.

A patient is scheduled for a bilateral adrenalectomy. During the postoperative period, what should the nurse expect related to the administration of corticosteroids? a. Reduced to promote wound healing b. Withheld until symptoms of hypocortisolism appear c. Increased to promote an adequate response to the stress of surgery d. Reduced because excessive hormones are released during surgical manipulation of adrenal glands

c. Although the patient with Cushing syndrome has excess corticosteroids, removal of the glands and the stress of surgery require that high doses of corticosteroids (cortisone) be administered postoperatively for several days before weaning the dose. The nurse should monitor the patient's vital signs postoperatively to detect whether large amounts of hormones were released during surgical manipulation, obtain morning urine specimens for cortisol measurement to evaluate the effectiveness of the surgery, and provide dressing changes with aseptic technique to avoid infection as usual inflammatory responses are suppressed.

The nurse is teaching the patient with hyperthyroidism the importance of stress management. She is teaching how to do deep breathing exercises and listens to the patient's thoughts on coping mechanism and spirituality. The nurse knows this is an important part of this patients plan because: a) Knowing how to cope with stress will help this patient be able to work through some of the difficult treatments of this disease b) Every patient needs to know how to cope with stress c) Extra stress with the patient with hyperthyroidism can lead to life threatening complications d) Listening to the patients thoughts instead of imposing one's own thoughts encourages the patient to become self-reliant

c. Extra stress on the patient with hyperthyroidism can precipitate a life-threatening condition known as thyroid storm. It is important for the nurse to explain procedures to lower patient anxiety to prevent this condition.

Which of the following nursing implications is most important in a client being medicated for Addison's disease? a. Administer oral forms of the drug with food to minimize its ulcerogenic effect. b. Monitor capillary blood glucose for hypoglycemia in the diabetic client. c. Instruct the client to never abruptly discontinue the medication. d. Teach the client to consume a diet that is high in potassium, low in sodium, and high in protein.

c. Instruct the client to never abruptly discontinue the medication. Rationale: The primary medical treatment of Addison's disease is replacement of corticosteroids and mineralcorticoids, accompanied by increased sodium in the diet. The client needs to know the importance of maintaining a diet high is sodium and low in potassium. Medications should never be discontinued abruptly because crisis can ensue. Oral forms of the drug are given with food in Cushing's disease.

A 44-year-old patient is admitted with tetany. Which laboratory value should the nurse monitor? a. Total protein b. Blood glucose c. Ionized calcium d. Serum phosphate

c. Ionized calcium

A patient with diabetes insipidus is treated with nasal desmopressin acetate (DDAVP). The nurse determines that the drug is not having an adequate therapeutic effect when the patient experiences a. headache and weight gain. b. nasal irritation and nausea. c. a urine specific gravity of 1.002. d. an oral intake greater than urinary output.

c. Normal urine specific gravity is 1.005 to 1.025 and urine with a specific gravity of 1.002 is very dilute, indicating that there continues to be excessive loss of water and that treatment of diabetes insipidus is inadequate. Headache, weight gain, and oral intake greater than urinary output are signs of volume excess that occur with overmedication. Nasal irritation and nausea may also indicate overdosage.

A patient with diabetes insipidus is treated with nasal desmopressin acetate (DDAVP). The nurse determines that the drug is not having an adequate therapeutic effect when the patient experiences a. headache and weight gain. c. a urine specific gravity of 1.002. b. nasal irritation and nausea. d. an oral intake greater than urinary output.

c. Normal urine specific gravity is 1.005 to 1.025 and urine with a specific gravity of 1.002 is very dilute, indicating that there continues to be excessive loss of water and that treatment of diabetes insipidus is inadequate. Headache, weight gain, and oral intake greater than urinary output are signs of volume excess that occur with overmedication. Nasal irritation and nausea may also indicate overdosage.

Cushings pt with a 24 Hr urine test would find cortisol levels to be___?

high

what are s/s of hyperparathyroidism?

high Ca+, low P-, sedation

The patient with diabetes insipidus is brought to the emergency department with confusion and dehydration after excretion of a large volume of urine today even though several liters of fluid were drunk. What is a diagnostic test that the nurse should expect to be done to help make a diagnosis? a. Blood glucose b. Serum sodium level c. Urine specific gravity d. Computed tomography (CT) of the head

c. Patients with diabetes insipidus (DI) excrete large amounts of urine with a specific gravity of less than 1.005. Blood glucose would be tested to diagnose diabetes mellitus. The serum sodium level is expected to be low with DI but is not diagnostic. To diagnose central DI a water deprivation test is required. Then a CT of the head may be done to determine the cause. Nephrogenic DI is differentiated from central DI with determination of the level of ADH after an analog of ADH is given.

The patient with diabetes insipidus is brought to the emergency department with confusion and dehydration after excretion of a large volume of urine today even though several liters of fluid were drunk. What is a diagnostic test that the nurse should expect to be done to help make a diagnosis? a. Blood glucose c. Urine specific gravity b. Serum sodium level d. Computed tomography (CT) of the head

c. Patients with diabetes insipidus (DI) excrete large amounts of urine with a specific gravity of less than 1.005. Blood glucose would be tested to diagnose diabetes mellitus. The serum sodium level is expected to be low with DI but is not diagnostic. To diagnose central DI a water deprivation test is required. Then a CT of the head may be done to determine the cause. Nephrogenic DI is differentiated from central DI with determination of the level of ADH after an analog of ADH is given.

A patient with mild iatrogenic Cushing syndrome is on an alternate-day regimen of corticosteroid therapy. What does the nurse explain to the patient about this regimen? a. It maintains normal adrenal hormone balance. b. It prevents ACTH release from the pituitary gland. c. It minimizes hypothalamic-pituitary-adrenal suppression. d. It provides a more effective therapeutic effect of the drug.

c. Taking corticosteroids on an alternate-day schedule for pharmacologic purposes is less likely to suppress ACTH production from the pituitary and prevent adrenal atrophy. Normal adrenal hormone balance is not maintained during glucocorticoid therapy because excessive exogenous hormone is used.

A patient with hypoparathyroidism resulting from surgical treatment of hyperparathyroidism is preparing for discharge. What should the nurse teach the patient? a. Milk and milk products should be increased in the diet. b. Parenteral replacement of parathyroid hormone will be required for life. c. Calcium supplements with vitamin D can effectively maintain calcium balance. d. Bran and whole-grain foods should be used to prevent GI effects of replacement therapy.

c. The hypocalcemia that results from PTH deficiency is controlled with calcium and vitamin D supplementation and possibly oral phosphate binders. Replacement with PTH is not used because of antibody formation to PTH, the need for parenteral administration, and cost. Milk products, although good sources of calcium, also have high levels of phosphate, which reduce calcium absorption. Whole grains and foods containing oxalic acid also impair calcium absorption.

During assessment of the patient with acromegaly, what should the nurse expect the patient to report? a. Infertility b. Dry, irritated skin m c. Undesirable changes in appearance d. An increase in height of 2 to 3 inches a year

c. The increased production of GH in acromegaly causes an increase in thickness and width of bones and enlargement of soft tissues, resulting in marked changes in facial features, oily and coarse skin, and speech difficulties. Infertility is not a common finding because GH is usually the only pituitary hormone involved in acromegaly. Height is not increased in adults with GH excess because the epiphyses of the bones are closed.

Which additional information will the nurse need to consider when reviewing the laboratory results for a patient's total calcium level? a. The blood glucose is elevated. b. The phosphate level is normal. c. The serum albumin level is low. d. The magnesium level is normal.

c. The serum albumin level is low.

The nurse will teach a patient to plan to minimize physical and emotional stress while the patient is undergoing a. a water deprivation test. b. testing for serum T3 and T4 levels. c. a 24-hour urine test for free cortisol. d. a radioactive iodine (I-131) uptake test.

c. a 24-hour urine test for free cortisol.

A nurse is caring for a client with Addison's disease. Which of the following nursing considerations should be employed when caring for this client? a. avoid sodium in the clients diet b. monitor and protect skin integrity c. document the specific gravity of urine d. monitor increases in blood pressure

c. document the specific gravity of urine

A 60-year-old patient is taking spironolactone (Aldactone), a drug that blocks the action of aldosterone on the kidney, for hypertension. The nurse will monitor for a. increased serum sodium. b. decreased urinary output. c. elevated serum potassium. d. evidence of fluid overload.

c. elevated serum potassium.

Which of the following would be a nursing priority for a client just DX with Addison's disease? a. avioding unnecessary activity b. encouraging client to wear a med alert tag c. ensuring the client is adequatly hydrated d. explaining that the client will need life long hormone therapy

c. ensuring the client is adequatly hydrated

Steroids decrease ____ by excreting it through the GI tract?

calcium

parathyroid gland regulates

calcium, phospherus and magnesium balance within blood and bone

Regular insulin is ____ color?

clear

NPH is ___ color?

cloudy

s/s of hypoglycemia?

cold/clammy, confused, H/A, nausea shaky, irritable, nervous, increased pulse

A Clinical Instructor is questioning a student nurse about disorders of the parathyroid glands. Which statement by the nursing student, would indicate the need for further teaching? a. "Hyperparathyroidism results in an increased release of calcium and phosphorus by bones, with resultant bone decalcification." b. "Hyperparathyroidism results in deposits in soft tissues and the formation of renal calculi." c. "Hypoparathyroidism results in impaired renal tubular regulation of calcium and phosphate." d. "Hypoparathyroidism results in decreased activation of vitamin D which then results in decreased absorption of calcium by the pancreas."

d. "Hypoparathyroidism results in decreased activation of vitamin D which then results in decreased absorption of calcium by the pancreas." Rationale: Choices 1, 2, and 3 are all correct statements. # 4 demonstrates a need for further teaching because hypoparathyroidism results in decreased activation of vitamin D which then results in decreased absorption of calcium by the intestines, not the pancreas.

Which statement by a 50-year-old female patient indicates to the nurse that further assessment of thyroid function may be necessary? a. "I notice my breasts are tender lately." b. "I am so thirsty that I drink all day long." c. "I get up several times at night to urinate." d. "I feel a lump in my throat when I swallow."

d. "I feel a lump in my throat when I swallow."

An 18-year-old male patient with a small stature is scheduled for a growth hormone stimulation test. In preparation for the test, the nurse will obtain a. ice in a basin. b. glargine insulin. c. a cardiac monitor. d. 50% dextrose solution.

d. 50% dextrose solution.

A patient suspected of having acromegaly has an elevated plasma growth hormone (GH) level. In acromegaly, what would the nurse also expect the patient's diagnostic results to indicate? a. Hyperinsulinemia b. Plasma glucose of <70 mg/dL (3.9 mmol/L) c. Decreased GH levels with an oral glucose challenge test d. Elevated levels of plasma insulin-like growth factor-1 (IGF-1)

d. A normal response to growth hormone (GH) secretion is stimulation of the liver to produce somatomedin C, or insulin-like growth factor-1 (IGF-1), which stimulates growth of bones and soft tissues. The increased levels of somatomedin C normally inhibit GH but in acromegaly the pituitary gland secretes GH despite elevated IGF-1 levels. When both GH and IGF-1 levels are increased, overproduction of GH is confirmed. GH also causes elevation of blood glucose and normally GH levels fall during an oral glucose challenge but not in acromegaly.

In a patient with central diabetes insipidus, what will the administration of ADH during a water deprivation test result in? a. Decrease in body weight c. Decrease in blood pressure b. Increase in urinary output d. Increase in urine osmolality

d. A patient with central diabetes insipidus has a deficiency of ADH with excessive loss of water from the kidney, hypovolemia, hypernatremia, and dilute urine with a low specific gravity. When vasopressin is administered, the symptoms are reversed, with water retention, decreased urinary output that increases urine osmolality, and an increase in BP.

Priority Decision: When replacement therapy is started for a patient with long-standing hypothyroidism, what is most important for the nurse to monitor the patient for? a. Insomnia c. Nervousness b. Weight loss d. Dysrhythmias

d. All these manifestations may occur with treatment of hypothyroidism. However, as a result of the effects of hypothyroidism on the cardiovascular system, when thyroid replacement therapy is started myocardial oxygen consumption is increased and the resultant oxygen demand may cause angina, cardiac dysrhythmias, and heart failure, so monitoring for dysrhythmias is most important.

A nurse on a surgical floor is caring for a post-operative client who has just had a subtotal thyroidectomy. Which of the following assessments should be completed first on the client? a. Assess for signs of tetany by checking for Chvostek's and Trousseau's signs b. Assess dressing (if present) and the area under the client's neck and shoulders for drainage. c. Administer analgesic pain medications as ordered, and monitor their effectiveness. d. Assess respiratory rate, rhythm, depth, and effort.

d. Assess respiratory rate, rhythm, depth, and effort. Rationale: All of the above assessments have importance, but airway and breathing in a client should always be addressed first when prioritizing care. Assess for signs of latent tetany due to calcium deficiency, including tingling of toes, fingers, and lips; muscular twitches; positive Chvostek's and Trousseau's signs; and decreased serum calcium levels. However, tetany may occur in 1 to 7 days after thyroidectomy so # 1 is not the highest priority. Assessing for hemorrhage is always important, but the danger of hemorrhage is greatest in the first 12 to 24 hours after surgery, and as this client is immediately post operative it is not the main concern at this time. Pain medication is important but according to Maslow, pain is a psychosocial need to be addressed after a physiologic need.

What is a cause of primary hypothyroidism in adults? a. Malignant or benign thyroid nodules b. Surgical removal or failure of the pituitary gland c. Surgical removal or radiation of the thyroid gland d. Autoimmune-induced atrophy of the thyroid gland

d. Both Graves' disease and Hashimoto's thyroiditis are autoimmune disorders that eventually destroy the thyroid gland, leading to primary hypothyroidism. Thyroid tumors most often result in hyperthyroidism. Secondary hypothyroidism occurs as a result of pituitary failure and iatrogenic hypothyroidism results from thyroidectomy or radiation of the thyroid gland.

What findings are commonly found in a patient with a prolactinoma? a. Gynecomastia in men b. Profuse menstruation in women c. Excess follicle-stimulating hormone (FSH) and luteinizing hormone (LH) d. Signs of increased intracranial pressure, including headache, nausea, and vomiting

d. Compression of the optic chiasm can cause visual problems as well as signs of increased intracranial pressure, including headache, nausea, and vomiting. About 30% of prolactinomas will have excess prolactin secretion with manifestations of impotence in men, galactorrhea or amenorrhea in women without relationship to pregnancy, and decreased libido in both men and women. There is decreased follicle-stimulating hormone (FSH) and luteinizing hormone (LH).

A patient with Graves' disease asks the nurse what caused the disorder. What is the best response by the nurse? a. "The cause of Graves' disease is not known, although it is thought to be genetic." b. "It is usually associated with goiter formation from an iodine deficiency over a long period of time." c. "Antibodies develop against thyroid tissue and destroy it, causing a deficiency of thyroid hormones." d. "In genetically susceptible persons, antibodies are formed that cause excessive thyroid hormone secretion."

d. In Graves' disease, antibodies to the TSH receptor are formed, attach to the receptors, and stimulate the thyroid gland to release triiodothyronine (T3), thyroxine (T4), or both, creating hyperthyroidism. The disease is not directly genetic but individuals appear to have a genetic susceptibility to develop autoimmune antibodies. Goiter formation from insufficient iodine intake is usually associated with hypothyroidism.

What characteristic is related to Hashimoto's thyroiditis? a. Enlarged thyroid gland b. Viral-induced hyperthyroidism c. Bacterial or fungal infection of thyroid gland d. Chronic autoimmune thyroiditis with antibody destruction of thyroid tissue

d. In Hashimoto's thyroiditis, thyroid tissue is destroyed by autoimmune antibodies. An enlarged thyroid gland is a goiter. Viral-induced hyperthyroidism is subacute granulomatous thyroiditis. Acute thyroiditis is caused by bacterial or fungal infection.

A nurse is caring for a 60 yr old client affected with hypoparathyroidism. When checking the lab report, the nurse finds that the clients calcium level was very low. Which of the following vitamins regulates the calcium level in the body? a. A b. D c. E d. K

d. K

expected finding in patient with adrenal insufficiency

high calcium

A patient who recently had a calcium oxalate renal stone had a bone density study, which showed a decrease in her bone density. What endocrine problem could this patient have? a. SIADH c. Cushing syndrome b. Hypothyroidism d. Hyperparathyroidism

d. The patient with hyperparathyroidism may have calcium nephrolithiasis, skeletal pain, decreased bone density, psychomotor retardation, or cardiac dysrhythmias. The other endocrine problems would not be related to calcium kidney stones or decreased bone density.

When providing discharge instructions to a patient who had a subtotal thyroidectomy for hyperthyroidism, what should the nurse teach the patient? a. Never miss a daily dose of thyroid replacement therapy. b. Avoid regular exercise until thyroid function is normalized. c. Use warm saltwater gargles several times a day to relieve throat pain. d. Substantially reduce caloric intake compared to what was eaten before surgery.

d. With the decrease in thyroid hormone postoperatively, calories need to be reduced substantially to prevent weight gain. When a patient has had a subtotal thyroidectomy, thyroid replacement therapy is not given because exogenous hormone inhibits pituitary production of TSH and delays or prevents the restoration of thyroid tissue regeneration. Regular exercise stimulates the thyroid gland and is encouraged. Saltwater gargles are used for dryness and irritation of the mouth and throat following radioactive iodine therapy.

A 61-year-old female patient admitted with pneumonia has a total serum calcium level of 13.3 mg/dL (3.3 mmol/L). The nurse will anticipate the need to teach the patient about testing for _____ levels. a. calcitonin b. catecholamine c. thyroid hormone d. parathyroid hormone

d. parathyroid hormone

A client is admitted to the hospital with a medical DX of hyperthyroidism. When taking a history which information would be most significant? A. edema, intolerance to cold, lethargy b. peri-orbital edema, lethargy mask like face c. weight loss, intolerance to cold, muscle wasting d. weight loss, intolerance to heat, exophthalmos

d. weight loss, intolerance to heat, exophthalmos

how propanolol is used in treatment of thyroid storm

decrease rapid heart rate caused by excessive thyroid stimulation

demopressin acetate (DDVAP) is prescribed for the treatment of diabetes insipidus. The nurse monitors the client after medication administration for which therapeutic response

decreased urine output it (promotes renal conservation)

What medication can you anticipate administering to a pt w/ SIADH? What does it do?

demeclocycline (Declomycin) - induces nephrogenic DI, body stops responding to ADH and urine output increases

The water deprivation test is used to diagnose

diabetes insipidus.

What 2 vascular problems do you worry about with HHNK?

diabetic retinopathy & nephropathy

#1 rule of steroids

do not stop taking steroids abruptly!

early manifestation of hypoglycemia

drowsiness

After blood sugar goes up for this patient, then what should they do? what happens if they do not do this?

eat complex carb and protein (peanut butter on a cracker), if they don't do this, their blood sugar level will drop rapidly (bottom out)

expected finding for a patient with primary cushing's disease

elevated blood glucose

Beta Blockers stop the release of what?

epi and norepi (fight or flight)

SIADH is caused by

excessive release of ADH

result of excessive secretion of ADH

excessive retention of water and dilutional hyponatremia

in HHS the patient produces enough insulin to prevent ketosis but not enough to prevent

hyperglycemia; pH will be normal but glucose will be over 600


Related study sets

Frankenstein Complete Vocabulary Set with Definitions

View Set

Chapter 1: The basic of nutrition.

View Set

Peds Q's: Cognitive and Mental Health & Neurologic and Sensory Function

View Set

Chapter 2 Exam: Nature of Insurance, Risk, Perils and Hazards

View Set

Constitutional Law In Class Quizzes

View Set